Trauma Orthopedic MCQS online Bank

(OBQ14.208) When treating a proximal tibia fracture, the surgeon decides to (1) use blocking screws in the proximal fragment, and (2) pick the intramedullary nail based on the location of the Herzog curve. Which of the following combinations will best prevent the classic deformity associated with this fracture? 

 

  1. Place blocking screws medial and posterior to the nail. Use a nail with a Herzog curve proximal to the fracture site.

  2. Place blocking screws medial and posterior to the nail. Use a nail with a Herzog curve distal to the fracture site.

  3. Place blocking screws lateral and posterior to the nail. Use a nail with a Herzog curve proximal to the fracture site.

  4. Place blocking screws lateral and posterior to the nail. Use a nail with a Herzog curve distal to the fracture site.

  5. Place blocking screws lateral and anterior to the nail. Use a nail with a Herzog curve proximal to the fracture site.

 

PREFERRED RESPONSE 3

 

Proximal tibial fractures develop an apex anterior (procurvatum) and valgus malalignment. Blocking (poller) screws should be placed in the concavity of the deformity, thus posterior and lateral to the nail. The Herzog curve should be proximal to the fracture site.

 

Up to 58% of proximal tibial fractures are malaligned. Malalignment arises because the nail fits loosely in the wide metaphyses and cannot control alignment. Without close fit of the nail at the fracture site, the nail will not align the fracture independent of a stable reduction and careful nail path. Blocking screws serve to reduce the size of the proximal metaphyseal canal and guide final nail passage.

 

Stinner et al. discuss strategies in proximal tibial fracture nailing. They describe an accurate starting point (using the twin peaks AP view or fibular bisector AP view, and flat plateau lateral view). They emphasize fracture reduction prior to reaming and implant placement.

 

Hiesterman reviewed nailing of extra-articular proximal tibial fractures. Techniques described include blocking screws, unicortical plating, using a universal distractor, nailing in flexion/locking in extension, semiextended nailing (including percutaneous suprapatellar quads-splitting approaches), multiple proximal interlocking screws (>=3).

 

Illustration A shows placement of a coronal blocking screw. Illustration B shows placement of a sagittal blocking screw. Illustration C shows the effect of the Herzog curve. A more distal Herzog curve leads to a "wedge" effect and fracture displacement whereas a proximal Herzog curve contains the fracture. The "wedge" effect occurs as the nail is seated and impinges on the posterior cortex of the distal segment accentuating an apex anterior deformity because of the effective widening of the nail above the bend and posterior force on the distal segment to match the nail shape.

 

Incorrect Answers:

Answers 1, 2, 4, 5: Blocking screws in the proximal fragment should be lateral and posterior to the nail. The Herzog curve should be proximal to the fracture site.

 

(OBQ14.15) What would be the most appropriate surgical indication for transferring fascicles of the ulnar nerve to the motor nerve of the biceps and fascicles of the median nerve to the motor nerve of the brachialis? 

 

  1. C8 - T1 nerve root avulsion 3 months ago

  2. C5 - C6 nerve root avulsion 2 months ago

  3. Upper brachial plexus palsy 22 months ago

  4. Medial and posterior cord injury from gunshot wound 2 months ago

  5. C6 ASIA A spinal cord injury PREFERRED RESPONSE 2

Transfer of fascicles from (1) ulnar nerve to the nerve to the biceps and (2) median nerve to the motor nerve of the brachialis would be appropriate in the treatment of an acute (<3-6 months) upper brachial plexus palsy.

 

Upper trunk injury (C5, C6) often results from the avulsion of both the C5 and C6 nerve roots. Injuries of this nature usually result from a downward force on the shoulder with lateral bending of the cervical spine in the opposite direction. This results in what is commonly called an Erb-Duchenne palsy. Patients often present with a flail shoulder and loss of elbow flexion. Other common treatments for C5 and C6 root avulsion include neurotization of the musculocutaneous (MSC) nerve by the spinal accessory (SA) or intercostal nerve, and neurotization of the supra-scapular nerve by the SA.

 

Liverneaux et al. looked at short term results of (1) ulnar nerve fascicle transfer to the nerve to the biceps and (2) fascicle of the median nerve to the motor branch to the brachialis in 15 patients with acute C5 - C6 nerve root avulsion injuries. Grade 4 elbow flexion was restored in each of the 10 patients. There was no secondary deficit in grip strength or sensation.They concluded that this double nerve transfer technique will likely reduce the need for secondary procedures to augment elbow flexion.

 

Teboul et al. reviewed thirty-two patients with an upper nerve-root brachial plexus injury that underwent ulnar nerve fascicle transfer to the nerve of biceps to restore elbow flexion. After the nerve transfer, twenty-four patients achieved grade 3 elbow flexion strength or better. They note that this procedure will spare the C5 nerve root and other nerves for grafting or transfer elsewhere.

 

Illustration A shows harvesting of an ulnar nerve fascicle for transfer. Illustration B

shows transfer of the fascicle of the ulnar nerve to the motor nerve of the biceps.

 

Incorrect Answers:

Answer 1: C8 - T1 nerve root avulsion would result in ulnar nerve dysfunction. Transfer of the non-functional ulnar nerve to the motor nerve of the biceps would be a redundant procedure.

Answer 3: Nerve transfers in upper brachial plexus palsy more than 20-24 moths postinjury is a relative contraindication due to the eventual loss of neuromuscular end plates at 20 to 24 months after denervation. Free functioning muscle transfers are more commonly indicated in late presenting injuries. Answer 4: Medial and posterior cord injury from gunshot wound likely leave the musculocutaneous nerve intact (lateral cord). Therefore, this transfer technique would not be indicated. Answer 5: C6 ASIA A spinal cord injury would likely result in motor and sensory quadriplegia. Nerve transfers using the ulnar nerve (C8-T1) would also be redundant as this nerve would be non-functional in this patient.

 

(SBQ12FA.31) A 30-year-old male patient involved in a hang-gliding accident sustains a knee dislocation with multiligamentous knee injury and transection of his peroneal nerve. He undergoes multiple reconstructive surgeries. Two years later, he continues to have a foot drop and dynamic tendon transfer is recommended. This treatment most commonly involves transferring a tendon from which native insertion point to which new insertion point? Review Topic

 

  1. Plantar distal phalanges to medial navicular

  2. Medial navicular to dorsal lateral cuneiform

  3. Plantar 1st metatarsal to dorsal lateral cuneiform

  4. 5th metatarsal base to dorsal medial cuneiform

  5. Plantar distal phalanx of the hallux to dorsal distal phalanx of hallux

 

PREFERRED RESPONSE 2

 

Dynamic tendon transfer to restore active dorsiflexion of the foot involves transferring the posterior tibial tendon (PTT) insertion on the medial navicular to the dorsal lateral cuneiform.

 

Common peroneal nerve (CPN) injuries following traumatic knee dislocation are common, with an incidence of 25-40%. CPN palsy is characterized by foot drop due to loss of ankle dorsiflexors with a steppage gait and eventual development of a supinated equinovarus foot secondary to the unopposed pull of the PTT. Nonsurgical management involves use of an ankle-foot orthosis and physical therapy. Surgical

options include acute primary repair, nerve grafting with either autologous sural nerve or nerve conduits and dynamic tendon transfer. The PTT is harvested from its insertion at the navicular, passed through the interosseous membrane (IOM) and anchored to the lateral cuneiform (see Illustration A). The classic bridle procedure involves concomitant anastamosis of the PTT to the tibialis anterior (TA) and peroneus longus (PL) tendons.

 

Garozzo et al reported a case series of 62 patients with post-traumatic CPN palsy who underwent a one-stage procedure consisting of nerve repair and PTT transfer. Nerve repair combined with PTT transfer improved postoperative outcomes compared to nerve repair alone. At 2-year follow up, neural regeneration was demonstrated in 90% of patients. The authors hypothesized that poor outcomes following nerve repair alone are due to force imbalance between the functioning flexors and paralyzed extensors, which is somewhat equalized by performing a PTT transfer at time of repair.

 

Niall et al reviewed 55 patients with traumatic knee dislocation and reported a 41% incidence of CPN injury, exclusively associated with dislocations involving disruption of the posterior cruciate ligament (PCL) and posterolateral corner (PLC). Complete neurologic recovery was found in only 21% of patients. The best prognosis was found with lesions in continuity, less than 7cm of nerve involvement, and short conduction block and muscle activity on nerve conduction and EMG studies.

 

Vigasio et al described a dynamic tendon transfer technique for traumatic complete CPN injury, involving transfer of the PTT to the TA rerouted to a new origin at the lateral cuneiform to restore ankle dorsiflexion and flexor digitorum longus (FDL) to the extensor digitorum longus (EDL) and extensor hallucis longus (EHL) to restore digit dorsiflexion. Rerouting the TA towards the transferred PTT ensures the PTT harvest length is sufficient. This avoids excessive tensioning of the PTT, which may limit tendon excursion and result in a static tenodesis rather than dynamic function, as well as the need for PTT lengthening which may decrease strength of the transfer

 

Illustration A is a series of intraoperative photographs demonstrating PTT transfer from Garg et al. An incision is made distal to the medial malleolus and the PTT is harvested subperiosteally (A). The PTT is delivered through a second incision ~15cm proximal to the medial malleolus (B-C). The PTT is then passed through the interosseous membrane and out a third incision over the anterior fibula (D). Lastly, the PTT is passed through a fourth incision over the dorsal midfoot and anchored to the lateral cuneiform (E).

 

Incorrect Responses:

Answer 1: Transferring the FDL (insertion = plantar distal phalanges) to the medial navicular is used for correction of flexible flatfoot deformity arising from PTT insufficiency. Some surgeons transfer the FDL to the medial navicular at the time of PTT transfer to the dorsum of the foot, to compensate for loss of PTT function and minimize risk of flatfoot development. Answers 3: Transferring the PL (insertion = plantar 1st metatarsal) is not recommended, as this muscle is innervated by the CPN via the superficial peroneal nerve and therefore would not be functional. Answer 4: Transferring the peroneus brevis (PB; insertion = 5th metatarsal base) is not recommended, this muscle is innervated by the CPN via the superficial peroneal

nerve and therefore would not be functional. Answer 5: Transferring the flexor hallucis longus (FHL; insertion = plantar distal phalanx of the hallux) to the insertion of the EHL (dorsal distal phalanx of hallux) is recommended for correction of claw toe deformity and would not help restore foot dorsiflexion in this patient.

 

(SAE09TR.94) The teardrop shape marked with an asterisk in Figure 61 represents what anatomic structure? 

 

 

 

  1. Anterior superior iliac spine

  2. Sciatic buttress

  3. A column of bone running from the anterior inferior iliac spine (AIIS) to the posterior superior iliac spine (PSIS)

  4. The most superior portion of the roof of the acetabulum

  5. Iliopectineal line

 

PREFERRED RESPONSE 3

 

The teardrop can be visualized on the obturator outlet view of the pelvis and represents a thick column of bone that runs from the AIIS to the PSIS. Half pins for eternal fixation frames or screws can be inserted into this column for fixation of fractures.

 

(SAE09FA.58) Figure 26 shows the clinical photograph of a patient who has developed a residual limb ulcer following a traumatic transtibial amputation 2 years ago. What is the preferred treatment to resolve the ulcer? 

 

 

 

  1. Avoid wearing the prosthesis until the ulcer is healed and perform local wound care.

  2. Obtain a new prosthesis with an energy-storing foot to dampen impact.

  3. Perform local wound care in conjunction with modification of the prosthetic socket and cushioned liner.

  4. Excise the wound and advance the soft-tissue envelope.

  5. Perform a distal tibiofibular bone bridge and advance the soft-tissue envelope.

 

PREFERRED RESPONSE 3

 

The first step in the treatment of an amputation residual limb (stump) ulcer is local wound care and adjustment of the residual limb-prosthetic interface, as well as adjusting prosthetic alignment. Surgical revision should be undertaken only when prosthetic modification is unsuccessful.

 

(SAE08OS.180) A 24-year-old man involved in a motorcycle accident sustained multiple injuries including a closed tibia fracture, liver laceration, and blunt chest trauma. He has a blood pressure of 80/50 mm Hg, a pulse of 130, and respirations of

20. He is intubated and being resuscitated. The calf is very swollen with compartment pressures: anterior 25 mm Hg, lateral 24 mm Hg, deep posterior 21 mm Hg, and

superficial posterior 21 mm Hg. What is the most appropriate treatment? Review Topic

 

  1. Observation

  2. Splinting and limb elevation above the heart

  3. Splinting and no limb elevation

  4. Immediate fasciotomy

  5. Continuous pressure monitoring with an indwelling catheter

 

PREFERRED RESPONSE 4

 

A compartment syndrome is best diagnosed with a "deltaP" (diastolic pressure minus compartment pressure) of less than or equal to 30 mm Hg. This patient is hypotensive and the "deltaPs" are all less than 30 mm Hg. Emergent fasciotomy is the preferred treatment.

 

(OBQ15.240) Which of the following is an FDA approved adjunctive treatment for an acute open tibia fracture being treated with an intramedullary nail? 

 

  1. Calcitonin

  2. rhBMP-2

  3. rhBMP-7

  4. Teraparatide

  5. Bisphosphonates

 

PREFERRED RESPONSE 2

 

rhBMP-2 has FDA approval for use when treating acute open tibia fractures with an intramedullary nail.

 

Open tibial shaft fractures can present many treatment challenges. Although its use remains somewhat controversial, rhBMP-2 has been shown to have many positive effects when used to treat acute open tibia fractures. These benefits include accelerated early fracture healing, decreased rates of hardware failure, decreased need for subsequent bone grafting procedures, and decreased infection rates. rhBMP-2 does have FDA approval specifically for use in open tibia fractures being treated with an intramedullary nail.

 

Alt et al. present a comparison of patients with Grade III open tibia fractures treated

with un-reamed nails with or without rhBMP-2. They found significant decreases in need for secondary interventions such as bone grafting or nail exchange. Mean time to fracture healing was less in the rhBMP-2 group, but this difference was not statistically significant.

 

Govender et al. present a prospective randomized study of 450 patients with open tibia fractures treated with an intramedullary nail with or without rhBMP-2. They found statistically significant decreases in need for secondary intervention, hardware failure, and infection as well as faster wound healing and faster time to fracture union.

 

Wei et al. provide a meta analysis regarding use of rhBMP-2 in open tibia fractures. Due to decreased rates of secondary interventions they estimated a net savings of

$6,000 per case when rh-BMP2 was used. They found no significant difference in rates of infection, postoperative pain, hardware failure, or fracture healing at 20 weeks.

 

Incorrect answers:

Answers 1, 3, 4, 5: These products do not have specific FDA approval as adjunct treatments for acute open tibia fractures. rhBMP-7 does have FDA approval for use in closed tibial non-unions.

 

(OBQ14.93) A 28-year-old male sustains a fall on icy ground and fractures his ankle. An intraoperative fluoroscopy image is shown in Figure A. This fixation suggests that the mechanism of injury was one of 

 

 

 

  1. Pronation-abduction

  2. Supination-adduction

  3. Pronation-external rotation

  4. Supination-external rotation

  5. Dorsiflexion-axial loading

 

PREFERRED RESPONSE 2

 

The fixation of the medial malleolus in the figure consists of a contoured reconstruction plate placed in buttress (antiglide) mode with screws running parallel to the joint surface and perpendicular to the fracture line. This fixation is indicated for vertical fractures of the medial malleolus from a supination-adduction (SA) shearing force.

 

A vertical fracture of the medial malleolus is characteristic of a Lauge-Hansen SA fracture. Bimalleolar SA fractures will also demonstrate a distal fibular avulsion. Another characteristic is impaction of anteromedial distal tibia by the talar dome. An anteromedial surgical approach allows visualization of this region, elevation of impaction, and bone grafting of the defect if necessary.

 

Dumigan et al. analyzed 4 different methods of treating vertical shear fractures with polyurethane models. They found that fixation with a 4 hole 1/3 tubular plate with 2 screws in the distal fragment was stiffer than fixation with the same plate with 1 screw in the distal fragment (3 in the proximal), fixation with two 3.5mm cortical screws with washers, and fixation with two 4mm cortical screws with washers. They recommend plating over screw-alone constructs.

 

McConnell et al. reported 8 cases of anteromedial marginal impaction in SA fractures. They note that 44 fractures were SA type. Following reduction and fixation, all 8 patients had excellent outcome and no arthritis. They recommend an anteromedial approach to the medial malleolus for visualization.

 

Figure A is a lateral radiograph showing fixation of both malleoli with reconstruction plates. Illustration A is an postoperative AP fluoroscopy image. Illustration B is an injury film demonstrating the same SA ankle fracture. Illustration C shows articular impaction at the medial shoulder of the ankle joint typical of SA injuries. Illustration D shows marginal impaction in SA fractures.

 

Incorrect Answers

Answer 1: Bimalleolar pronation-abduction fractures comprise an avulsion fracture of the medial malleolus and comminuted fibula fracture proximal to the syndesmosis (with a butterfly fragment). Answer 3: Bimalleolar pronation-external rotation fractures comprise a transverse medial malleolar fractures and a spiral fibula fracture (anterosuperior to posteroinferior) above the syndesmosis. Answer 4: Bimalleolar supination-external rotation fractures comprise a transverse medial malleolar fracture and short oblique fibula fracture (anteroinferior to posterosuperior).

Answer 5: Axial loading leads to pilon fractures. A dorsiflexed position leads to anterior plafond injury.

 

(SBQ12TR.76) A 24-year-old male sustains the fracture dislocation shown in Figure

A. How is this fracture pattern best classified? 

 

 

 

  1. Moore I

  2. Moore II

  3. Schatzker III

  4. Schatzker V

  5. OTA type 41B3

 

PREFERRED RESPONSE 2

 

Figure A shows a Moore Type II fracture. Moore type II fractures consist of fractures involving the entire tibial condyle, where the fracture line begins in the opposite compartment and extends across the tibial eminence.

 

A number of classification systems are used to describe tibial plateau fractures. The main classification systems that are widely used include Schatzker, AO/OTA, Hohl and Moore. The Moore classification describes high energy fracture-subluxation injuries of the knee. This is thought to have clinical implications as the type of fracture pattern can indicate an associated soft-tissue injury.

 

Moore retrospectively reviewed over a 1000 proximal tibia fractures to devise a classification systems based on the characteristic of five types of tibia plateau fracture patterns. He believed this system helped to better understand knee instability and concomitant soft-tissue injury. For example, Moore Type III fractures (rim avulsion) are associated with a high rate of neurovascular injury.

 

Figure A shows a fracture involving the entire tibial condyle. Illustration A shows a schematic diagram of the Moore classification (I - V). Illustration B shows a table that

describes the Moore classification (I - V).

 

Incorrect Answers:

Answer 1: Moore Type I is a coronal split fracture of the medial tibia plateau which displaces distally. It does not cross over to the contralateral side of the eminence. Answer 3: Schatzker Type III is a pure lateral plateau depression. Answer 4: Schatzker Type V is a bicondylar fracture where the metaphysis and diaphysis remain intact. Answer 5: OTA type 41B3 is a partial articular fracture with a split-depression of the unilateral plateau.

 

(SAE12FA.68) A middle-aged man sustains traumatic loss of the second, third, and fourth toes in a lawnmower accident. The wound is grossly contaminated with soil. Penicillin is added to his antibiotic regimen for coverage of what bacteria? Review Topic

 

  1. Clostridium

  2. Acinetobacter

  3. Pseudomonas

  4. Mycobacterium

  5. Staphylococcus aureus

 

PREFERRED RESPONSE 1

 

In farm or soil-contaminated wounds, including lawnmower injuries, penicillin is added to broad-spectrum cephalosporin and aminoglycoside therapy to cover against Clostridium. Psuedomonas is frequently seen after puncture wounds through the shoes. Acinetobacter is generally a hospital-acquired infection.

 

(OBQ15.18) A patient presents to the emergency department with the injury seen in Figure A. Which of the following is true about radial nerve palsies associated with isolated humeral shaft fractures after low velocity gunshot wounds? 

 

 

  1. The initial treatment involves debridement, irrigation, nerve exploration, and osteosynthesis.

  2. The radial nerve palsy is often a result of neurotmesis.

  3. Initial treatment involves splinting and observation for return of neurologic function.

  4. Electrophysiologic testing for radial nerve palsies is indicated after 2-3 weeks without improvement.

  5. The radial nerve palsy will not resolve regardless of attempted interventions.

 

PREFERRED RESPONSE 3

 

When a patient sustains an isolated humeral shaft fracture and radial nerve palsy from a GSW, the initial treatment involves splinting with observation.

 

The majority of humeral shaft fractures are treated initially with a coaptation splint and then transitioned to a functional brace. Absolute surgical indications for operative management include: open fracture, brachial plexus injury, compartment syndrome, floating elbow, or vascular injury. A radial nerve palsy is not an indication for surgical management of an isolated humeral shaft fracture. Radial nerve injury from a low-velocity GSW is similar to that of blunt force trauma and thus, immediate exploration is not necessary. Instead, nerve function should be observed. Seventy percent of these nerve injuries will resolve spontaneously. Additionally, the patient should receive a short course of antibiotics as any low velocity GSW would be treated.

 

Guo et al retrospectively reviewed the electrophysiologic data for 40 radial nerve palsies caused by GSWs and blunt trauma. After characterizing the palsies by level of injury, completeness of nerve injury, and other associated nerves injured, they found there to be no difference in any of these variables between GSW induced radial nerve palsies and blunt trauma induced palsies.

 

Vaidya et al retrospectively reviewed the outcomes of 54 patients with humeral shaft fractures resulting from low velocity GSWs comparing operative and non-operative treatments. They found that patients receiving non-operative management did well and that 70% of radial nerve palsies in the non-operative treatment group resolved on

their own. They recommended non-operative treatment for the majority of isolated humeral shaft fractures resulting from civilian gunshot wounds.

 

Figure A is a radiograph of a humeral shaft fracture after a GSW. Illustration A is a radiograph of a humeral shaft fracture with a coaptation in place. Illustration B is an example of a functional brace.

 

Incorrect answers:

Answer 1: This is the initial treatment for an open humeral shaft fracture associated with an radial nerve palsy. Answer 2: Radial nerve palsy from a low velocity GSW is usually axonotmesis. Neurotmesis the most common injury in open fractures. Answer 4:EMG testing is not indicated until 3-4 months of radial nerve palsy with no neural recovery.

Answer 5: Seventy percent of radial nerve palsies will spontaneously resolve

 

(SAE12TR.88) What mechanism of injury is most likely to cause a fracture of the anteromedial facet of the coronoid? 

 

  1. Extension and axial load

  2. Varus and posteromedial rotation

  3. Valgus and posteromedial rotation

  4. Varus and posterolateral rotation

  5. Valgus and posterolateral rotation

 

PREFERRED RESPONSE 2

 

The mechanism of injury in a fracture of the anteromedial facet of the coronoid is typically a varus and posteromedial rotation force on the forearm which is the opposite of a terrible triad injury. First, the lateral collateral ligament is injured and then the medial coronoid is compressed against and then under the medial trochlea.

(SBQ12TR.86) Figure A shows intraoperative radiographs of a 45-year-old patient with a left elbow injury. What would be the next most appropriate step in this patients care? 

 

 

 

  1. Early range of motion

  2. Hinged elbow brace for 4 weeks

  3. Repair lateral collateral ligament

  4. Remove and upsize implant

  5. Remove and downsize implant

 

PREFERRED RESPONSE 5

 

The intraoperative images are consistent with overstuffing of the ulnohumeral joint during a radial head replacement. The most appropriate next step would be removing and downsizing the implant.

 

Overstuffing the radiohumeral joint by >2.5 mm can significantly alter elbow kinematics. It has also shown to lead to pain and early joint disease. The most sensitive method to assess for overstuffing of the joint is by direct visualisation intraoperatively. This can be performed by visualising the lateral aspect of the ulnohumeral joint when the radial head is resected and comparing this to when the trial radial head is reduced in place. In comparison, radiographic asymmetry of the medial ulnohumeral joint has been shown to be less sensitive. Radiographic findings of incongruity of ulnohumeral joint only occurs when significant overlengthening of the radius occurs.

 

Frank et al. examined the effect of radial head thickness in seven cadaver specimens. They found that incongruity of the medial ulnohumeral joint would only become apparent radiographically after overlengthening of the radius by >or=6 mm.

 

Doornberg et al. examined seventeen computed tomography scans of the elbow to investigate the height of the radial head relative to the lateral edge and central ridge of the coronoid process. They found that the radial head was on average only 0.9 mm more proximal than the lateral edge of the coronoid process.

Figure A shows intraoperative radiographs of a patient that has undergone a radial head arthroplasty. There is significant widening of the medial ulnohumeral joint on an AP radiograph as well as widening of the ulnohumeral joint on the lateral radiograph,

 

Incorrect Answers:

Answer 1: Early range of motion is supported after radial head arthroplasty. However, the next most appropriate step in this scenario would be the intra-operative assessment for radiohumeral joint overstuffing. Answer 2: Hinged braces may be used in the post operative period. However, overstuffing the radiohumeral joint is of first priority. Answer 3: There is no indication in this question for lateral collateral ligament repair Answer 4: Removing the implant and upsizing the radial head would be the opposite of what is needed in this scenario.

 

(SAE11OS.126) A 30-year-old man has had severe knee pain and swelling for 1 week. He reports he previously had acromioclavicular joint pain that disappeared. He denies any fever. Aspiration of a cloudy fluid from the knee reveals a WBC count of greater than 50,000 with 90% polymorphonucleocytes. While awaiting culture results, what is the most appropriate action? 

 

  1. Cortisone injection

  2. Open surgical debridement

  3. Immediate arthroscopic lavage

  4. Intravenous vancomycin for presumptive MRSA infection

  5. Obtain sexual activity history and select appropriate antibiotic

 

PREFERRED RESPONSE 5

 

The patient has polyarticular gonococcal arthritis. Acute septic arthritis in adults can be separated into two major patient groups: young (age 15 to 40 years) healthy, sexually active patients with gonococcal pyogenic arthritis and elderly or immunocompromised patients with nongonococcal septic arthritis. In gonococcal septic arthritis, the infecting organism is Neisseria gonorrhea. It is the most common cause of acute joint infection in persons 15 to 40 years of age in the U.S. The clinical presentation is variable, but typically includes migratory polyarthralgias, fever, rash, urethral or vaginal discharge, and tenosynovitis. A patient with disseminated gonococcal infection may report few genital symptoms. More than 50% of these infections are polyarticular. Because patients with gonococcal septic arthritis are healthy, prompt antibiotic treatment results in a generally good prognosis. MRSA septic arthritis would be associated with fever, more rapid onset of symptoms, and is rarely polyarticular.

 

(SAE08UE.78) Figure 37 shows a coronal T2-weighted MRI scan. What is the name of the labeled torn structure? 

 

 

 

  1. Brachialis tendon

  2. Biceps tendon

  3. Flexor/pronator origin

  4. Medial collateral ligament (MCL)

  5. Lateral collateral ligament (LCL)

 

PREFERRED RESPONSE 5

 

The labeled structure is the LCL, and it is avulsed from the lateral humeral epicondyle. This is the most common site of injury for the LCL. The biceps and brachialis tendon insertions are not well visualized in this section. The MCL and flexor/pronator origin are intact.

 

(SBQ12TR.70) Pelvic packing can be performed to temporarily treat a hemodynamically unstable patient with a pelvic ring fracture. Which of the following is the preferred location of the skin incision to perform pelvic packing? 

  1. Right anterior superior iliac spine (ASIS) to mid-symphysis, left lateral window incision

  2. Left ASIS to mid-symphysis, right lateral window incision

  3. Subumbilical incision

  4. ASIS to ASIS bilaterally

  5. Pararectus incision

 

PREFERRED RESPONSE 3

 

The preferred skin incision location is a subumbilical incision, 6-8cm extending upwards from the pubic symphysis towards the umbilicus; this allows access to all of the appropriate areas for pelvic packing.

 

Following skin incision, the rectus fascia is then divided in the midline which allows for access to both sides of the bladder for packing deep in the pelvic brim. On each side, 3 lap pads are placed from sacroiliac joint to the retropubic space, all placed below the level of the pelvic brim.

 

Hak et al. review the options for emergent treatment in life threatening hemorrhage secondary to pelvic fractures. The authors offer several options for emergent treatment, which includes the use of pelvic binders, the placement of external fixators, pelvic packing and interventional angiography. Goals include reduction of pelvic volume and stopping rapid hemorrhage to save a patient's life. Pelvic packing, properly performed, is done through a subumbilical incision, as described above.

 

Osborn et al. retrospectively reviewed and compared emergent pelvic packing to angiography in hemorrhagic pelvic fracture clinical scenarios. The authors noted comparable results in mortality with a noted decrease in need for post-procedure transfusions in the pelvic packing group.

 

Cothren et al. reported their outcomes following an institutional algorithmic change from pelvic ex-fix/angiography to pelvic packing and ex-fix. Since their institutional change, the authors noted a significant decrease in transfusions, need for angiography and mortality.

 

Incorrect answers:

Answers 1,2: While these approaches may give access to the pelvic brim, each only give access to one side; one needs to circumferentially access and pack the pelvis. Answer 4: An ASIS to ASIS skin incision is an unnecessary dissection with too many important structures that lie very close to the skin, including the femoral vessels. Answer 5: A pararectus incision is an alternative approach to access the quadrilateral plate for acetabular fractures and does not play a role in pelvic packing.

(OBQ13.81) Which of the following deformities is most common after the amputation shown in Figure A? 

 

 

 

  1. Pes cavus

  2. Pes planus

  3. Hindfoot valgus

  4. Equinovarus

  5. Calcaneovalgus

 

PREFERRED RESPONSE 4

 

The most common deformity after a midfoot amputation as shown in Figure A is an equinuovarus deformity due to the pull of the Achilles and plantarflexors in face of loss of the common extensors and distal insertion of the tibialis anterior.

 

Ng et al. review foot and ankle amputations, and review the issues inherent with each amputation level, including prosthesis fitting and use. They also mention that careful repair of all released or transected tendons is needed to maintain a plantigrade foot.

 

Early reviews the importance of soft tissue balancing with midfoot amputations. They note that the attachment of the resected tendons into the more proximal retained bones is critical for success in restoration of foot position and ambulation capabilities.

 

Figure A shows a midfoot amputation as the result of trauma. Illustration A shows the lateral view of the amputation, with an obvious equinus deformity.

 

Incorrect Answers:

Answer 1, 2, 3, 5: Loss of the common extensor tendons of the toes would not lead to the listed deformities.

 

(SAE09TR.96) Figure 63 shows the radiographs of a 23-year-old man who sustained a twisting injury at work. Swelling, tenderness, and ecchymosis are noted about the entire midfoot. What associated injury is most likely to be problematic? 

 

 

 

  1. Peroneal tendon tear

  2. Lateral process talus fracture

  3. Talar neck fracture

  4. Lisfranc injury

  5. Deltoid ligament tear

 

PREFERRED RESPONSE 4

 

This cuboid compression fracture (“nutcracker” injury) is associated with subtle injury to the Lisfranc complex. This diagnosis must be made to ensure proper treatment.

 

(SAE12FA.81) A patient underwent an open reduction and internal fixation of a calcaneus fracture 6 months ago via an extensile lateral approach. He now reports burning pain on the lateral side of his ankle and foot. A local cortisone injection at the site of the tenderness, about 7 cm above the lateral heel, provided temporary relief of the pain. What is the recommended course of management for the persistent burning pain? 

 

  1. Subtalar fusion

  2. Neuroplasty of the superficial peroneal nerve

  3. Neuroplasty of the sural nerve and implant removal

  4. Excision and burial of the sural nerve in deep muscle or vein

  5. Electromyography/nerve conduction velocity studies to evaluate local nerve entrapment versus radiculopathy

 

PREFERRED RESPONSE 4

 

The patient has a sural nerve neuroma, which is a known complication of the extensile lateral approach. Of the available choices, excision and burial of the sural nerve in muscle or vein is the best choice because it gives better pain relief due to the better blood supply in muscle than bone. Recent authors advocate burying the nerve in vein as the best option. Neuroplasty is a possibility (but not of the superficial peroneal nerve), but the sural nerve is usually very sensitive and often pain relief with a release is incomplete. Additionally, implant removal is not indicated because of the patient's complaints; also, the implants should not be removed at 6 months. A subtalar fusion is the choice for posttraumatic arthritis from the calcaneus fracture. Electromyography/nerve conduction velocity studies are reasonable choices if there was an indication the pain could be coming from the back or there was no clear evidence of a sural nerve neuroma.

 

(SAE11OS.71) A 54-year-old woman sustains the injury seen in Figures 71a and 71b. The injury involves her nondominant extremity. What should the patient be told regarding her expected outcome? 

 

 

 

  1. She should expect to return to full function and regain full range of elbow motion.

  2. Reduction and casting has equivalent outcomes to those of surgical treatment.

  3. This type of injury is associated with a high rate of complications.

  4. Nerve dysfunction is commonly associated with this injury.

  5. Ulnohumeral instability is the major complication seen with this fracture pattern.

 

PREFERRED RESPONSE 3

 

This is a Bado type 2 (posterior) Monteggia lesion, which is associated with higher rates of complications than other types of Monteggia lesions. The injury is associated with indirect high-energy trauma and less often pathologic causes. Of the four types of Monteggia lesions, the type 2 or posterior type is associated with the worst prognosis. These injuries are best treated surgically with dorsal plating of the ulna and reduction with fixation or arthroplasty of the radial head. The major complications seen with this injury pattern are nonunion and plate failure. Almost all patients have some loss of elbow range of motion. Satisfactory results based on functional scores for this injury are not universal. Neurologic injury and ulnohumeral instability are unusual with this type of injury. Full functional recovery is not expected with nonsurgical management.

 

(SBQ12TR.101) An otherwise healthy young adult male sustains a transverse radial shaft and ulna fracture. He undergoes definitive surgical fixation with two nonlocking compression plates (LCPs) as shown in Figure A. What is the principle of this fixation technique on bone healing? 

 

 

 

  1. Absolute stability with direct healing by callus formation

  2. Relative stability with indirect healing by callus formation

  3. Absolute stability with direct healing by internal remodeling

  4. Relative stability with indirect healing by internal remodeling

  5. Absolute stability with endochondral bone formation

 

PREFERRED RESPONSE 3

 

Definitive surgical fixation for a simple transverse both bone forearm fracture would include open reduction and internal fixation with absolute stability with direct healing by internal remodelling (i.e., primary bone healing). The radius is fixed with a 7-hole

3.5mm LC-DCP plate in compression mode. The ulna was fixed with a 8-hole 3.5mm LC-DCP neutralization plate with (2) lag screws.

 

Restoration of the radial bow, length, and alignment in conjunction with primary bone healing would optimize the restoration of pronation/supination in the forearm. Compression plating and anatomic reduction of articular fractures are examples of absolute stability. Casting, bridge plating, external fixation, and intramedullary nailing provide relative stability, with secondary bone healing resulting.

 

Moed et al. reviewed 56 patients with open diaphyseal forearm fractures treated with immediate internal fixation. The complications included deep infection in 2 patients and non-union in 6. This study indicated immediate stable plate fixation was a beneficial method of treatment for open diaphyseal forearm fractures.

 

Jones et al. discussed current literature on treating adult diaphyseal forearm fractures with open reduction internal fixation versus intramedullary nailing, and concluded a randomized prospective study is needed. They stated current literature is inconclusive.

 

Figure A shows a both bone forearm fracture fixed with absolute stability. The radius is fixed with a 7-hole 3.5mm LC-DCP plate in compression mode. The ulna was fixed with a 8-hole 3.5mm LC-DCP neutralization plate with (2) lag screws.

 

Incorrect Answers:

Answers 1-2, 4-5: Absolute stability is a biological process of osteonal bone remodelling. This is predominantly intramembranous ossification. Enchondral bone formation occurs in non-rigid fracture healing (secondary bone healing).

 

(SAE08OS.66) A 25-year-old motorcyclist has a knee dislocation that is reduced by the trauma surgeon in the emergency department. Radiographs show no fracture and a

reduced knee joint. What is the most appropriate initial step for evaluation of a potential arterial injury? 

 

  1. Pulse oximeter measurement at the great toe

  2. Angiography

  3. Measurement of the ankle-brachial index (ABI)

  4. Doppler ultrasound

  5. Assessment of capillary refill in the nail beds

 

PREFERRED RESPONSE 3

 

A high index of suspicion should exist for an arterial injury after any knee dislocation. Due to collateral circulation around the knee, pulses may still be present, as well as normal capillary refill. Though angiography is the gold standard for assessment of both major and minor (intimal) injury to the arterial system, it is invasive and not always readily available. Assessment of the ABI can be done without specialized equipment and personnel. When the ABI (systolic BP distal to injury/systolic BP of uninjured upper extremity) is less than 0.9, consideration of invasive testing or surgical exploration is recommended.

 

(OBQ15.32) A 22-year-old male cyclist was struck by a car. He complains of right knee pain and swelling, as well as reduced sensation and weakness in his right foot. His leg compartments are soft and not tender. Distal pulses in the extremity are palpable. Radiographs of the knee, as shown in Figures A and B, were taken after a closed reduction maneuver was performed. What would be the next best step in management of this patient? 

 

 

 

  1. Non-operative managment, but arrange for early follow-up in clinic

  2. Intra-compartmental pressure measurements

  3. Ankle-brachial index measurements

  4. Knee spanning external fixation

  5. Open reduction internal fixation

 

PREFERRED RESPONSE 3

 

This patient presents with a Schatzker IV tibia plateau fracture with lower extremity neurologic deficits. The next best step would be to investigate for an acute vascular injury with ankle-brachial index measurements.

 

Fracture-dislocations of the knee must be suspected with all Schatzker type IV injuries as this fracture pattern is usually associated with high energy trauma. Identifying this injury should prompt a thorough assessment of the neurovascular structures across the knee. After closed reduction and emergent immobilization of the knee, ankle brachial indices (ABI) must be immediately performed. If <0.9, further vascular testing is warranted, such as MR or CT angiography.

 

Berkson et al. reviewed high energy tibia fractures. They state that Schatzker Type IV fractures typically requires more energy than corresponding lateral plateau fractures, due to denser bone on the medial side.

 

Chang et al. described an anatomic sub-classification of Schatzker IV fractures. They describe Group 1* fractures as classic medial unicondylar fractures. Group 2* fractures are complicated variants characterized by medial condyle fractures with lateral plateau extension. Usually these have articular impaction of the centroposterior lateral plateau.

 

Figures A and B show AP and lateral radiographs of the knee demonstrating a classic medial unicondylar Schatzker IV fracture. Note the anterior subluxation of the tibia in relation to the femur. Illustration A shows the multiple CT images of this fracture pattern.

 

Incorrect Answers:

Answer 1: A vascular injury should be investigated in patients presenting with high energy tibia plateau fractures or fracture patterns associated with vascular injuries (e.g. Schatzker type 4,5,6) Answer 2: This patients foot parasthesias is not due to increased leg compartment pressures. The subjective neurovascular findings is likely related to stretch or injury to the common peroneal nerve Answer 4: Knee spanning external fixation may be considered in situations of significant soft-tissue injury or documented vascular injury to protect the repair. The next step would be to investigate a vascular injury before applying an external fixator. Answer 5: This patient requires investigations for vascular injury, nerve injury and soft-tissue injury before planning operative fixation.

(OBQ15.150) A 32-year-old female sustained the injury seen in Figure A after a motor vehicle accident. On physical exam there was obvious deformity about the arm with a laceration that probed to bone over the lateral aspect of the arm. The patient was neurovascularly intact. She was treated with an intramedullary nail. Which of the following is true? 

 

 

 

  1. Intramedullary nailing is associated with an increased risk of iatrogenic comminution.

  2. There is a lower risk of iatrogenic radial nerve palsy with anterograde intramedullary nailing when compared to compression plating.

  3. There is a lower risk of nonunion with antegrade intramedullary nailing when compared to compression plating.

  4. The musculocutaneous nerve is at risk with lateral to medial distal locking screw placement

  5. There is a higher risk of shoulder impingement with antegrade nailing compared to compression plating.

 

PREFERRED RESPONSE 5

 

When compared to compression plating, anterograde intramedullary nailing results show increased risk for shoulder impingement.

 

Options for operative management of humeral shaft fractures mainly consist of intramedullary nail or plate and screw constructs. The main advantage to intramedullary nailing is when the soft tissue envelope makes a large incision undesireable or the fracture pattern dictates a relative stability construct - such as segmental or massively comminuted injuries. The disadvantages include trauma to the rotator cuff, post operative shoulder pain, indirect reduction leading to increased risk of malrotation, and increased reoperation for implant removal.

 

Li et al. performed a randomized controlled trial with 45 patients that investigated the difference in post operative malrotation and functional outcomes when comparing intramedullary nails versus open reduction and internal fixation. They concluded that

when comparing the two operative options, patients who underwent intramedullary nailing had a greater degree of malrotation, which was associated with decreased range of motion. Additionally, they found lower functional scores with patients who underwent intramedullary nailing.

 

Kurup et al. performed a systematic review comparing outcomes between compression plating and intrameduallary nailing for operative treatment of humeral shaft fractures. With a total of 260 patients, they found no difference with blood loss, fracture union, iatrogenic radial nerve palsy, iatrogenic fracture comminution, elbow impingement, return to pre-injury occupation, and functional shoulder scores. They did show a statistically significant increase in shoulder impingement and reduction of range of motion when using an intramedullary nail.

 

Figure A is a AP radiograph of a comminuted humeral shaft fracture. Illustration A is a radiograph of a humerus fixed with an intramedullary nail. Illustration B is a radiograph of a humerus fixed with a compression plate.

 

Incorrect Answers:

Answer 1,2,3: There is no difference between compression plating and intramedullary nailing in regards to risk of iatrogenic comminution, iatrogenic radial nerve palsy, and nonunion.

Answer 4: The musculocutaneous nerve is at wrist with anterior to posterior distal locking screw placement. The radial nerve is at risk with lateral to medial screw placement.

 

(OBQ14.182) A 29-year-old female has sustained the acute injury shown in Figure A. Which of the following is an indication for open reduction internal fixation in this patient? 

 

 

 

  1. Medial sided tenderness

  2. Medial sided swelling

  3. Positive cotton test

  4. Medial clear space widening with gravity stress radiographs

  5. Positive squeeze test

 

PREFERRED RESPONSE 4

 

Figure A shows a minimally displaced Weber B ankle fracture. The need for operative treatment would be dependent on fracture stability. A gravity stress test would best demonstrate fracture displacement, syndesmotic injury and medial sided ligamentous integrity.

 

In patients who present with no medial widening on standard ankle radiographs and no clinical symptoms of deltoid ligament injury, the integrity of the deltoid ligament remains unknown. The gravity stress radiograph may be used to help identify a deltoid ligament injury in association with an isolated distal fibular fracture. Stage-IV supination-external rotation fractures, which involve the deltoid ligament, are more likely to be treated operatively as they are often considered unstable ankle fractures.

 

Egol et al. reviewed 101 patients with isolated fibular fracture and an intact mortise. They found that medial tenderness, swelling, and ecchymosis were not sensitive with regard to predicting widening of the medial clear space on stress radiographs. Interestingly, they report that good functional results can be obtained in patients with widening of the medial clear space on a stress radiograph in the absence of medial signs.

 

Gill et al. compared the effectiveness of gravity stress radiograph as compared to manual stress radiograph for the detection of deltoid ligament injury in isolated fibular fracture. A total of twenty-five patients with SER type-II fracture and SER Type IV-equivalent fractures were enrolled. They found the gravity stress radiograph was equivalent to the manual stress radiograph for determining deltoid ligament injury.

 

Figure A shows a mortise radiograph displaying a minimally displaced Weber B ankle fracture. Illustration A shows the positioning for a gravity stress radiograph. The patient is in the lateral decubitus position with the injured leg dependent and off the end of the table, a mortise view is taken in 10° of internal rotation of the tibia.

 

Incorrect Answers:

Answers 1-2: Differentiation between stage-II and stage-IV supination-external rotation fractures (Lauge-Hansen Classification) is clinically relevant as an intact deltoid ligament stabilizes the ankle mortise and open reduction is thought to be unnecessary. The best way to determine deltoid integrity would be to perform a gravity stress radiograph. Asnwer 3: The cotton test is an intra-operative test to assess for syndesmotic injury. It would not be used in an awake patient for this purpose. Answer 5: A positive squeeze test is suggestive of a syndesmotic injury. However, this test has a lower sensitivity for detecting syndesmotic injury in comparison to stress radiographs.

 

(SAE11OS.1) Which of the following methods of treating a vertically oriented (eg, Pauwels III) femoral neck fracture is mechanically optimal? 

 

  1. Two parallel fully threaded screws

  2. Three parallel partially threaded screws

  3. Three parallel fully threaded screws

  4. Four parallel partially threaded screws

  5. Sliding hip screw and side plate

 

PREFERRED RESPONSE 5

 

Vertical fractures have a higher rate of displacement and nonunion because of shearing forces across the fracture. Biomechanical and clinical studies indicate that for the vertically oriented fracture of the femoral neck, the most stable fixation construct is a sliding hip screw and side plate. Antirotation screws may be used as well. Nonsurgical management carries a high risk of early displacement because of shear forces. Three screws are loaded as a cantilever and have less resistance to displacement compared with a fixed-angle device with a side plate. Fully threaded screws will not allow any compression and have the same drawbacks as partially threaded screws. The addition of a fourth screw has not been shown to be of benefit.

 

(SBQ12TR.68) Figure A is a radiograph of a 75-year-old woman that fell onto her non-dominant shoulder from a standing height. She was treated non-operatively for 9 months but continues to complain of pain when she elevates her arm. In patients with this type of fracture pattern, what factor has the greatest impact on fracture healing? 

 

 

  1. Hand dominance

  2. Angulation of fracture

  3. Smoking

  4. Early physical therapy

  5. Diet

 

PREFERRED RESPONSE 3

 

This patient has an impacted varus proximal humerus fracture. Smoking has been shown to increase the nonunion risk up to 5.5 times with these fractures.

 

Impacted varus proximal humerus fractures can be managed effectively with nonoperative care. The major factors that influence non-union are age and smoking. Solid bony union can be seen in 93-98% of patients at 1 year, with more than 97% of people returning to pre-injury level of function. The angulation of fracture, hand dominance and physical therapy does not seem to influence bone union or functional outcomes with this fracture pattern.

 

Court-Brown et al. looked at the outcomes of impacted varus fractures. They determined that the age of the patient was the major factor in overall outcome. They showed that the best results occurred in younger patients, but results deteriorate with advancing age. Physical therapy was not found to impact outcome.

 

Hanson et al. showed that impacted varus fractures can be successfully managed with non-operative care. They found that overall fracture displacement had a minor impact of fracture healing and functional outcome. The predicted risk of delayed union and nonunion was 7% with patients that smoke. This was 5.5 times greater than nonsmokers.

 

Figure A shows an AP radiograph of a varus angulated proximal humerus fracture. This radiograph shows delayed atrophic union.

 

Incorrect Answers:

Answers 1,2,4: These factors do not have a significant influence on bone healing. Answer 5: Diet has shown to improve bone healing but this does not have the greatest impact.

 

(SAE09TR.12) Which of the following is most commonly associated with an open clavicular fracture? 

 

  1. Scapulothoracic dissociation

  2. Closed head injury

  3. Calcaneus fracture

  4. Pelvic ring injury

  5. Open tibial fracture

 

PREFERRED RESPONSE 2

 

Open clavicular fractures are rare and result from high-energy trauma. In a series of 20 patients with open clavicular fractures, 13 (65%) sustained a closed head injury. Fifteen (75%) had associated pulmonary injuries and 35% had a cervical or thoracic spine fracture. Only one demonstrated scapulothoracic dissociation. Screening for pulmonary and closed head injuries should be considered in the setting of traumatic open clavicular fractures.

 

(SAE13PE.113) Figures 113a and 113b are the radiographs of a 7-year-old girl who was evaluated for a visible elbow deformity by a foster parent. She thought the child fell, but her history was vague. On physical examination, a large prominence was seen over the posterolateral elbow, and the girl lacks the terminal 20 degrees of elbow extension. She has 75 degrees of elbow pronation and supination. She was nontender on examination. What is the most appropriate next treatment step? Review Topic

 

 

  1. Child abuse workup

  2. Closed reduction

  3. Open reduction with possible osteotomy

  4. Observation

 

PREFERRED RESPONSE 4

 

The most appropriate management of this condition is observation. The patient most likely has a congenital dislocation of the radial head, although this may also represent a posttraumatic deformity. The absence of findings on physical examination speaks against an acute injury. The appearance of the radial head reveals the typical findings of a congenital dislocation, namely the convex appearance of the proximal radial articular surface. These children typically have very functional range of motion and do not require treatment unless they are symptomatic. There is nothing in this child's history to suggest abuse.

 

(SAE09TR.34) To avoid an injury to the L5 nerve root when placing an S1 sacroiliac screw, what area of the sacrum should be avoided on the lateral C arm image shown in Figure 21? 

 

 

  1. A

  2. B

  3. C

  4. D

  5. E

 

PREFERRED RESPONSE 1

 

Safe placement of a sacroiliac screw depends on excellent imaging of and understanding of pelvic anatomy. There are variations in the anatomy of the upper sacrum. Patients with dysplasia of the sacrum can have “in-out-in” screws placed that exit the ilium, pass anterior to the sacral ala, and injure the L5 nerve root. To make sure that this does not occur, a lateral image of the sacrum is used to ensure that the starting point is in the “safe zone.” The starting point needs to be below the iliac cortical density (ICD) which parallels the sacral alar slope. This will prevent placing screws into the recessed ala of patients with a dysplastic sacrum. The triangular area anterior to the ICD is labeled A in the figure, B represents the sacral canal, C is S2, D is the anterior border of the sacrum, and E represents the greater sciatic notches.

 

Illustration A (Routt et al) shows the important landmarks and safe zone of the lateral sacral view for screw placement.

 

(SBQ12TR.81) A 24-year-old patient presents after a fall from the balcony of a third story building in which he landed on his feet. He reports lumbar back pain and numbness in his perineum region. Radiographs of his hips and pelvis are seen in Figure A, while CT images are shown in Figures B and C. How is this fracture pattern best classified? 

 

 

  1. Young-Burgess APC Type II

  2. Young-Burgess LC Type I

  3. "U" Type Spinopelvic Dissociation

  4. Denis Zone-I

  5. Denis Zone-II

 

PREFERRED RESPONSE 3

 

This patient has a Denis zone-III "U" Type Spinopelvic Dissociation of the sacrum.

 

The Denis classification system for sacral fractures is based on anatomical fracture zones. Zone-III fractures involve the spinal canal and have a high rate of associated neurologic injury. It is important to recognize bilateral sacral fractures with a transverse component, as this often causes spinopelvic discontinuity and possible cauda equina. The lack of mechanical continuity between the spine and pelvis will most likely require reduction and fixation for initial stability. There may also be a need for sacral decompression with fixation mechanisms given the onset of neurologic symptoms.

 

Mehta et al. reviewed sacral fractures. They report that patients with a transverse sacral fracture involving more proximal sacral segments (S1, S2, or S3) tend to have a higher prevalence of bladder dysfunction than do those involving distal sacral segments (S4 or S5).

 

Schildhauer et al. report the best way to visualize Denis zone-III "U" type fractures is to obtain a lateral view of the sacrum or sagittal reformatted images with a CT scan. Standard pelvic radiographs often miss this injury.

 

Figure A shows an AP pelvis with suggestion of sacral irregularities. No fracture pattern can be indentified. Figures B and C show CT scan images showing a "U" Type fracture pattern with angulation and translational displacement of the cephalad and caudad parts of the sacrum. Illustration A shows examples of complex sacral Denis zone-III fractures.

Incorrect Answers:

Answer 1: APC Type II fractures do not classify fracture extensions into the sacral canal. The posterior SI ligaments are considered intact. There is usually disruption of sacrospinous and sacrotuberous ligaments. Answer 2: LC Type I is an oblique or transverse ramus fracture and ipsilateral anterior sacral ala compression fracture. Answer 4,5: Figures A-C demonstrate a "U" type fracture of the sacrum.

 

(SAE12TR.57) Figure 57 is the radiograph of a 58-year-old woman who is right-hand dominant and has fallen on her flexed right elbow and is seen in the emergency department reporting isolated episodes of right elbow pain. Examination reveals that the skin is contused but intact, and her distal neurovascular examination is normal. What is the most appropriate treatment? Review Topic

 

 

 

  1. Percutaneous pinning

  2. Closed reduction and extension casting

  3. Fragment excision and triceps advancement

  4. Open reduction and internal fixation with plate fixation

  5. Open reduction and internal fixation with tension band wire construct

 

PREFERRED RESPONSE 4

 

The patient has sustained an isolated, closed fracture of the olecranon without associated instability. The bone is radiographically osteopenic and the fracture is displaced, comminuted, and includes articular marginal impaction. Plate fixation is preferred in the presence of comminution or associated transolecranon or radiocapitellar instability. Displaced fractures are generally treated surgically in an effort to restore articular congruity, restore extensor function, and to allow for early mobilization in an effort to maximize functional outcomes. A tension band wire

construct is a commonly used technique but is reserved for simple fracture patterns without comminution. Excision and triceps advancement can be considered in elderly, low-demand patients that have small unreconstructable fracture patterns without associated elbow instability.

 

(OBQ13.102) Which of the following injuries is most likely associated with the fracture seen in Figure A? 

 

 

 

  1. Medial meniscal tear

  2. Lateral meniscal tear

  3. Lateral collateral ligament rupture

  4. Medial collateral ligament rupture

  5. Posterior cruciate ligament rupture

 

PREFERRED RESPONSE 2

 

Lateral meniscal tears are most commonly associated with Schatzker II tibial plateau fractures (split/depressed).

 

Soft tissue pathology is common in tibial plateau fractures. In general, fractures that are largely displaced and/or a result of high energy trauma are more likely to have associated soft tissue pathology. A majority of meniscal injuries that occur in the setting of tibial plateau fractures are meniscocapsular detachments. This has important implications for healing (more reliable healing in the vascular zone). Additionally, the meniscus usually remains in close contact with the femoral condyle, while the tibial plateau widens around it. It is generally agreed upon that meniscal tears should be repaired, if possible, at the time of internal fixation to decrease the likelihood of

postraumatic arthritis.

 

Gardner et al. review 62 patients with Schatzker II tibial plateau fractures that had an MRI preoperatively. For displaced fractures, the incidence of lateral meniscal tears was 83%, while the incidence of lateral collateral and posterior cruciate ligament injuries was 30%.

 

Ringus et al. attempted to determine if the degree of lateral tibial plateau fracture depression on computed tomography (CT) images predicted the presence of lateral meniscus tears. Fractures with > 9mm depression had an eight-fold increase in lateral meniscal tears, and those younger than 48 years-old had a four-fold increase in lateral meniscal tears.

 

Illustration A shows an MRI of a Schatzker II tibial plateau fracture with a lateral meniscal detachment and a medial meniscal tear. Illustration B shows the Schatzker Classification, I-VI.

 

Incorrect Answers:

Answers 1, 3, 4, 5: While these soft tissue injuries may occur, lateral meniscal tears are more common for this particular fracture pattern.

 

(OBQ13.235) A 70-year-old female presents with right thigh ache for 6 months. Except for a history of osteoporosis, she is otherwise healthy. She has been on antiresorptive therapy for 8 years. Her radiograph is shown in Figure A. Four months later, she trips over a rug and falls, sustaining the injury shown in Figures B and C. Which of these statements is TRUE regarding surgical fixation of this fracture compared with conventional fractures? 

 

 

  1. There is an increased risk of revision surgery with plate fixation.

  2. There is an increased risk of iatrogenic fracture with plate fixation.

  3. There is an increased risk of non-union with plate fixation.

  4. There is an increased risk of iatrogenic fracture with intramedullary nail fixation.

  5. There is an increased risk of hardware failure with nail fixation.

 

PREFERRED RESPONSE 4

 

This patient has a bisphosphonate-related (BP) fracture. There is increased risk of iatrogenic fracture with IM nailing of this fracture.

 

Subtrochanteric fractures are fractures extending from the lesser trochanter to 5cm distal to it. BP fractures are characterized by (1) focal lateral cortical thickening, (2) transverse fracture orientation, (3) medial spike and (4) lack of comminution. There is increased risk of iatrogenic fracture with IM nailing because BP fractures have thickened, brittle cortices and the mismatch between medullary diameter and increased proximal nail diameter results in iatrogenic fracture.

 

Weil et al. reviewed the outcome of surgically treated bisphosphonate fractures. IM nailing resulted in healing in 54% of bisphosphonate fractures with 98-99% of conventional fractures. In their study, 46% required revision procedures.

 

Prasarn et al. compared plate and nail treated bisphosphonate fractures with conventional fractures. They found that the bisphosphonate group had more major and minor complications (68%) than the conventional group (10%). The most common complications were intraoperative femoral shaft comminution (nail) and hardware failure (plate).

 

Figure A shows diffuse cortical thickening with an antero-lateral cortical ridge. Figure B shows a transverse subtrochanteric fracture extending through the middle of the cortical ridge seen previously. Note also healed fractures of the left superior and inferior rami. Figure C is a post-reduction radiograph showing the transverse fracture through the beak-shaped region of the previous insufficiency fracture. Illustration A shows a typical bisphosphonate-related fracture with transverse fracture orientation,

focal lateral cortical thickening (white arrows), medial beak (black arrow), and lack of comminution. Illustration B shows a conventional subtrochanteric fracture. Illustration C shows intraoperative iatrogenic fracture with anterolateral comminution during nailing. Illustration D shows fixation with a proximal femoral hook LCP Plate with late hardware failure at 3 months.

 

Incorrect Answers:

Answers 1, 3: Increased non-union after IM nailing has been observed, necessitating more revision surgery compared with conventional fractures. Revision surgery includes nail dynamization, exchange nailing, and conversion to plating. These procedures are more frequently performed for osteoporotic fractures. Answer 2: Compared with conventional fractures, there is no increased risk of iatrogenic fracture with plating. Answer 5: There is an increased risk of plate hardware failure because of because of a varus moment arm and dependence on intramembranous healing inhibited by bisphosphonates.

 

(SAE11OS.48) Figure 48 shows the radiograph of a 17-year-old boy who sustained a gunshot wound to his forearm. There is a small entrance wound on the volar surface. The exit wound is dorsal and more than 15 cm in size, with loss of skin and an extensive amount of devitalized muscle hanging out of the wound. Vascular supply to the hand is excellent, the ulnar and median nerves are intact in the hand, but the radial sensory nerve function is absent. After repeated surgical debridements of the wound and bone, definitive treatment for the fracture would most likely be which of the following? Review Topic

 

 

 

  1. Spanning external fixation of the radius

  2. Open reduction and internal fixation of the radius with free fibular flap interposition

  3. Open reduction and internal fixation of the radius with interposed strut allograft

  4. Open reduction and internal fixation of the radius with massive cancellous allografting

  5. Open reduction and internal fixation of the radius with massive cancellous autografting

 

PREFERRED RESPONSE 2

 

The injury needs a very complex traumatic reconstruction. After repeat debridements, there will be a very long segmental loss of the radius, with a significant loss of skin and muscle covering the bone. Spanning external fixation represents a good temporary fixation tool but will not be a definitive solution. The preferred procedure is a vascularized fibular graft with associated skin flap from the lateral leg. This surgical option brings healthy vascularized bone and soft-tissue coverage into an area with significant bone and soft-tissue loss. Placement of large quantities of allograft material, especially strut allograft, is generally contraindicated in the setting of open fractures with soft-tissue compromise because of the risk of infection. Internal fixation and massive cancellous autografting is usually limited to one defect of less than 5 cm with intact soft-tissue covering.

 

(SAE11UE.53) A 41-year-old woman with diabetes mellitus fell onto her outstretched arm and sustained an injury to the right elbow. Radiographs are shown in Figures 53a and 53b. What is the most appropriate management? 

 

 

 

  1. Open reduction and internal fixation

  2. MRI of the elbow to assess the integrity of the collateral ligaments

  3. Immobilization in a long arm cast for 3 weeks

  4. Short-term immobilization in a splint, followed by early motion exercises

  5. Radial head replacement

PREFERRED RESPONSE 1

 

The radiographs reveal a capitellum fracture with anterior displacement. To regain concentric and stable joint motion, this fragment requires reduction and stabilization. Without a joint dislocation, the ligaments are unlikely to be damaged and do not require further assessment with MRI. Closed reduction may be considered, but is unlikely to be successful. Without anatomic reduction of the fracture fragment, immobilization in either a long arm cast or a splint will not provide optimal outcomes. Based on the radiographs, the radial head is intact and does not require replacement.

 

(OBQ15.251) A 76-year old patient underwent partial foot amputation through the talonavicular and calcaneocuboid joints. Besides Achilles tendon lengthening, what additional procedure(s) may be required to prevent the most common post-operative deformity? 

 

  1. Posterior capsule release

  2. Anterior tibialis transfer to the talar neck

  3. Anterior tibialis transfer to cuboid

  4. Flexor hallucis longus transfer to calcaneus

  5. Peroneus brevis transfer to calcaneus

 

PREFERRED RESPONSE 2

 

Achilles tendon lengthening AND anterior tibialis transfer to the talar neck would be required to prevent equinovarus deformity.

 

Partial foot amputation through the talonavicular and calcaneocuboid joints is also known as the Chopart amputation. Chopart amputation alone is known to result in significant equinovarus deformity. This deformity results in excessive pressure on the anterior wound during gait, causing pain and wound complications. Transfer of the tibialis anterior tendon to talar neck will provide force, and muscle tone, that promotes ankle dorsiflexion. Lengthening of the Achilles tendon will also reduce the equinus moment force across the ankle joint.

 

Dillion et al. examined the gait patterns of partial foot amputees. They found that amputations proximal to the metatarsal heads compromised the normal propulsive function of the foot and ankle. The ideal level of amputation to maintain normal propulsive function was distal to the metatarsal heads (i.e., disarticulating the

metatarsophalangeal (MTP) joint). Illustration A is a lateral radiograph showing a Chopart amputation.

Incorrect Answers:

Answer 1: Achilles tendon lengthening and posterior capsule are insufficient to prevent the equinovarus deformity. Answer 3: The cuboid is removed with this amputation technique. Answer 4: Flexor hallucis longus transfer to the calcaneus would exacerbate the equinovarus deformity.

Answer 5: Achilles tendon shortening would exacerbate the equinovarus deformity.

 

(SAE09TR.100) A 25-year-old male polytrauma patient undergoes initial temporary external fixation for a femoral shaft fracture. He is converted to a femoral nail at 7 days. This management can be expected to result in 

 

  1. higher infection rates.

  2. higher nonunion rates.

  3. equal union and infection rates.

  4. higher rate of ARDS.

  5. higher mortality rate.

 

PREFERRED RESPONSE 3

 

Recently Harwood and associates investigated the principles of damage control orthopaedics (DCO) as they apply to patients with femoral shaft fractures. When they compared those who underwent initial external fixation of femoral shaft fractures with conversion to an intramedullary nail to those who underwent intramedullary nailing as their initial treatment, they found the following: overall infection rates were comparable in patients receiving DCO versus primary intramedullary fixation; open fracture was an independent risk factor for infection regardless of the treatment method; contamination rates in external fixator pin sites rose considerably when left in place more than 2 weeks and logistic regression analysis suggests that infection rates may increase when conversion to an intramedullary nail occurs after 2 weeks following external fixation; and there was no significant difference in time to union among treatment groups.

(SAE12FA.29) A patient falls off a roof and sustains the fracture shown in Figure 29. What is the most likely complication that results from injury to the structure that is located at the arrow? 

 

 

 

  1. Paresthesias on the plantar aspect of the foot

  2. Pain or popping with great toe flexion

  3. Loss of the arch

  4. Inability to flex the lesser toes

  5. Spring ligament rupture

 

PREFERRED RESPONSE 2

 

The arrow points to the sustentaculum tali, which is fractured off the tuberosity of the calcaneus. The flexor hallucis longus (FHL) tendon runs directly under this structure. An injury to this structure could cause stenosis around the FHL tendon which would cause pain with great toe flexion. Paresthesias on the plantar aspect of the foot refers to the medial plantar nerve. Loss of the arch refers to the plantar fascia which attaches at the calcaneal tuberosity. Inability to flex the lesser toes refers to the flexor digitorum longus tendon which runs superior to the sustentaculum. The spring ligament runs from the navicular to the calcaneus, but does not attach under the sustentaculum tali.

 

(OBQ14.40) A polytrauma patient underwent the following procedures: (1) statically locked intramedullary nailing for a right femoral shaft fracture; (2) open reduction with plate-and-screw fixation [ORIF] for a right simple distal fibula fracture; (3) ORIF right middle third radius and ulna fracture; and (4) ORIF left humeral shaft fracture. What is the appropriate weightbearing status for this patient? Review Topic

  1. Weight bearing as tolerated in all extremities

  2. Early protected weight bearing right lower extremity in walking cast, weight bearing as tolerated left upper extremity, non-weight bearing right forearm

  3. Weight bearing as tolerated in bilateral lower extremities and right upper extremity, non-weight bearing left upper extremity

  4. Non-weight bearing bilateral upper extremities and right lower extremity

  5. Non-weight bearing right upper and lower extremities, weight bearing as tolerated left upper and lower extremities without walking cast

 

PREFERRED RESPONSE 2

 

The standard postoperative weightbearing for locked medullary nailing for femoral shaft fractures and humeral shaft fractures is weight bearing as tolerated (WBAT). Simple ORIF ankle fractures may be managed with early protected weight bearing. ORIF right middle third radius and ulna fracture should be managed with a period of non-weight bearing due to risk of secondary displacement of the fracture.

 

Tingstad et al. examined the effect of immediate weightbearing on plated fractures of the humeral shaft. They reported that immediate weightbearing on humeral shaft fractures, treated with plating and full weightbearing, did not have any negative effect on the union or malunion rates.

 

Brumback et al. evaluated the feasibility, safety and efficacy of immediate weightbearing after treatment of femoral shaft fractures with statically locked IM nail. Using biomechanical and clinical data, they showed that all fractures united with no loss of fixation or hardware failure.

 

Starkweather et al. retrospectively assessed the complications and loss of reduction in patients who bore weight in a short leg cast within 15 days after surgical repair of acute unilateral closed ankle fractures. Of the 81 ankle fracture radiographs, 80 (98.8%) showed no displacement in fracture reduction on the final follow-up examination. These results suggest early protected weightbearing may be safe.

 

Incorrect Answers:

Answers 1, 3-5: Protected weight bearing has shown to shorten hospital stay and return of function with lower extremity fractures, without a significant increase in hardware failure. Both bone forearm fractures should not be treated with early weight bearing.

 

(SAE08UE.37) Figures 18a through 18c show the clinical photograph, radiograph, and CT scan of a 21-year-old man who reports persistent pain after injuring his right

shoulder 4 months ago. What is the most likely factor associated with this patient’s diagnosis? 

 

 

 

  1. Shortening of 3 cm

  2. Severity of trauma

  3. Duration of immobilization

  4. Type of immobilization

  5. Closed reduction

 

PREFERRED RESPONSE 2

 

The more severe the trauma, the higher the rate of subsequent clavicular nonunion. Neither duration nor type of immobilization has been clearly demonstrated to be a causative factor in the development of nonunion. Similarly, closed reduction has not been found to alter the healing course in midshaft clavicular fractures.

 

(SAE10BS.67) Glenohumeral disarticulation often leads to which of the following changes? 

 

  1. Hiking of the shoulder girdle

  2. Hypertrophy of the amputated shoulder girdle

  3. Improvement in thoracic spinal deformity

  4. Protraction of the shoulder

  5. Winging of the scapula

 

PREFERRED RESPONSE 1

Postural abnormalities are common after high upper extremity amputation. Normally the weight of the upper extremity and the shoulder girdle muscles keep the shoulder balanced. When the arm is amputated and the scapula remains, the shoulder girdle muscles are unopposed, resulting in upward movement often called "hiking" of the shoulder girdle. In a growing child, removal of the entire upper limb can result in scoliosis of the spine due to muscle imbalance. Abnormal shoulder elevation can often be minimized by corrective exercises and wearing a shoulder prosthesis.

 

(SAE11UE.63) A 35-year-old man is involved in a motorcycle collision and reports left elbow pain. Examination of the left arm reveals diffuse swelling and ecchymosis. His distal neurovascular status is intact. A radiograph of the injury is shown in Figure

63. The most appropriate surgical approach includes which of the following? Review Topic

 

 

 

  1. Anterior approach of Henry

  2. Lateral Kocher approach

  3. Medial column approach

  4. Posterior extensile elbow approach with olecranon chevron osteotomy

  5. Closed reduction and percutaneous screw fixation

 

PREFERRED RESPONSE 4

 

Intra-articular distal humerus fractures are best approached through a posterior elbow approach, including an olecranon chevron osteotomy to clearly visualize the reduction of the articular surface. The other stated approaches will not provide sufficient visualization of the joint surface to allow stable reduction. A closed reduction and screw fixation will not offer optimal fracture stability.

 

(SAE12TR.62) When planning pin placement for external fixation of the tibia, what is the maximum extent of the knee capsular reflection from the subchondral joint line? 

 

  1. 4 mm

  2. 6 mm

  3. 10 mm

  4. 14 mm

  5. 20 mm

 

PREFERRED RESPONSE 4

 

Intracapsular pin placement is a concern for septic arthritis. Reid and associates and DeCoster and associates have demonstrated that the maximum distal extent of the knee capsule is 14 mm from the subchondral line and occurs in the posterolateral region. The recommended placement of external fixation pins is greater than 14 mm from the subchondral line of the proximal tibia.

 

(SAE08OS.98) In a polytraumatized patient with a high lactate level, large base deficit, and pulmonary injury, what is the best initial treatment for a femoral shaft fracture? 

 

  1. Unreamed femoral nail

  2. Reamed femoral nail

  3. Plating of the femur

  4. Skeletal traction

  5. External fixation

 

PREFERRED RESPONSE 5

 

To prevent the "secondary hit" phenomenon in the polytraumatized patient, the use of Damage Control Orthopaedics has been accepted as the best initial treatment option until the patient has been successfully resuscitated. The use of reamed or unreamed

intramedullary nails in the acute setting is to be avoided because of the possibility of increasing the patient's morbidity. Plating is time consuming and leads to blood loss which should be avoided in the severely injured patient. Skeletal traction and supine positioning are detrimental in the trauma patient. Expedient external fixation, resuscitation, and later definitive treatment with an intramedullary nail is considered to be the best choice for the polytraumatized patient with a femoral shaft fracture.

 

(SAE08OS.123) A patient sustains an injury to the pelvis as a result of high-energy trauma. A radiographic and clinical assessment is performed. A lateral view of the pelvis will best identify 

 

  1. pubic symphysis diastasis.

  2. sagittal plane pelvic rotation.

  3. sacroiliac diastasis.

  4. spinopelvic dissociation.

  5. a vertical transforaminal sacral fracture.

 

PREFERRED RESPONSE 4

 

Radiographic assessment of the pelvis is an essential part of the initial evaluation of trauma patients. In addition to an AP radiograph, 40-degree caudal (inlet) and 40-degree cephalad (outlet) views are obtained. The caudal/inlet view is useful for detecting the integrity of the pelvic ring because it shows the sacroiliac joints and the sacrum, which can help detect either sacroiliac dislocations or sacral fractures and their displacement in the anterior-posterior plane. The outlet view detects superior or inferior displacement and sagittal plane flexion or rotation of the pelvis. A lateral radiograph of the sacrum identifies transverse fractures of the sacrum and coccyx and the degree of their displacement. A lateral view should be obtained in instances of bilateral sacral fractures and U-shaped sacral fractures in which there are bilateral vertical fractures through the sacral foramina that connect with a transverse fracture line between the second or third sacral segments. Some patients with these fractures have sacral fracture-dislocations with spinopelvic dissociation and a high incidence of neurologic injuries. The lateral view discloses the extent of displacement of the upper sacral segment (usually S2 forward on S3) and the resultant kyphotic deformity.

(OBQ14.167) Figures A-C are images of a 37-year-old man who presents with isolated muscle atrophy due to a compressed nerve. Which of the following sequences correctly describes the pathway of this nerve through the brachial plexus, before it innervates the affected muscles? 

 

 

 

  1. C5-C7 nerve roots; upper/middle trunks; anterior division; lateral cord

  2. C5-C6 nerve roots; upper trunk; posterior division; posterior cord

  3. C5-C6 nerve roots; upper trunk

  4. C5-C7 nerve roots

  5. C8-T1 nerve root; lower trunk; anterior division; medial cord

 

PREFERRED RESPONSE 2

 

Figures A-C shows atrophy of teres minor and deltoid due to compression of the axillary nerve. The correct pathway of the axillary nerve within the brachial plexus is, C5-C6 nerve roots; upper trunk, posterior division, posterior cord.

 

Quadrilateral space syndrome is a condition defined by axillary nerve, +/- posterior humeral circumflex artery compression in the quadrilateral space. It most commonly affects the dominant shoulder in overhead movement athletes (e.g. basketball players) or other throwing athletes. Physical examination may reveal weakness with the arm positioned in abduction and external rotation. In situations of long-standing compression, there may also be atrophy of the teres minor and deltoid muscle.

 

Chafik et al. dissected thirty-one cadaveric human shoulders to describe the neuromuscular anatomy of teres minor. They showed that the primary nerve branch to teres minor travelled in a fascial sling 44 mm medial to the muscular insertion. This

area may be the potential site of greatest compression and tethering of this nerve in patients with isolated teres minor atrophy.

 

Friend et al. performed a cadaveric dissection of nine shoulder specimens to look at the anatomical variability in course, length and branching pattern of both the teres minor nerve and the axillary nerve. These were compared to a case-based study of these two male patients with isolated atrophy of teres minor. They concluded that there is no good anatomical predictor of nerve compression outside the quadrilateral space as there is considerable anatomical variation in its origin and course, as well as potential site of compression.

 

Figure A-C are MRI images that show atrophy of the teres minor muscle and possibly deltoid muscle. The rotator cuff muscles are labeled in Illustration A. The teres minor muscle is labeled in Illustration B. Illustration C shows a diagram of the brachial plexus.

 

Incorrect

 

 

 

 

 

 

Answers:

Answer

1:

This

describes

the

 

musculocutaneous

nerve.

Answer

3:

This

describes

 

the

suprascapular

nerve.

Answer

4:

This

describes

the

 

long thoracic

nerve.

Answer 5: This describes the ulnar nerve.

 

(SAE07PE.26) The parents of a 15-month-old child report that he is not yet walking. Further evaluation, rather than reassurance and observation, should be conducted if the child is not performing what other activity? 

 

  1. Talking

  2. Sitting

  3. Buiilding a tower of blocks

  4. Drinking from a cup

  5. Scribbling on paper

 

PREFERRED RESPONSE 2

 

A child not ambulating at age 15 months is still within normal limits. The child should be able to sit by age 9 months. The remaining milestones listed are reached later in development.

(SAE09FA.10) Figures 4a through 4c show the radiographs of a 43-year-old woman who sustained a twisting injury to her right ankle. She has ankle pain and tenderness medially and laterally. To help determine the optimal treatment, an external rotation stress radiograph of the ankle is obtained. This test is designed to evaluate the integrity of what structure? 

 

 

 

  1. Posterior talofibular ligament

  2. Distal tibiofibular syndesmosis

  3. Anterior talofibular ligament

  4. Deltoid ligament

  5. Calcaneofibular ligament

 

PREFERRED RESPONSE 4

 

In the presence of a supination external rotation-type fracture of the distal fibula (Weber type B), stability of the ankle is best assessed by performing an external rotation stress AP view of the ankle. This test is used to assess the integrity of the deltoid ligament. The presence of a deltoid ligament rupture results in instability and generally is best managed surgically. The gravity stress test can also be used.

 

(SAE09TR.63) A 30-year-old man is brought to the emergency department after a motor vehicle accident. He has a closed midshaft femoral fracture and an intra-abdominal injury. He is currently in the operating room and the exploration of his abdomen has been completed. His initial blood pressure was 70/30 mm Hg and is now 90/50 mm Hg after 4 liters of fluid and 2 units of blood. His initial serum lactate was

3.0 mmol/L (normal < 2.5), 1 hour postinjury it was 3.5 mmol/L, and it is now 5

mmol/L. His core temperature is 93 degrees F (34 degrees C). What is the most appropriate management for the femoral shaft fracture at this point? 

 

  1. Reamed intramedullary nailing

  2. Traction

  3. External fixation

  4. Open plating

  5. Mast suit

 

PREFERRED RESPONSE 3

 

The patient has several indications that he is not ready for definitive fixation of the femoral shaft fracture at this point. He is cold with a core temperature of 93 degrees F, and hypothermia of less than 95 degrees F (35 degrees C) has been shown to be associated with an increased mortality rate in trauma patients. The patient has also not been resuscitated based on his increasing lactate levels and although controversial, it has been shown that temporary external fixation leads to a lower incidence of multiple organ failure and acute respiratory distress syndrome.

 

(SAE11UE.17) Figure 17 shows the radiograph of an 82-year-old right-hand dominant woman who fell while weeding her garden. She has severe right shoulder pain. She is neurovascularly intact. What is the most appropriate treatment? Review Topic

 

 

 

  1. Rest, ice, nonsteroidal anti-inflammatory drugs, activity as tolerated, and follow-up in 4 weeks

  2. Coaptation splinting and follow-up in 4 weeks

  3. Surgical replacement with hemiarthroplasty or reverse total shoulder arthroplasty

  4. Physical therapy for range-of-motion exercises

  5. Closed reduction, splinting, and follow-up in 4 weeks

PREFERRED RESPONSE 3

 

The patient has a displaced four-part proximal humerus fracture. The humeral head is displaced and if allowed to heal in this position, the patient will likely have a stiff and painful shoulder. The humerus is at risk for osteonecrosis given the displacement of the fracture. Given a patient age of 82 years, replacement options of either hemiarthroplasty or reverse total shoulder arthoplasty, allow maximal restoration of function. Physical therapy is not indicated in this acute fracture. Closed reduction techniques will not be successful in this displaced fracture.

 

(SAE11PA.39) A 63-year-old man has had increasing left leg pain over the last several months. History reveals that he has had recurring cyclic pain in the leg for the past several years. Radiographs show an enlarged, sclerotic tibia, with thickened coarse trabeculae and varus bowing. What is the most appropriate management for this patient? Review Topic

 

  1. Vitamin D

  2. Calcium supplement

  3. Methotrexate

  4. Nonsteroidal antiinflammatory drugs (NSAIDs)

  5. Bisphosphonate therapy

 

PREFERRED RESPONSE 5

 

Based on the signs and symptoms, Paget's disease is the most likely diagnosis. In Paget's disease, an elevated alkaline phosphatase level and high output heart failure may be seen. Hearing loss can be seen when there is involvement of the skull, and malignant degeneration is uncommon but recognized as a risk. Patients are often treated with bisphosphonate medications during the active disease process to help control osteoclastic activity and pain. Vitamin D and calcium are more appropriate for treatment of osteoporosis. Methotrexate is not indicated for the treatment of Paget's disease. NSAIDs may be helpful to treat pain associated with Paget's disease but will not alter the clinical course.

(SAE09TR.35) An otherwise healthy 37-year-old man fell off the flatbed of a delivery truck and landed directly on his dominant left hand. Surgical stabilization of a distal radius fracture is performed. An intraoperative radiograph is shown in Figure 22. What is the next most appropriate step in management? 

 

 

 

  1. Immobilization of the wrist in ulnar deviation for 4 weeks before starting range-of-motion exercises

  2. In situ Kirschner wire fixation of the carpal bones for 6 weeks

  3. Extending the volar incision used for fracture fixation and repairing the injured structures in addition to percutaneous Kirschner wire fixation

  4. Performing a separate dorsal incision and repairing the injured structures in addition to percutaneous Kirschner wire fixation

  5. Arthroscopic repair of the injured structures and percutaneous Kirschner wire fixation

 

PREFERRED RESPONSE 4

 

The intraoperative radiograph reveals a scapholunate ligament disruption. Repair of the stout dorsal scapholunate interosseous ligaments is required. Interestingly, the results of scapholunate ligament injuries associated with distal radius fractures appear to be superior to those of isolated ligament injuries.

 

(OBQ13.87) A 40-year-old male patient sustains a bimalleolar ankle fracture and undergoes open reduction and internal fixation. Four months later, he returns for follow-up with mild ankle discomfort, and a radiograph is shown in Figure A. What is the most appropriate next step in treatment? 

 

 

  1. Syndesmosis sagittal plane reduction and fixation

  2. Syndesmosis coronal plane reduction and fixation

  3. Osteotomy and revision of the fibula and syndesmosis

  4. Retrieval of osteochondral fragment

  5. Revision plating of the fibula and syndesmosis reduction and fixation.

 

PREFERRED RESPONSE 3

 

This patient has undergone ORIF of the lateral malleolus with shortening of the lateral malleolus and lateral tibiotalar tilt. Revision surgery would entail bone grafting and re-plating of the fibula.

 

Malunion of the fibula component of ankle fractures lead to tibiotalar instability and post-traumatic ankle arthritis. The distal fragment is usually shortened and externally rotated. The osteotomy can restore length and correct rotation. Markers for potential instability include: (1) asymmetry of the medial-lateral clear spaces, (2) talar tilt

>2mm, (3) talar subluxation, (4) abnormal talocrural angle (normal, 75-86deg).

 

Chu et al. opined that reconstruction for distal fibula malunions should include: (1) osteotomy, (2) +/- syndesmotic fixation and (3) autologous bone graft. They recommend: (1) low oblique osteotomy for fractures below the syndesmosis, (2) transverse osteotomy above the syndesmosis for high fractures (PER4) and low fractures with tibiofibular instability, (3) inspection of the tibiofibular joint through an anterolateral window to ensure anatomic reduction.

 

Weber et al. described a method of corrective lengthening osteotomy of the fibula in 23 cases. They described 3 criteria for assessing normal fibular length. Seventeen patients had good-excellent results, and 6 had fair-poor results (1 of these 6 needed ankle fusion).

 

Figure A is an AP radiograph of a distal fibula fracture fixed in a shortened position with lateral talar tilt and degenerative changes at the anterolateral tibiotalar joint. Illustration A is an anteroposterior radiograph after fibular osteotomy and correction

with medial distal tibial autograft to correct talar tilt and restore anatomic fibular length. Illustration B shows the normal talocrural angle. Illustration C shows the Weber-Simpson method of fibula lengthening used in Illustration A.

 

Incorrect Answers:

Answer 1, 2, 5: There is no syndesmotic disruption. Answer 4: There is no evidence of a loose osteochondral fragment.

 

(SAE09TR.1) A 26-year-old woman sustained a nondisplaced femoral neck fracture and treatment consisted of use of percutaneous cannulated screws. At her 3-month follow-up visit, she reports hip pain and is unable to ambulate. A radiograph is shown in Figure 1. What is the next most appropriate treatment? 

 

 

 

  1. Bone grafting and revision open reduction and internal fixation

  2. Hemiarthroplasty

  3. Dynamic hip screw without angular correction

  4. Valgus intertrochanteric osteotomy

  5. Core decompression

 

PREFERRED RESPONSE 4

 

Femoral neck fracture nonunion is a challenging problem for orthopaedic surgeons. Vertical fractures are more prone to nonunion due to shear stress rather than compressive forces across the fracture site. Several authors have suggested these fractures are more common in young adults due to injury type and bone composition. It is widely regarded that an effort should be made to salvage the femoral head if vascularity remains. The most common method to treat this complication is valgus intertrochanteric osteotomy of the femur. This functionally makes a vertical fracture

more horizontal, converting shear into compressive forces. It also helps correct the varus position of the fracture nonunion.

 

(SAE12FA.4) Figures 4a and 4b are the radiographs of an isolated injury. What is the next most appropriate step in management? 

 

 

 

  1. CT

  2. MRI

  3. Closed reduction and casting

  4. Application of a spanning external fixator

  5. Immediate open reduction and internal fixation (ORIF)

 

PREFERRED RESPONSE 4

 

Successful treatment of a pilon fractures requires a complete understanding of the fracture configuration. This information is not available using radiographs alone; therefore, CT is used to define the fracture anatomy but only after stabilization and distraction of the fracture via external fixation. MRI does not adequately show the detail of the bone fragments. Immediate ORIF is contraindicated because of the high rate of soft-tissue complications with this treatment regimen, whereas closed treatment has a high rate of poor outcomes because of arthritis. Delayed ORIF is the recommended treatment, but this occurs after temporary stabilization and CT scanning.

(SAE09FA.22) A 68-year-old man fell off a 20-foot mountain cliff and was seen in the emergency department the following morning. A radiograph is shown in Figure

12. He is a nonsmoker with medical comorbidities of hypertension and hypercholesterolemia that is well controlled with medicine and diet. Capillary refill and sensation are intact distally and the patient is able to move his toes with mild discomfort. Serosanguinous fracture blisters are present laterally, and the foot is swollen and red. What is the most appropriate management? 

 

 

 

  1. Short leg cast for 6 weeks

  2. Splinting with early range of motion at 3 weeks

  3. Immediate open reduction and internal fixation through a medial approach

  4. Delayed open reduction and internal fixation

  5. Fusion

 

PREFERRED RESPONSE 4

 

Whereas a patient age of older than 50 years used to be a contraindication for open reduction and internal fixation of displaced intra-articular calcaneal fractures, new data suggest that the presence of associated medical comorbidities that affect wound healing such as smoking, diabetes mellitus, and peripheral vascular disease are more relevant to postoperative functional outcome. Surgical treatment of Sanders II and III displaced intra-articular calcaneal fractures with initial Bohler angles of > 15 degrees results in better outcomes as compared to nonsurgical management. Indications for primary fusion might include Sanders IV fractures in which articular congruity or Bohler angles cannot be restored. Given the condition of the soft tissues at presentation, delayed fixation is recommended.

 

(SAE11OS.75) A 60-year-old woman sustains the injury shown in Figure 75. Prior to her injury, she lived independently and was a community ambulator without need for

any assistive devices. What treatment will give her the greatest long-term painless hip function with the lowest reoperation rate? 

 

 

 

  1. Total hip arthroplasty

  2. Hemiarthroplasty

  3. In situ percutaneous pinning

  4. Open reduction and percutaneous pinning

  5. Open reduction and internal fixation with an intramedullary device

 

PREFERRED RESPONSE 1

 

Cemented hemiarthroplasty is typically used to treat displaced femoral neck fractures in elderly patients. Recently, however, there has been a growing realization that many of these patients would be candidates for total hip arthroplasty had they presented with arthritis rather than fracture. Recent randomized studies have demonstrated improved outcomes up to 4 years following total hip arthroplasty compared with hemiarthroplasty in pain and functional scores. The rate of dislocation is higher following total hip arthroplasty. However, some patients with hemiarthroplasties required later conversion to total hip arthroplasty because of acetabular wear. In situ pinning is not recommended for patients with a displaced fracture. Open reduction and internal fixation of displaced femoral neck fractures in elderly patients is not recommended because of the risk of nonunion and osteonecrosis.

 

(OBQ15.212) A 25-year-old male sustains the isolated injury shown in Figure A. Antegrade intramedullary nailing is planned for definite fixation within the next 12 hours. After obtaining lateral radiographs of the injury site, what would be the next best step in management of this patient? 

 

 

  1. Convert to skeletal leg traction

  2. Radiograph/CT scan hip

  3. MRI scan hip

  4. Frog leg lateral hip radiograph

  5. Oncology consultation

 

PREFERRED RESPONSE 2

 

This patient has sustained a femoral shaft fracture. The next best step in management would be to CT scan the hip to assess for an associated femoral neck fracture.

 

Femoral neck fractures are seen less than 10% of the time with femoral shaft fractures, but they are frequently missed on initial evaluation. The neck fracture line is almost vertical and nondisplaced, or minimally displaced. Therefore fine (2-mm) cut CT scan through the femoral neck should be ordered in the preoperative workup of these patients.

 

Tornetta et al. reported that they reduced the delay in diagnosis of concomitant femoral neck fractures by 91% by instituting a protocol that included: dedicated AP internal rotation plain radiograph, a fine (2-mm) cut CT scan through the femoral neck, an intraoperative fluoroscopic lateral radiograph prior to fixation, as well as postoperative AP and lateral radiographs of the hip in the operating room prior to awakening the patient.

 

Figure A shows a trauma view lower extremity radiograph with an isolated left midshaft femur fracture.

 

Incorrect Answers:

Answer 1: Converting to skeletal leg traction would be necessary if a delay in treatment was expected. Answer 3: MRI scan of the left hip would not be necessary acutely for the identification of an associated fracture. Answer 4: Frog leg lateral hip radiograph would be much to painful to perform in the

setting of fracture. Answer 5: The age, injury and fracture pattern are not consistent with a pathologic femur fracture.

 

(SAE12TR.48) Figures 48a and 48b are the initial radiograph and clinical photograph of a 21-year-old woman who sustained a severe mangling injury to her left foot in a rollover ATV crash. After multiple dTbridements, she underwent definitive transmetatarsal amputation with split-thickness skin graft. She did have intact plantar sensation. Figure 48c shows the clinical photograph 10 days after amputation. Her wounds healed without infection and she did not require further surgery. At 2 years after injury, which of these factors is most likely to be associated with improved level of satisfaction? Review Topic

 

 

 

  1. Female gender

  2. Use of skin graft

  3. Ability to return to work

  4. Retained plantar sensation

  5. Initial treatment by amputation

 

PREFERRED RESPONSE 3

 

The LEAP study is a multicenter prospective study evaluating multiple aspects of reconstruction versus amputation in the treatment of mangled extremity injuries. With regard to patient satisfaction, treatment variables such as decision for reconstruction versus amputation, or initial presence or absence of plantar sensation have little impact. In addition, demographic factors such as age, gender, socioeconomic status,

and education level do not predict patient satisfaction. Instead, the most important predictors of patient satisfaction at 2 years after injury include the ability to return to work, absence of depression, faster walking speed, and decreased pain.

 

(SBQ12TR.26) A 44-year-old male presents after being struck by a car. He is hypotensive, and has an obvious open tibia and a closed femoral shaft fracture. A pelvic radiograph is obtained and shown in Figure A. A representative CT scan image in shown in Figure B. The patient is placed in a pelvic binder, and his blood pressure normalizes with crystalloid and a blood transfusion. He then undergoes pelvic angiography, and his internal iliac artery is embolized. Which of the following definitive treatment options is most appropriate in this case? Review Topic

 

 

 

  1. Anterior pelvic external fixation with pins placed in the AIIS and a posterior sacroiliac screw

  2. Open reduction and internal fixation of the pubic symphysis with a two hole plate and posterior fixation with a iliosacral screw

  3. Open reduction and internal fixation of the pubic symphysis with a non-locking multi-hole plate and posterior fixation with a iliosacral screw

  4. Anterior pelvic external fixation and posterior triangular osteosynthesis

  5. Open reduction and internal fixation of the pubic symphysis with a locked multi-hole plate and posterior sacral plate fixation

 

PREFERRED RESPONSE 3

 

The radiographs and CT scan show a fracture pattern consistent with an APC-III injury. This is most appropriately treated with pubic symphysis ORIF with a multi-hole plate and posterior iliosacral screw fixation.

 

APC-III pelvic ring injuries are associated with with disruption of the anterior and posterior SI ligaments (SI dislocation) as well as disruption of sacrospinous and sacrotuberous ligaments. They are commonly associated with vascular injury and retroperitoneal bleeding.

Sagi et al studied the radiographic and clinical outcome of symphyseal plating techniques, specifically comparing two hole (THP) vs multi-hole plating (MHP). Retrospective review of charts and radiographs immediately after the index procedure to latest follow-up was performed. When comparing the 2 different methods of anterior fixation, they found that the rate of fixation failure was greater in group THP (33%) than group MHP (12%). When evaluating the presence of a malunion as a result of these 2 treatment methods, there were more present in the THP group (57%) than in the MHP group (15%). On the basis of these findings, the authors recommended multi-hole plating of unstable pubic symphyseal disruptions.

 

Grimshaw et al performed a biomechanical study to determine the failure risk and potential benefit to use of locked fixation constructs in simulated pelvic ring injuries. The authors used a six-hole 3.5-mm plate specifically designed for the symphysis pubis with the capability of fixation in locked or unlocked mode, six pelves were fixed with locked screws and six pelves were fixed standard unlocked bicortical screws. Biomechanical testing was performed, and no abrupt failures were noted. However, locked plating of the pubic symphysis did not appear to offer any advantage over the standard unlocked technique

Illustrations A, B, and C show the AP, outlet, and inlet postoperative radiographs. Incorrect Answers:

1,4-Anterior pelvic fixation is typically not used as definitive treatment of these

injuries unless performed in a damage-control setting. The patient in this case is stable.

2-Multi-hole plate fixation of the pubic symphysis is preferred over 2-hole plate fixation.

5-Posterior sacral plating is not indicated in this fracture pattern, as there is no significant sacral comminution of vertical shear component. Furthermore, no distinct advantage has been found with locked symphyseal plating.

 

(SAE09TR.78) Figure 50 shows the radiograph of a 26-year-old man who sustained an isolated open injury to his foot. Examination reveals no gross contamination in the wound. There is a palpable dorsalis pedis pulse and sensation is present on the dorsal and plantar aspects of the foot. Initial treatment should consist of wound debridement, antibiotics, and 

 

 

  1. talectomy.

  2. reimplantation of the talus.

  3. reimplantation of the talus with acute triple arthrodesis.

  4. Syme amputation.

  5. transtibial amputation.

 

PREFERRED RESPONSE 2

 

The radiograph shows a complete extrusion of the talus. Reimplantation of the talus after wound debridement has been reported to be safe and successful, and provides for flexibility with any future reconstructive procedures.

 

(SAE08UE.19) Which of the following surgical devices employed for stabilization of the sternoclavicular joint is associated with the highest incidence of life-threatening complications? 

 

  1. Percutaneous pins

  2. Cannulated screws

  3. Cerclage wire

  4. Balser plate

  5. AO locking plate

 

PREFERRED RESPONSE 1

 

Numerous reports have documented serious complications including death from migration of intact or broken Kirschner wires or Steinmann pins into hilar structures such as the heart, pulmonary artery, and the aorta.

 

(OBQ15.208) Patient outcomes can be improved and early post-injury management errors decreased by adhering to the Advanced Trauma Life Support (ATLS) protocol. This protocol includes criteria for inter-hospital transfer when the patient's needs exceed the resources available. Which of the following would it be acceptable to manage without interfacility transfer to a trauma center? 

 

  1. A 29-year-old man with an open pelvic fracture

  2. A 42-year-old man with a major extremity crush injury

  3. A 31-year-old woman with an open forearm fracture

  4. A 36-year-old man with a spleen laceration after blunt trauma who is clinically stable and whose spleen laceration does not meet criteria for operative management

  5. An 18-year-old woman with multiple proximal long bone fractures

 

PREFERRED RESPONSE 3

 

Patients with severe open fractures should be managed at a trauma center. A patient with an isolated open forearm fracture may not require transfer if her injury is isolated, relatively free of contamination and not otherwise complicated by other factors such as tissue ischemia or tissue loss.

 

While most patients can receive all their definitive care at any hospital to which they present, early recognition of those that require transfer to a trauma center is essential to maximizing outcomes. Inherent in such identification is physician's' assessment of their own and their institutions' abilities and the limits thereof.

 

Van Olden et al. prospectively studied severely injured patients consecutively presenting to two community hospitals in the Netherlands. Comparison was made between patients treated prior to and after introduction of ATLS training to physicians at the studied hospitals. There was no difference in overall mortality between the two groups. however, there was a significant reduction in mortality in the first 60 minutes after admission, from 24.2% pre-ATLS to 0.0% post-ATLS.

 

Ali et al. studied trauma outcomes and mortality in patients presenting to the largest hospital in Trinidad and Tobago for the four years prior to and after introduction of ATLS training for physicians in the emergency department. Trauma mortality decreased to 33.5% from 67.5% after introduction of ATLS. Post-injury functional outcomes were also improved.

 

Illustration A shows the ATLS guidelines for interfacility transfer in table format. Incorrect answers:

Answer 1. Open pelvic fracture is an indication for transfer. Answer 2. Major crush is an indication for transfer. Answer 4. Expectant management of splenic and liver lacerations is predicated on the immediate availability of operative facility and appropriate staffing should the patient destabilize during non-operative management. Therefore, these patients must be transferred if expectant management is pursued. Answer 5. The presence of multiple proximal long bone fractures is an indication for transfer.

 

(SAE09TR.18) Which of the following factors is a significant predictor of reoperation following open reduction and internal fixation of intertrochanteric fractures with a sliding-compression hip-screw device? 

 

  1. Standard obliquity fracture pattern

  2. Tip-apex distance of 15 mm

  3. Fracture through the lateral femoral cortex

  4. Sliding-compression hip-screw device with a two-hole side plate

  5. Fracture of the lesser trochanter

 

PREFERRED RESPONSE 3

 

As shown by Palm and associates from the Hip Fracture Study group, the integrity of the lateral femoral cortex in intertrochanteric hip fractures is a significant predictor of reoperation. Baumgartner and associates have shown that a tip-apex distance of greater than 25 mm is associated with a high risk of femoral head cut-out. Lastly, intertrochanteric hip fractures can be described as standard obliquity or reverse obliquity when describing the fracture pattern. Mechanistically, a reverse obliquity pattern is important to recognize because it reflects the presence or absence of a lateral buttress to which the proximal fracture fragment may compress.

 

(SAE10PE.30) A 10-day-old girl has decreased active motion of the left upper extremity. The mother reports a difficult vaginal delivery with presumed shoulder dystocia. Examination shows full passive range of motion of the shoulder, elbow, and

wrist but only active flexion of the fingers and wrist. Factors predictive of a good outcome include which of the following? 

 

  1. Breech delivery

  2. Absence of an ipsilateral clavicle fracture

  3. Horner's sign and an APGAR score of 10 at 1 minute

  4. Return of active biceps before 3 months and preservation of full passive shoulder range of motion

  5. Absent Moro and Babinski reflexes

 

PREFERRED RESPONSE 4

 

Return of active biceps before 3 months and preservation of full passive shoulder range of motion are predictors of a good outcome. Breech delivery is usually associated with preganglionic injury. Preganglionic injury can result in a Horner's sign, which includes ptosis, myosis, and anhydrosis. Preganglionic injuries are unlikely to recover. The Moro reflex is elicited by dropping a baby's head a short distance and observing active elbow extension and fanning of the fingers, followed by elbow flexion and crying. Absence of the Moro reflex suggests a poor prognosis.

 

(SAE11OS.166) At what age does the lateral epicondyle normally ossify in males? 

 

  1. 2 to 4 years

  2. 5 to 6 years

  3. 7 to 8 years

  4. 9 to 11 years

  5. 12 to 14 years

 

PREFERRED RESPONSE 5

 

The lateral epicondylar epiphysis is the last to ossify in the elbow at age 12 to 14 years in males. The first secondary ossification center to ossify is the capitellum, which ossifies during the first 6 months of life. Next is the radial head, ossifying between age 3 and 6 years. The medial epicondyle appears between 5 and 7 years; the trochlea and olecranon at 8 and 10 years, respectively. In females, the appearance of ossification centers is about a year earlier than males.

 

(OBQ13.188) A 17-year-old football player is tackled with an opposing player's helmet hitting him hard in the abdomen. He is knocked backwards and suffers a diaphyseal femur fracture. He denies any loss of consciousness. Vital signs reveal a heart rate of 118, mean arterial pressure (MAP) of 68, and a respiration rate of 32 per minute. A FAST ultrasound study shows trace free fluid in the perisplenic space. A CBC taken prior to bolus IV fluids reveals a hematocrit of 48%, and a blood gas shows a lactate level of 1.8 and a base excess of -2.0.

 

Which of the follow statements regarding the patient's hemodynamic status is correct? 

 

  1. A well-placed and well-calibrated arterial line would be the most helpful clinical tool for determining when this patient is out of shock

  2. The hematocrit well within normal limits means the patient is not in hemodynamic shock

  3. A combination of heart rate greater than 120 and MAP less than 65 equates to poor tissue perfusion levels

  4. Normal lactate levels and base excess are markers of adequate tissue perfusion

  5. His orthopaedic injury alone cannot explain his vital sign derangements and an exploratory laparotomy is indicated

 

PREFERRED RESPONSE 4

 

Normal lactate levels or base excess indicate adequate tissue perfusion.

 

Hypovolemic shock leads to poor tissue perfusion due to inadequate flow or oxygenation. If a patient is in compensated shock (i.e. normal vital signs), there may be ongoing inadequate perfusion of some end-organs. Elevated lactate or a base deficit are markers of poor end-organ perfusion, thus when normalized indicate appropriate end-organ perfusion even if vital sign derangements persist.

 

Rossaint et al. wrote a comprehensive review article in 2006 in which they discuss principles of fluid management, coagulopathy, hypothermia and tissue oxygenation in hypovolemic shock. In addition to prolonged elevated lactate levels correlating to mortality, lactate levels (or base deficits) can be used to evaluate for compensated shock in the setting of normal hemodynamic status.

 

Illustration A shows the classification of hypovolemic shock. Note the percent of blood loss required for vital sign abnormalities.

 

Incorrect Answers:

Answer 1: While an arterial line is often helpful in the setting of uncompensated hemodynamic shock, it would not be as important as measures of adequate tissue perfusion for ruling out compensated shock. Answer 2: The hematocrit is expected to stay normal for a short time even in the

setting of massive blood loss. The hematocrit only changes once the patient has physiologic or iatrogenic fluid shifts in response to the blood loss. Answer 3: Vital sign derangements indicate uncompensated shock, but do not directly measure tissue perfusion or end-organ damage Answer 5: Though uncommon, bleeding from isolated femur fractures can lead to Class II shock (blood loss 15-30%)

 

(SAE08OS.91) What is the most common complication associated with open reduction and internal fixation using a 90/90 plate configuration and olecranon osteotomy for an OTA type C2 distal humerus fracture? 

 

  1. Nonunion of the lateral column

  2. Nonunion of the medial column

  3. Nonunion of the olecranon osteotomy

  4. Pain related to the plates

  5. Pain related to the olecranon fixation

 

PREFERRED RESPONSE 5

 

The most common complications associated with open reduction and internal fixation of distal humerus fractures are those associated with repair of an associated olecranon osteotomy. Complications associated with olecranon osteotomy fixation include failure of fixation (5%) and the need for secondary removal of painful hardware (70%). Nonunion of a distal humerus fracture treated with 90/90 plating is uncommon and results from inadequate fixation, excessive soft-tissue stripping, or use of inadequate plate fixation such as one third tubular plates. Heterotopic ossification is seen in approximately 4% of cases, infection 4%, and ulnar nerve palsy 7%. Although a relatively minor complication, the need for removal of painful hardware from the olecranon osteotomy is by far the most common complication seen in these cases.

 

(SAE11OS.23) A subtrochanteric femur fracture in which the lesser trochanter is intact is associated with what deformity? 

 

  1. Adduction and extension of the proximal fragment

  2. Adduction and flexion of the proximal fragment

  3. Abduction and extension of the proximal fragment

  4. Abduction and flexion of the proximal fragment

  5. Predominantly internal rotation of the proximal fragment

 

PREFERRED RESPONSE 4

 

The most commonly seen deformity in subtrochanteric femur fractures is abduction and flexion of the proximal fragment. Subtrochanteric fractures can pose challenges in reduction because of the muscle attachments proximal and distal to the fragment. The gluteus medius and gluteus minimus attach to the greater trochanter and abduct the proximal fragment. The iliopsoas attaches to the lesser trochanter, flexing and externally rotating the proximal fragment. The short external rotators (piriformis, superior and inferior gamellus) and the obturator internus also cause external rotation of the proximal fragment.

 

(SAE11OS.96) A 23-year-old woman sustains the injury seen in Figures 96a and 96b. Treatment should consist of which of the following? 

 

 

 

  1. Total elbow arthroplasty

  2. Closed reduction and casting

  3. Open reduction and internal fixation of both the radial head and distal humerus

  4. Open reduction and internal fixation of the radial head and excisional arthroplasty of the distal humerus

  5. Hinged external fixation with associated ligamentous reconstruction PREFERRED RESPONSE 3

This injury represents a complex partial articular fracture of the distal end of the humerus with an associated radial head fracture. Given this patient's young age, partial or complete arthroplasty is not an option. Closed reduction will lead to poor elbow function. Ligamentous repair is not indicated and external fixation will not aid in articular reduction. The patient requires open reduction and internal fixation of both components of the intra-articular injury. This is best accomplished through an extensile lateral approach or an olecranon osteotomy. Headless screws are preferred for articular reconstruction in these cases.

 

(SAE12TR.39) Figures 39a and 39b are the radiographs of a 45-year-old man with diabetes who fell 12 feet from a ladder and sustained an isolated closed injury to his left leg. Examination revealed that he was neurovascularly intact and compartments were soft. A damage control knee spanning external fixator was applied and after 2 weeks in the frame, his blisters have resolved and his skin now wrinkles. What is the most appropriate treatment? 

 

 

 

  1. Conversion to a periarticular 'hybrid' frame

  2. Open reduction and internal fixation with a lateral nonlocking plate

  3. Open reduction and internal fixation with a lateral locking plate

  4. Open reduction and internal fixation with medial and lateral plates

  5. Open reduction and internal fixation with posteromedial and lateral plates

 

PREFERRED RESPONSE 5

 

The patient has sustained a severely comminuted bicondylar fracture of the tibial plateau. The mechanism and radiographs highlight the high-energy mechanism of the injury and should warrant aggressive monitoring for compartment syndrome which is relatively common in this scenario. A staged surgical approach is warranted with application of a spanning damage control external fixator to maintain length and

alignment while the soft-tissue injury recovers and to allow for surveillance and examination of the limb. The radiographs reveal a comminuted bicondylar pattern with significant depression of the lateral articular surface and a split fracture with condylar widening. This element of the fracture will require direct elevation of the joint surface and reduction/buttress of the lateral condyle. This is best achieved with a lateral plate with subchondral rafting screws. The medial articular surface is coronally split and the posteromedial fragment is displaced. This fragment requires direct reduction and buttress via a separate posteromedial approach which is frequently performed prior to the lateral approach and fixation. A lateral buttress plate or a lateral locking plate alone does not reliably capture or adequately support the displaced posteromedial fragment. A medial and lateral plate construct is less soft-tissue friendly, particularly if inserted through a single incision. A medial plate would also fail to give direct buttress to the posteromedial fragment.

 

(SAE12TR.58) A 41-year-old man is involved in a high-speed motor vehicle crash and sustains a closed femoral midshaft fracture and a unilateral pulmonary contusion with a hemothorax, requiring placement of a chest tube. He has an initial blood pressure of 90/50 mm Hg. After receiving two liters of crystalloid, he has a blood pressure of 115/70 mm Hg and a heart rate of 90 bpm. He has normal mentation and does not require ventilator support. An arterial blood gas reveals that his delta base is

-2 mmol/L. What is the most appropriate treatment for his femoral fracture? Review Topic

 

  1. Skeletal traction

  2. Temporizing external fixation

  3. Reamed intramedullary nailing

  4. Unreamed intramedullary nailing

  5. Open reduction and internal fixation

 

PREFERRED RESPONSE 3

 

The patient responded to crystalloid resuscitation and hemodynamic parameters and the base deficit indicate that he is adequately resuscitated for definitive fracture care. In a resuscitated patient, a reamed nail is not detrimental in the setting of a pulmonary injury and is favorable for fracture union. An unreamed nail has a higher nonunion rate than a reamed nail for femoral fractures. In a skeletally mature patient with a midshaft fracture, an intramedullary nail is preferred to open reduction and internal fixation. In an adult patient, skeletal traction should be considered only as a temporary treatment prior to surgical fixation of the femoral fracture.

 

(SAE11UE.91) Figure 91 shows the radiograph of a 57-year-old man who fell 6 feet off a ladder. He is neurovascularly intact but reports shoulder pain. What is the most appropriate acute treatment for this patient? 

 

 

 

  1. Physical therapy for range of motion, advancing to strengthening as tolerated

  2. Sling immobilization and a recheck in 1 week with radiographs

  3. CT scan of the shoulder

  4. Open reduction and surgical stabilization with plates and screws

  5. Ice, nonsteroidal anti-inflammatory drugs, and activity as tolerated

 

PREFERRED RESPONSE 2

 

The patient has sustained a traumatic surgical neck fracture of the humerus. Sling immobilization and a recheck in 1 week with radiographs is appropriate to check for maintenance of alignment. The fracture is minimally displaced and therefore does not require surgical stabilization or further diagnostic imaging. Surgical reduction and plating is not indicated in this nondisplaced fracture. Physical therapy and activity as tolerated at this point are contraindicated because of the acuity of the fracture.

 

(OBQ15.57) A 26-year-old male underwent statically locked intramedullary nail fixation for a comminuted left femur fracture. An early post-operative computed tomography (CT) scanogram was taken to check rotational alignment, as shown in

Figure A. What would be the next best step in the management of this patient? 

 

 

 

 

  1. Observation and close follow-up

  2. Dynamization of the intramedullary nail

  3. Revision surgery, internally rotate distal fragment by 19 degrees

  4. Revision surgery, externally rotate distal fragment by 8 degrees

  5. Revision surgery, internally rotate proximal fragment by 11 degrees

 

PREFERRED RESPONSE 1

 

The CT scanogram shows the operative left femur is 8 degrees externally rotated compared to the native right femur. No correction is required unless malalignment is

>15 degrees and symptomatic. Therefore, the most appropriate next step would be to continue with postoperative observation and close follow-up.

 

The primary purpose of CT scanogram is to measure the angle of rotation of the femoral neck relative to the femoral condyle. To do this, the right and left femurs must be scanned together using a 5mm helical slice scanner at the hip and knee. The first slice should reveal the alignment of the femoral neck, so as to allow for measurement of the femoral neck-to-horizontal (FNH) angle. The second slice should reveal the alignment of the posterior femoral condyles. This allows measurement of the posterior condyle-to-horizontal (PCH) angle. Finally, to calculate the rotational alignment (RA), the FNH angle and PCH angles are subtracted (e.g., RA = FNH -PCH). Normal RA is usually +5 to +20 degrees, which is also referred to as 5 to 20 degrees of femoral anteversion.

 

Lindsey et al. reviewed femoral malrotation following intramedullary nail fixation. They showed the incidence of rotational malalignment was ~28%. Normal femoral neck anteversion (angle of the femoral neck relative to the transverse axis through the femoral condyles) is ~11-13°. However, they noted that some patients have up to 15° difference in rotation in native limbs. Therefore <15 degrees of rotational difference after fixation is considered acceptable.

 

Gugala et al. examined the long-term functional implications for patients with iatrogenic femoral malrotation following femoral intramedullary nail fixation. They

showed that patients can compensate for even significant femoral malrotation (up to 30 degrees) and tolerate it well. However, external femoral malrotation (more common) appears to be better compensated/tolerated than internal malrotation.

 

Figure A shows that the left femoral neck is externally rotated (ER) by 15° to the horizontal (ER15). The right femoral neck is externally rotated (ER) by 4° to the horizontal (ER4). The left distal fragment is ER10. The right distal fragment is internally rotated (IR) by 9°. Thus, left femur has a total (ER15)-(ER10)= (+15)-(+10)=(+5), and right femur has (ER4)-(IR9)= (+4)-(-9)=(+13) to the horizontal. Therefore, the difference is 8 degrees.

 

Incorrect Answers:

Answer 2: Dynamization of the intramedullary nail is not appropriate as the left leg is shorter, and there is no mention of fracture gap distances or stability. Answers 3-5: Revision surgery would be indicated if rotational malalignment were

>15 degrees and symptomatic.

 

(SAE09FA.83) A 38-year-old man underwent a transtibial amputation for chronic posttraumatic foot and ankle pain and chronic calcaneal osteomyelitis. Postoperative radiographs are seen in Figures 41a and 41b. What is the proposed purpose of the surgical modification seen in the radiographs? 

 

 

 

  1. Reduces shrinkage of the residual limb

  2. Creates a more stable platform for load transfer

  3. Reduces wound healing complications by avoiding the soft-tissue dissection necessary to transect the fibula at a level proximal to the tibia

  4. Connecting bone strut provides an attachment point for more effective myodesis

  5. Allows a more proximal resection level to decrease tension on the wound

PREFERRED RESPONSE 2

 

The Ertl modification of a below-knee amputation has been proposed to create a more stable “platform” to aid in transferring the load of weight bearing between the residual limb and the prosthetic socket. It is felt that a stable platform allows total contact loading over an enlarged stable surface area. Early studies have suggested that this modification may enhance the patient’s perceived functional outcome.

 

(SAE12TR.86) What is the best way to determine whether a radial head implant is too thick intraoperatively? 

 

  1. Visually assess the radiocapitellar joint.

  2. Visually assess widening of the lateral ulnohumeral joint.

  3. Assess widening of the radiocapitellar joint on an AP radiograph.

  4. Assess the elbow for concentric reduction on a lateral radiograph.

  5. Assess widening of the medial ulnohumeral joint on an AP radiograph.

 

PREFERRED RESPONSE 2

 

Widening of the medial ulnohumeral joint on an AP radiograph is only visible after overlengthening of the radial head by 6 mm or more. At least in this cadaver study, the most sensitive method was to visually assess the lateral aspect of the ulnohumeral joint with the radial head resected and then with the trial radial head in place. This method allows detection of any overlengthening.

 

(SAE12FA.19) Which of the following occurs frequently after nonsurgical management of displaced intra-articular fractures of the calcaneus? 

 

  1. Return to normal function

  2. Narrowing of the calcaneus

  3. Lengthening of the calcaneus

  4. Plantar flexion of the talus

  5. Peroneal tendinitis

PREFERRED RESPONSE 5

 

Peroneal tendinitis and stenosis are typically seen following nonsurgical management and results from lateral subfibular impingement, whereby the displaced, expanded lateral wall subluxates the peroneal tendons against the distal tip of the fibula or might even dislocate the tendons. Nonsurgical management of displaced calcaneal fractures offers little chance for return to normal function because of the development of a calcaneal malunion. The articular surface is not reduced, the heel remains shortened and widened, the talus is dorsiflexed in the ankle mortise, and the displaced lateral wall causes impingement and binding of the peroneal tendons.

 

(OBQ15.139) Which of the following post-reduction forearm fractures patterns may be treated non-operatively in an otherwise healthy 22-year old male? 

 

  1. Displaced diaphyseal fracture of the radius

  2. Non-displaced diaphyseal fracture of the radius, displaced diaphyseal fracture of the ulna

  3. Displaced diaphyseal fractures of both bones of the forearm, with less than 10 degrees angulation after closed reduction

  4. An isolated mid-shaft ulna fracture translated 20%, with less than 5 degrees of angulation

  5. Gustilo grade II open fracture of the radius

 

PREFERRED RESPONSE 4

 

In adults, minimally displaced fractures of the ulna may be treated non-operatively.

 

Even in the setting of minimal displacement, fractures involving the radial diaphysis, or both bones of the forearm, are at high risk of displacing further and progressing to malunion or nonunion. Given the potential for a resulting loss of forearm rotation, open reduction internal fixation is indicated for almost all adult diaphyseal radius and both bone fractures.

 

Schulte et al. review the management of both bone forearm fractures in adults. They review biomechanics, fixation techniques, outcomes and complications. They note that the goals of fixation in simple patterns are 'cortical opposition, compression, and restoration of forearm geometry.'

 

Anderson et al. treated 330 acute diaphyseal forearm fractures with compression plating from 1960 to 1970. At 4 months to 9 years follow up, they achieved a 97.9%

union rate for the radius and 96.3% union rate for the ulna.

 

Illustration A shows measurement of radial bow. A dotted line perpendicular to the line drawn from the radial tuberosity to the ulnar aspect of the distal radius can be used to measure radial bow when drawn at the point of maximum distance to the ulnar edge of the radius.

 

Incorrect answers:

Answers 1, 5. Any diaphyseal fracture of the radius in an adult warrants internal fixation, even if non-displaced. Answers 2, 3. Any diaphyseal fracture of both bones of the forearm should be treated with internal fixation in adults.

 

(SAE09TR.38) Figures 23a and 23b show the radiographs of a 75-year-old woman who sustained an injury to her nondominant hand. Initial treatment should consist of 

 

 

 

  1. closed reduction and splinting.

  2. open reduction and internal fixation through a volar approach.

  3. external fixation and Kirschner wire fixation.

  4. intrafocal pinning and casting.

  5. acceptance of alignment and bracing.

 

PREFERRED RESPONSE 1

 

Definitive treatment decisions for displaced distal radius fractures in the elderly are based on a number of factors related to the fracture pattern and patient demographics. The first step in any treatment algorithm is a closed reduction and splinting with

reassessment of alignment parameters. This is an extra-articular fracture with dorsal angulation. Low-demand elderly patients can be treated well with accepted minor malreduction.

 

(SBQ12TR.38) A 58-year-old male is involved in a motor vehicle collision and sustains the injury shown in Figure A in addition to right 5th and 6th rib fractures. Upon evaluation in the emergency department, he is noted to have a 2 centimeter laceration over the anterior aspect of his left leg with visible bone. Vitals and labs are normal. Which of the following statements is most accurate regarding surgical management for this patient? Review Topic

 

 

 

  1. Reamed intramedullary nailing is favored due to increased rates of union

  2. Unreamed intramedullary nailing is favored due to presence of concomitant rib fractures

  3. Reamed intramedullary nailing is favored due to decreased rates of infection

  4. Unreamed intramedullary nailing is favored due to less local trauma

  5. Both unreamed and reamed intramedullary nailing are equivalent

 

PREFERRED RESPONSE 5

 

Both unreamed and reamed intramedullary nailing are equivalent treatments in patients with open tibia fractures. Intramedullary nailing is the treatment of choice for stable patients with tibial shaft fractures.

 

Tibial shaft fractures can be the result of low energy twisting injuries or higher energy axial loads. Closed fractures with acceptable alignment can be often be treated with closed reduction and casting. Intramedullary nailing, unreamed or reamed, is the treatment of choice for open fractures except in the setting of damage control

orthopaedics when an external fixator may be more appropriate.

 

Bhandari et al. investigated reamed and unreamed intramedullary nailing for tibial shaft fractures in a randomized trial ("SPRINT" Trial - Study to Prospectively Evaluate Reamed Intramedullary Nails in Patients with Tibial Fractures Investigators). They concluded that reamed nailing was more beneficial (decreased rate of primary outcome event: need for bone grafting, implant exchange or removal for infection, debridement for infection) for closed fractures, but had no benefit in open fractures.

 

Finkemeier et al. evaluated consecutive patients treated with unreamed and reamed intramedullary nailing and found similar rates of union in both open and closed tibial shaft fractures at six and twelve months.

Figures A shows AP and lateral xrays of the left tibia showing a tibial shaft fracture. Incorrect Answers:

Answer 1-4: Unreamed and reamed intramedullary nailing for open tibia fractures are

similar in terms of union rates, infection rates, and rates of re-operation. There is a theoretical risk of pulmonary complications with reaming but it has not been proven.

 

(OBQ14.128) A 24-year-old male presents following a motorcycle crash with an isolated injury to his right lower extremity. He has a 3x2cm wound over the fracture site, and he immediately receives Gram positive and Gram negative coverage along with a tetanus booster. The patient is splinted, optimized, and brought to the operating room where the wound is debrided and classified as a Type IIIB fracture. Deemed stable, the plastic surgery team arrives and acutely performs a free flap for coverage, following definitive fixation with an intramedullary nail. All of the following are factors that have been shown to increase infection risk EXCEPT: Review Topic

 

 

 

  1. Time to antibiotic administration

  2. Thoroughness of debridement

  3. Time to initial debridement

  4. Ability to close/cover an open wound

  5. Time to definitive fixation

 

PREFERRED RESPONSE 5

 

Time to definitive fixation is not a modifiable risk factor concerning open fractures. The other factors are risk factors that have been studied in regards to infection, and all are more important than definitive fixation. Definitive fixation can wait until complete closure and/or coverage.

 

When concerning management of open fractures, the most important factor is a thorough debridement. However, the quality of debridement is often not able to be quantified and thus, often not mentioned in studies. While early clinical and animal studies have shown that initial debridement should occur within 6 hours of injury, more recent clinical trials have not found a significant correlation within that urgent time frame, but rather recommend initial debridement as soon as possible within 24 hours. Time to antibiotic administration has been found to have a significant impact in lowering infection risk. Immediate administration in the emergency room is recommended. The ability to cover and/or close an open wound also has a significant impact on infection. Recent studies have recommended placing hardware after fasciotomy closure and have also demonstrated lower infection rates when flaps are placed within 72 hours of injury.

 

Pape and Webb concisely review the evolution of open fractures and wound management. The authors describe the early days where amputation was favored, to wet-to-dry dressings, to the advent of negative pressure wound therapy. Throughout, however, the authors emphasize the importance of soft tissue coverage. They also stress the importance of a technically thorough debridement, the most important factor of any wound management.

 

Scheneker et al. performed a systematic review and meta-analysis of 16 studies to determine if time to the operating room for debridement was an independent, modifiable risk factor in regards to subsequent infection following open tibia fracture. At the time of the study, the gold standard (based on a previous rat model), had recommended initial debridement within 6 hours of injury. The results of this meta-analysis, however, could not find conclusive evidence to suggest that late debridement alone placed the patient at a significantly higher risk for infection. The authors provided a moderate recommendation that initial debridement should occur as soon as possible within 24 hours, although more data is required in order to find a definitive time.

 

The SPRINT investigators report a landmark study that randomized over 1200 patients to either reamed or unreamed tibial IMN with the primary outcome analyzed as return to the operating room for either non-union treatment or deep infection. A notable difference between the two cohorts was a significantly higher primary event rate in the unreamed group.

Figure A exhibits a distal third open tibia fracture.

 

Incorrect answers:

Answer 1: Antibiotic administration as soon as an open fracture has been diagnosed is a significant risk factor in minimizing infection risk. Answer 2: Although a non-quantifiable measure, a thorough debridement is the most important component of treating an open fracture. Answer 3: Initial animal models cite a 6 hour window to initial debridement, however, clinical trials have not found a significant window that can affect increased or lowered infection risk. Answer 4: Coverage and/or closure of any open wounds or soft tissue defects is a significant factor in lowering infection risk; when flap coverage is needed, coverage within 72 hours is optimal.

 

(SAE09TR.58) Which of the following findings best describes the acetabular fracture shown in Figure 38? 

 

 

 

  1. Posterior column with articular impaction and a free fragment

  2. Anterior column with articular impaction

  3. Posterior wall with an intra-articular fragment

  4. Posterior wall with articular impaction and a free intra-articular fragment

  5. Posterior wall with articular impaction

 

PREFERRED RESPONSE 4

 

The CT scan shows a posterior wall fracture with impaction of the articular surface and a free fragment within the joint. Proper treatment of this injury requires not only reduction and fixation of the posterior wall fragment but also removal of the free fragment and elevation of the depressed articular segment.

 

(OBQ14.226) Figures A and B are radiographic images of an 85-year-old woman with isolated left hip pain. She describes a non-syncopal fall from standing 4 hours ago. Physical examination reveals pain with log-rolling the left thigh and the inability to bear weight on the affected leg. The radiologist reports no fracture in the left hip. What would be the next best step? Review Topic

 

 

 

  1. Stress view radiographs of the left hip

  2. Non-weight bearing and pelvic bone scan in 7 days

  3. Non-weight bearing and repeat the CT scan after 48 hours from injury

  4. MRI hip and pelvis

  5. Weight bearing activity as tolerated with close follow-up

 

PREFERRED RESPONSE 4

 

The next best step would be an MRI hip and pelvis to investigate for an occult fracture of the left hip.

 

Moderate evidence supports MRI as the advanced imaging of choice for diagnosis of presumed hip fracture not apparent on initial radiographs. MRI has been shown to be able to detect occult fractures earlier than bone scan, with better spatial resolution. Usually the MRI should be obtained in less than 24 hours from the time of injury. For situations in which MRI is not immediately available, bone scan can be considered after 72 hours form the time of injury. However, this may compromise patient care and put the patient at risk of fracture displacement.

 

Cannon et al. reviewed the imaging of choice in occult hip fracture. They showed that physical examination yields a poor sensitivity identifying occult hip fractures, with log-rolling and straight-leg raise as 50% and 70%, respectively. The most sensitive modality for occult fracture identification was MRI.

Iwata et al. retrospectively reviewed a cohort of 35 patients with clinically suspected fractures of the hip that underwent MRI. All radiographs were negative. In 26 of these patients, a T1-weighted coronal MRI showed a hip fracture with 100% sensitivity.

 

Roberts et al. reviewed the 2015 AAOS Clinical Practice Guideline: Management of Hip Fractures in the Elderly. They report moderate evidence that supports MRI as the advanced imaging of choice for diagnosis of presumed hip fractures not apparent on initial radiographs.

 

Figure A is a AP radiograph of the left hip and pelvis. Apart from a healed fracture of the ischiopubic rami and generalized osteopenia, there is no obvious hip fracture. Figure B is a coronal CT image that does not demonstrate evidence of an acute hip fracture. Illustration A is a T1 weighted MR image that shows a non displaced fracture (white arrow) through the intertrochanteric region of the left proximal femur.

 

Incorrect Answers:

Answer 1: Stress view radiographs would not be recommended. Answer 2: If MRI is contraindicated, for example if a pacemaker is present, then a bone scan at 72 hours is the next test of choice. Waiting 7 days would not be appropriate for this patient. Answer 3: Repeating the CT scan will not help to identify fracture, unless the fracture becomes displaced. A delay in identifying a fracture pattern by 48 hours, while the patient is non-mobile, significantly increases their risk of complications. Answer 5: Weight bearing activity as tolerated with close follow-up may be suggested if the patient is clinically able to walk and advanced imaging is negative for fracture. The best modality to rule-out occult fracture is MRI.

 

(SBQ12TR.32) A 20-year-old male is taken to the emergency department following a motorcycle collision with the injury seen in Figures A and B. He undergoes serial debridements with placement of an antibiotic bead pouch, followed by intramedullary nailing and free tissue transfer. His preoperative examination is notable for absent plantar sensation. Which of the following is predictive of a worse long term outcome in this patient? 

 

 

  1. Absent plantar sensation at presentation

  2. Depression at 3 months post-injury

  3. Use of an intramedullary nail

  4. Free tissue transfer instead of rotational flap

  5. Need for multiple debridements

 

PREFERRED RESPONSE 2

 

The presence of depression at the 3 months post-injury is a significant predictor of worse long term outcome as well as lower return to work rates in patients that present with limb threatening lower extremity trauma.

 

The findings of the Lower Extremity Assessment Project (LEAP study) has challenged many of the long-held beliefs of lower extremity trauma surgery, including the concept that patients with absent plantar sensation should undergo immediate amputation . The LEAP study has also brought to light the importance of socioeconomic and psychosocial factors in patient outcomes following these injuries.

 

Bosse et al analyzed a subgroup of 55 patients from the LEAP study with absent plantar sensation at presentation. Those patients that underwent limb salvage despite absent sensation (n=26) had no significant difference in functional outcome when compared with either insensate patients who underwent amputation or a matched control group of sensate patients with similar injuries. All but one patient in the salvage group had return of some sensation at 2-year follow up and the authors concluded that an insensate plantar foot at presentation should not be included in the algorithm for amputation.

 

Mackenzie and Bosse reviewed the results of the LEAP study with particular emphasis on the social and economic variables that influence outcomes in severe lower extremity. The authors note that 19% of patients screened positive for severe depression. The presence of anxiety, pain and depression at 3 months post-injury were significant predictors of poor outcome

 

Incorrect Answers:

Answer 1: Absent plantar sensation does not predict worse outcomes and patients may have significant return of sensation at 2-year follow up. Answer 3: No difference has been demonstrated with methods of internal fixation for type IIIB tibia shaft fractures. Answer 4: No difference has been shown with the use of free tissue versus rotational

tissue for type IIIB tibia fractures. Answer 5: The number of debridements has not been shown to influence outcome.

 

(SAE11OS.39) Tension band wire fixation is best indicated for which of the following types of olecranon fractures? 

 

  1. Comminuted fractures

  2. Fractures that involve the coronoid process

  3. Fractures associated with Monteggia fracture-dislocations

  4. Oblique fractures distal to the midpoint of the trochlear notch

  5. Transverse fractures through the midpoint of the trochlear notch

 

PREFERRED RESPONSE 5

 

Tension band wiring may not provide adequate stability to prevent displacement in a comminuted fracture. Plate fixation is most commonly recommended for comminuted fractures of the olecranon. Additionally, plate fixation is used for oblique fractures distal to the midpoint of the trochlear notch, fractures that involve the coronoid process, and those associated with Monteggia fracture-dislocations. Tension band wiring is best indicated for simple transverse fractures through the midpoint of the trochlear notch.

 

(OBQ14.111) The dominant arterial blood supply to the patella enters at which anatomical location? 

 

  1. Proximal pole

  2. Mid-lateral

  3. Mid-medial

  4. Directly anterior

  5. Distal pole

 

PREFERRED RESPONSE 5

 

The largest arterial contribution to the patella will enter at the distal (inferior) pole of the patella, with the dominant artery entering inferomedially.

The arterial blood supply to the patella is made up of branches of six main arteries: the descending genicular, the superior medial and lateral genicular, the inferior medial and lateral genicular, and the anterior genicular. Several of these branches contribute to the anastomotic network that surround the patella. From the ring, there are two main interosseous blood supply systems to enter the patella, known as the midpatellar and polar vessel systems. The distal pole of the patella is considered to be the largest arterial contribution to the peripatellar ring and the polar vessel system.

 

Lazaro et al. used twenty matched pairs of fresh-frozen cadaveric knees to isolate the dominant blood supply to the patella. After cannulating the superficial femoral artery, anterior tibialis artery, and posterior tibialis artery and performing magnetic resonance imaging, they found that the largest arterial contribution to the patella entered at the inferior pole in 100% of the specimens. In sixteen specimens (80%), the dominant artery entered the medial aspect of the distal pole. In three specimens (15%), it entered the lateral aspect of the distal pole.

 

Illustation A shows the arterial supply system to the patella. The dominant arterial supply enters at the distal (inferior) pole of the patella, with the dominant geniculate arteries entering inferomedially (labelled with a green star). Illustration B shows an anatomical illustration of the patellar blood supply. Note the dominant distal pole blood supply (arrow).

 

Incorrect Answers:

Answers 1-4: Two main arterial systems (midpatellar and polar vessel systems) form the intraosseous blood supply of the patella, which are supplied by the medial/lateral and superior/inferior genicular branches, respectively. The dominant arterial supply comes from the inferior branches.

 

(SAE12TR.53) Figure 53 is the radiograph obtained at the time of transfer to the trauma center of a 41-year-old man who was involved in a motor vehicle accident. What is the most appropriate initial management? 

 

 

  1. MRI scan

  2. CT scan of the pelvis

  3. Application of skeletal traction

  4. Closed reduction of the right hip

  5. Open reduction and internal fixation

 

PREFERRED RESPONSE 4

 

The radiograph reveals a displaced transverse posterior wall acetabular fracture, and the hip is dislocated. On recognition of a hip dislocation, the hip should be promptly reduced. A time to reduction of greater than 12 hours has been associated with adverse outcomes. Although skeletal traction and a CT scan are essential elements in this patient's care, the hip should be reduced prior to these actions. An MRI scan is not indicated in this patient, particularly with the hip dislocated. The ultimate surgical treatment for this injury will be open reduction and internal fixation, but the patient should be stabilized, the hip reduced, and appropriate imaging obtained before taking the patient to surgery.

 

(SAE09SN.65) Sacral fractures are most likely to be associated with neurologic deficits when they involve what portion of the sacrum? 

 

  1. Zone 1 (the ala)

  2. Zone 2 (the foramina)

  3. Zone 3 (the central canal)

  4. Zones 1 and 2

  5. The sacral laminae

PREFERRED RESPONSE 3

 

Denis divided the sacrum into three zones: zone 1 represents the lateral ala, zone 2 represents the foramina, and zone 3 represents the central canal. A fracture is classified according to its most medial extension. Those in zone 3 are typically bursting-type fractures or fracture-dislocations and are most prone to neurologic sequelae.

 

(SBQ12TR.66) A 38-year-old male is involved in a high speed motor vehicle collision. He has a Glasgow Coma Scale of 13 and receives 2 liters of fluid en route to the emergency department. Upon evaluation in the emergency department, he is found to have a bilateral femoral shaft fractures, a right ankle fracture, and a left both bone forearm fracture. He also has 2 left sided rib fracture and a grade II liver laceration. His heart rate is 130 and blood pressure is 85/50. All of the following would be indications to practice damage control orthopaedics in this patient except: Review Topic

 

QID:3981

  1. Bilateral femur fractures

  2. Rib fractures

  3. Lactate of 5.2

  4. Urine output of 20 cc/hr

  5. Heart rate and blood pressure

 

PREFERRED RESPONSE 2

 

Rib fractures without evidence of further thoracic trauma would not be an indication to practice damage control orthopaedics. This patient is underresuscitated based on his lactate level, urine output, and vital signs and definitive management should be delayed.

 

Damage control orthopaedics is the practice of delaying definitive management of fractures and utilizing temporary stabilization (such as an external fixator) until a patient has recovered from the initial physiologic insult of trauma. Patients are at increased risk for perioperative complications such as ARDS and multi-system organ failure during the acute period after polytrauma. In addition to underresuscitation, other indications to practice damage control orthopaedics include: injury severity score>40 (or >20 with thoracic trauma), bilateral femoral fractures, hypothermia below 35 degrees Celsius, and pulmonary contusions.

Pape et al. (2007) studied the incidence of acute lung injuries in polytrauma patients undergoing either intramedullary nailing or external fixation and later definitive fixation of femoral shaft fractures. They found that patients undergoing immediate intramedullary nailing were nearly 6.7 times more likely to have acute lung injury

 

The Canadian Orthopedic Trauma Society studied the effect of reamed versus unreamed femoral nailing on incidence of ARDS for femoral shaft fractures in trauma patients using a randomized controlled study. They found no difference between the groups.

 

Pape et al. also examined the pathophysiological cascades that accompany soft tissue injuries of the extremities, abdomen, and pelvis and recommend a more comprehensive for evaluation of patients with these injuries.

 

Incorrect Answers:

Answer 1: Bilateral femur fractures are an indication to practice damage control orthopaedics and delay definitive fixation Answers 3,4,5: All of these answers suggest that the patient is underresuscitated.

Definitive fixation should be delayed

 

(SAE08UE.44) A 32-year-old woman sustained an elbow dislocation, and management consisted of early range of motion. Examination at the 3-month followup appointment reveals that she has regained elbow motion but has a weak pinch. A clinical photograph is shown in Figure 21. What is the most likely diagnosis? Review Topic

 

 

 

  1. Flexor pollicis longus rupture

  2. Median nerve palsy

  3. Ulnar nerve palsy

  4. Anterior interosseous nerve palsy

  5. Posterior interosseous nerve palsy

PREFERRED RESPONSE 4

 

The photograph shows the characteristic attitude of the hand when an anterior interosseous nerve palsy is present. The patient is unable to flex the interphalangeal joint to the joint of the thumb. Anterior interosseous nerve palsies are often misdiagnosed as tendon ruptures.

 

(SBQ12TR.54) A 37-year-old male cashier is shot in the leg. He sustains the injury shown in Figures A and B, and is subsequently taken to the operating room for intramedullary nailing. Figure C shows a radiograph of the nail starting point (*). What complication is most likely to result? 

 

 

 

  1. Varus malunion

  2. Nonunion

  3. Valgus malunion

  4. Malrotation

  5. Superficial peroneal nerve injury

 

PREFERRED RESPONSE 3

 

This patient is presenting with a comminuted fracture of the proximal third of the tibia. He is appropriately undergoing intramedullary nail fixation, however, the start point illustrated in Figure C is too medial and often leads to a valgus malunion.

Intramedullary nail fixation is more technically demanding in proximal tibial fractures than diaphyseal fractures. The valgus deformity is due to imbalanced muscle forces on the proximal fragment, which are then accentuated by a start point that is too medial. An apex anterior (procurvatum) deformity can also occur and results from the pull of the patellar tendon or a posteriorly directed nail that deflects off the posterior tibial cortex and rotates the proximal fragment. The ideal starting point for proximal tibial fractures is slightly lateral to the medial aspect of the lateral tibial spine on a true AP x-ray and very proximal and just anterior to the anterior margin of the articular surface.

 

Nork et al. reported the results of intramedullary nailing of proximal tibial fractures with emphasis on techniques of reduction. Various techniques were found to be successful including attention to the proper starting point, the use of unicortical plates, and the use of a femoral distractor applied to the tibia.

 

Lowe et al. describe surgical techniques for complex proximal tibial fractures. They describe the extended leg position, use of a femoral distractor, temporary plate fixation, blocking (Poller) screws, and use of percutaneous clamps as means to achieve reduction during fixation.

 

Figure A and B show an AP and lateral radiograph of a comminuted extra-articular fracture through the proximal third of the tibia. Figure C demonstrates a start point that is too medial (represented by the asterisk) for intramedullary nail fixation. Illustration A and B show the ideal start point for intramedullary nail fixation of the tibia on AP and lateral radiographs.

 

Incorrect Answers:

  1. Varus malunion is more likely to occur in midshaft tibia fractures with an intact fibula.

  2. Nonunion after a proximal tibial fracture treated with intramedullary nailing is less common than malunion.

  1. Malrotation occurs most commonly after IM nailing of fractures through the distal third of the tibia.

  2. The superficial peroneal nerve is at risk during distal screw fixation using a LISS plating technique for fracture fixation.

 

(SAE12TR.15) Figures 15a and 15b are the radiographs of a 28-year-old man who fell from a height and sustained an isolated closed diaphyseal femur fracture that was treated with reamed antegrade femoral nailing 8 months ago. He now reports persistent pain during ambulation. He smokes one pack of cigarettes per day but is otherwise healthy. He denies any infectious history or symptoms, and laboratory

studies show a normal WBC count, erythrocyte sedimentation rate, and C-reactive protein. What is the most appropriate treatment? 

 

 

 

  1. Bone graft in situ

  2. External bone stimulation

  3. Reamed antegrade exchange nailing

  4. Reamed retrograde exchange nailing

  5. Open biopsy to assess for infection followed by delayed surgical stabilization

 

PREFERRED RESPONSE 3

 

The patient has an uninfected symptomatic nonunion of the femur after reamed antegrade nailing with an appropriately sized implant. The fracture is well aligned and has some callus response indicating reasonable vascularity. Auto-dynamization has occurred via fatigue failure of the distal interlocking screws but the patient remains symptomatic and the fracture line is evident, consistent with nonunion. Reamed exchange nailing is preferred because it allows for improved mechanics via a larger diameter nail and repeat interlock and improved biologics via reaming which is felt to elicit an inflammatory reaction and generate bone graft in situ. Unfortunately, the results of exchange nailing are not as good in patients who smoke and smoking cessation should be counseled and encouraged. The data on external bone stimulation on unhealed fractures of the femur with an intramedullary nail present are lacking. In light of a benign clinical examination and history and normal blood work parameters with reference to infection, open biopsy of the nonunion prior to definitive surgical treatment is unwarranted. The patient has mechanical instability and bone grafting in situ will not address this issue in terms of promoting progression to union or allowing for improved function and less pain.

(SAE09TR.39) A 43-year-old man sustained a closed, intra-articular pilon fracture. It has now been 1 year since he underwent open reduction and internal fixation. Which of the following statements most accurately describes his perceived outcome? Review Topic

 

  1. His clinical outcome will correlate closely with his initial reduction.

  2. His outcome will correlate with his radiographic score on the Ankle Osteoarthritis Score.

  3. He will likely require a late ankle arthrodesis.

  4. He will demonstrate marked limitations with regard to recreational activities.

  5. He will perceive improvements for a period of over 2 years.

 

PREFERRED RESPONSE 5

 

Marsh and associates retrospectively reviewed 56 tibial plafond fractures and found that the patients perceived improvement in their function and pain for an average of

2.4 years. They demonstrated some limitations in recreational activities but not marked limitations. Patients were unlikely to need a late arthrodesis (13%), and their outcomes did not correlate well with assessments of reduction or arthritis scores.

 

(SAE08OS.21) During an ilioinguinal approach for fixation of the anterior pelvic ring, brisk bleeding is encountered as the dissection is extended along the superior pubic ramus approximately 5 cm from the midline. What structure has most likely been injured? 

 

  1. A branch of the femoral artery

     

  2. An anastomosis between the external iliac artery and obturator artery

     

  3. An anastomosis between the external iliac and femoral artery

     

  4. Internal iliac artery

     

  5. External iliac artery

 

PREFERRED RESPONSE 2

The corona mortis, or "crown of death," is a common anatomic variant that consists of an anastomosis between the obturator and the external iliac or inferior epigastric arteries or veins. Its reported incidence is over 80%. It is located behind the superior pubic ramus at a variable distance from the symphysis pubis (3 cm to 9 cm). It is at risk during surgical approaches to the anterior pelvic ring. If accidentally cut, the vessel can retract making control of hemorrhage difficult.

 

(SAE09FA.30) Figures 17a through 17c show the radiographs of a 38-year-old man following a motorcycle accident. The posterior portion of the talus extruded through a posterolateral wound. The extruded talar body is visible in the wound along with some road debris. Management should now consist of surgical irrigation, debridement, and 

 

 

 

  1. removal of the extruded talus and placement of an external fixator.

  2. immediate tibiocalcaneal fusion.

     

  3. reimplantation of the talus, external fixation, and/or open reduction and internal fixation of the talar neck fracture.

     

  4. reimplantation followed by primary tibiotalar arthrodesis.

  5. Syme amputation.

 

PREFERRED RESPONSE 3

 

The extruded talus should be placed in sterile bacitracin solution, irrigated thoroughly, gently debrided, and immediately replanted in the OR. Open reduction and internal fixation of the talar fracture may be attempted immediately depending on the soft-tissue envelope, or delayed after soft-tissue stabilization with an external fixator. A retrospective study of 19 patients with an extruded talus reported that 12 patients had no subsequent surgery after definitive fixation, 7 had subsequent procedures, and 2 patients developed infections that were treated successfully at an average of 42-month follow-up. Successful outcome in this series was attributed to multiple debridements, soft-tissue stabilization, and primary wound closure.

 

(SAE09TR.98) A 71-year-old woman who reports long-term use of oral steroids for asthma is referred for treatment of a distal humerus fracture. Radiographs reveal diffuse osteopenia and a severely comminuted intra-articular fracture. What is the most appropriate treatment? 

 

  1. Long arm cast immobilization

     

  2. Total elbow arthroplasty

     

  3. Open reduction and internal fixation

     

  4. Osteoarticular allograft

     

  5. Resection arthroplasty

 

PREFERRED RESPONSE 2

 

Several studies have documented the satisfactory outcomes of total elbow arthroplasty when osteosynthesis is not feasible for fixation of a distal humerus fracture, particularly in the physiologically older patient with low functional demands. Total elbow arthroplasty should be considered when a comminuted intra-articular distal humerus fracture occurs in a woman older than age 65 years, particularly with such associated comorbidities as systemic steroid use, osteoporosis, or rheumatoid arthritis.

 

(SAE09FA.34) A 52-year-old woman slipped on ice in her driveway. Radiographs are shown in Figures 19a and 19b. The patient was treated in a short leg cast with weight bearing as tolerated for 6 weeks. Due to persistent tenderness at the fracture site, a CAM walker was used for an additional 8 weeks. Nine months after the injury, the patient still walks with a limp and reports pain with deep palpation at the fracture site. What is the next most appropriate step in management? Review Topic

 

 

 

  1. CT scan

  2. Repeat period of immobilization

     

  3. Referral to pain management for sympathetic blocks

     

  4. Continued observation and physical therapy

     

  5. Acupuncture

 

PREFERRED RESPONSE 1

 

Persistent pain at the fracture site in the absence of infection is most likely due to a nonunion, best detected by CT. Walsh and DiGiovanni reported on a series of closed rotational fibular fractures in which nonunions were detected by CT in the absence of standard ankle radiographic findings. Repeat immobilization would not be appropriate at this late date. Pain management/sympathetic blocks would be considered if the patient displayed pain with light touch and disproportionate pain consistent with a complex mediated pain syndrome. Acupuncture would be expected to be of limited benefit.

 

(SAE11UE.113) A 32-year-old male hockey player who is right-hand dominant was checked from behind and landed with full force into the boards. In the emergency department he reports shortness of breath. Figure 113 shows a 2-D CT scan. What is the best initial treatment for this injury? 

 

 

 

  1. Observation

  2. Closed reduction with a towel clip

     

  3. Open reduction

     

  4. Open reduction and internal fixation

     

  5. Open reduction and sternoclavicular ligament allograft reconstruction

 

PREFERRED RESPONSE 2

 

The CT scan shows a posterior sternoclavicular joint dislocation. Initial management involves attempted closed reduction in the operating room. This can be performed with a towel clip and anterior translation of the displaced clavicle. However, the orthopaedic surgeon should be prepared to open this injury and reconstruct the joint if necessary. Furthermore, it is recommended that a thoracic surgeon be available prior to beginning these procedures. Open reduction should be done only if closed reduction is unsuccessful.

 

(SAE13PE.75) Which muscles are responsible for the displacement of the proximal fragment of the fracture shown in Figure 75? 

 

 

 

  1. Iliopsoas, hip abductors, hip external rotators

  2. Iliopsoas, hip adductors, hip internal rotators

     

  3. Rectus femoris, hip abductors, hip external rotators

     

  4. Hamstrings, hip abductors, hip internal rotators

 

PREFERRED RESPONSE 1

 

The radiograph shows a subtrochanteric femoral shaft fracture in a skeletally immature patient. The proximal fragment is displaced into flexion, abduction, and external rotation. Flexion is attributable to the pull of the iliopsoas at the lesser trochanter. Abduction is attributable to the pull of the abductor muscles (gluteus medius and minimus) at the greater trochanter. External rotation is attributable to the pull of the small external rotators, including the piriformis. The majority of the adductor musculature originates on the symphysis pubis and bypasses the proximal femur, inserting further distally on the adductor tubercle. The hamstrings originate on the ischial tuberosity and also bypass the proximal fragment, inserting distally on the proximal tibia and fibula.

 

(SBQ12TR.39) A 36-year-old male falls from a 10-ft scaffold and suffers the injuries shown in Figures A and B. The patient is placed in a spanning external fixator and brought back to the operating room once his soft tissues are amenable. Planning to use a dual-incision approach, what is the correct interval to use when approaching the medial side? 

 

 

 

  1. Popliteus and pes anserine

  2. Lateral head of the gastrocnemius and pes anserine

     

  3. Politeus and lateral head of the gastrocnemius

     

  4. Iliotibial band and medial head of the gastrocnemius

     

  5. Pes anserine and medial head of the gastrocnemius

 

PREFERRED RESPONSE 5

 

The posteromedial approach to the tibial plateau is between the the pes anserine tendons and the medial head of the gastrocnemius.

 

A dual-incision approach is often utilized to optimally place definitive fixation for bicondylar tibial plateau fractures. For fractures that require posterior or posteromedial fixation, the correct interval is between the pes anserine and the medial head of the gastrocnemius.

 

Higgins et al. in a large cohort morphological review, noted a high incidence of a posteromedial fragment in bicondylar fractures. Occurring at a high frequency, the

authors recommended direct visualization and reduction via a dual approach rather than using indirect reduction techniques.

 

Falker et al. describes a step-by-step approach to utilizing the posteromedial approach for the tibial plateau and placing an anti-glide plate.

 

Figure A and B exhibit a bicondylar tibial plateau fracture with a posteromedial fragment noted on the lateral x-ray. Illustration A exhibits the surrounding anatomy and interval in between the medial head of the gastrocnemius and the pes anserine.

 

Incorrect answers:

Answers 1-4. These intervals are not the correct intervals utilized in a posteromedial approach to the tibial plateau.

 

(SAE12TR.87) Figures 87a and 87b are the radiographs of an 18-year-old pedestrian who was struck by a car. During intramedullary nailing, it is difficult to maintain proper alignment. Poller blocking screws placed in the proximal fragment at which position(s) relative to the nail can help prevent the typical deformity? 

 

 

 

  1. Anterior only

  2. Anterior and medial

     

  3. Anterior and lateral

     

  4. Posterior and medial

     

  5. Posterior and lateral

PREFERRED RESPONSE 5

 

This is a proximal one third tibial shaft fracture. Typically nailing of this fracture creates a valgus and procurvatum malalignment that must be addressed. This can be difficult when using an intramedullary nail in the wide metaphyseal bone of the proximal tibia. To help direct and center the nail in the metaphysis, blocking screws can be used. Blocking screws should be placed where the nail should not travel. If the nail was passed with the proximal fragment in this position, it would occupy the lateral and posterior aspects of the metaphyseal fragment. To prevent this, blocking screws should be placed in the lateral and posterior aspects of the proximal fragment.

 

(SAE12TR.51) A 31-year-old man sustained an unstable closed left posterior hip dislocation in a motorcycle accident. A postreduction radiograph is shown in Figure 51a. 3-D CT scans are shown in Figures 51b and 51c. What is the optimal surgical approach that will allow for the most appropriate treatment? 

 

 

 

  1. Surgical hip dislocation (“Berne” approach)

  2. Watson-Jones approach

     

  3. Smith-Peterson approach

  4. Kocher-Langenbeck approach

     

  5. Extensile iliofemoral approach

 

PREFERRED RESPONSE 1

 

The radiograph and CT scans show a posterior wall acetabular fracture with an associated femoral head fracture. As is the case in most of these injuries, the femoral head fracture is located on the anterior aspect of the femoral head. Surgical dislocation with a trochanteric flip osteotomy as described by Solberg and associates and Henle and associates allows for exposure and treatment of the posterior wall fracture as well as surgical dislocation for treatment of the femoral head fracture. A Smith-Peterson approach or Watson-Jones approach would allow for anterior exposure and may help to address the femoral head fracture, but not the posterior wall fracture. A Kocher-Langenbach approach would allow exposure of the posterior wall fracture, but not the femoral head fracture. An extensile iliofemoral approach is unnecessary for this injury pattern.

 

(SAE12TR.52) What is the most common complication following surgery for a "terrible triad" elbow fracture-dislocation? 

 

  1. Arthritis

     

  2. Infection

     

  3. Re-dislocation

     

  4. Restricted range of elbow motion

     

  5. Posterior interosseous nerve (PIN) palsy

 

PREFERRED RESPONSE 4

 

Recurrent instability, PIN palsy, infection, and posttraumatic arthritis have all been reported following these injuries; however, elbow contracture or loss of motion is nearly universal following these injuries.

 

(SAE09TR.79) An 86-year-old woman sustained a fracture of the humerus and underwent surgical fixation 8 weeks ago. There was no radial nerve function below the elbow after surgery. Radiographs are shown in Figures 51a and 51b. What is the most appropriate management at this time? 

 

 

 

  1. Nerve conduction velocity studies and electromyography

  2. Exploration and grafting of the radial nerve

     

  3. Tendon transfers

     

  4. Observation for another 2 months

     

  5. Removal of the plate, neurolysis of the radial nerve, and intramedullary rodding of the humerus

 

PREFERRED RESPONSE 4

 

Most radial nerve palsies associated with closed fractures of the humerus resolve spontaneously, including Holstein-Lewis lesions (radial nerve palsy associated with oblique distal third fractures of the humerus). Initial sign of recovery at the brachioradialis may not occur for 4 months. There has been no evidence of deleterious effects occurring during this observation period. There are advocates of early exploration of the nerve. Exploration in the intermediate period between 1 and 4 months is not supported. As overall alignment of the fracture is acceptable, there is no need for hardware exchange until nonunion is clearly identified.

 

(SAE12TR.70) Pelvic packing for a hemodynamically unstable patient with a pelvic ring fracture is best described by which of the following techniques? 

 

  1. Placing a pelvic external fixator followed by packing the pelvis with lap pads via a subumbilical incision

     

  2. Placing lap pads for packing via a subumbilical incision in the angiography suite

     

  3. Placing lap pads for packing using the lateral window of the ilioinguinal approach (anterior approach to the internal iliac fossa)

  4. Packing the retroperitoneum with lap pads after exploration of the abdomen by the general surgeons

     

  5. Direct exploration of the pelvic vasculature via a midline incision followed by packing with lap pads

 

PREFERRED RESPONSE 1

 

For the technique of pelvic packing patients are placed supine on an operating room table. For rotationally and/or vertically unstable fracture patterns, an external fixator is then placed to stabilize the pelvis so that the volume of the pelvis is decreased and the packing has counterforce acting against it. An approximately 6 cm to 8 cm midline incision is made extending upwards from the pubic symphysis and heading toward the umbilicus. The rectus fascia is then divided in the midline. The bladder is retracted to one side and three lap pads are packed deep to the pelvic brim. The bladder is retracted to the other side and three more lap pads are placed on that side as well. The first sponge is placed at the level of the sacroiliac joint, the second anterior to the first sponge, and the third in the retropubic space lateral and just deep to the bladder. All should be placed below the level of the pelvic brim. The fascia is then closed. If the patient is hemodynamically unstable after stabilization, then packing of the pelvis angiography should be considered.

 

(SAE08UE.66) A 40-year-old unrestrained passenger reports chest wall pain after a motor vehicle accident. Which of the following structures is most important in preventing the injury shown in Figure 33? 

 

 

  1. First rib

  2. Intra-articular disk ligament

     

  3. Costoclavicular ligament

     

  4. Interclavicular ligament

     

  5. Posterior sternoclavicular joint capsule

 

PREFERRED RESPONSE 5

 

Through cadaveric study, Spencer and associates measured anterior and posterior translation of the sternoclavicular joint. The study demonstrated that the posterior sternoclavicular joint capsule is the most important structure for preventing both anterior and posterior translation of the sternoclavicular joint.

 

(SAE08OS.119) Plate fixation of olecranon fractures is recommended over tension band wire fixation when 

 

  1. the fracture is proximal and only involves the olecranon tip.

     

  2. the fracture is transverse.

     

  3. the fracture is comminuted.

     

  4. the fracture is widely displaced.

     

  5. there is associated osteoporosis.

PREFERRED RESPONSE 3

 

Tension band wire fixation of olecranon fractures is recommended for fracture patterns that are proximal to the coronoid process and are relatively transverse to withstand compressive forces. When comminution is present, a neutralization technique such as plating is preferred over a compressive technique such as tension band wire fixation. Such neutralization plating, if performed correctly, does not have the risk of narrowing the sigmoid notch as tension band wire fixation would. Fractures of the tip of the olecranon, transverse fractures, fractures associated with osteoporosis, and displaced fractures are all relative indications for tension band wire fixation.

 

(SAE11OS.10) A 6-month-old child has the deformity seen in Figure 10. There are no other known associated problems. What is the etiology of this condition? Review Topic

 

 

 

  1. Exposure to teratogens

  2. Multifactorily inherited

     

  3. A defect of the apical ectodermal ridge

     

  4. Excess production of fibroblast growth factor

     

  5. Inherited as an autosomal dominant

 

PREFERRED RESPONSE 3

The radiograph demonstrates a type IV radial clubhand (radial dysplasia) with complete absence of the radius. This is a pre-axial deficiency usually with complete absence of the thumb. The condition is thought to be caused by an injury to the formation of the apical ectodermal ridge early in embryology. It is not an inherited condition unless it is associated with other syndromic problems. It is not known to be associated with specific teratogens. Fibroblast growth factor is involved in angiogenesis, wound healing, and embryonic development, but is not known to be associated with radial clubhand.

 

(SAE12FA.23) Which of the following factors has been shown to increase the risk of peroneal tendon pathology in patients who have undergone posterior plating of lateral malleolar fractures? 

 

  1. Use of cut or trimmed plates

     

  2. Use of straight (uncontoured) plates

     

  3. Use of locked plating

     

  4. Low plate placement with a prominent screw head in the distal hole

     

  5. Low antiglide plate placement

 

PREFERRED RESPONSE 4

 

Low plate positioning with a prominent screw head in the most distal hole of the plate was shown to be correlated with peroneal tendon lesions. Distal plate placement in the absence of prominent screws was not associated with tendon lesions. Trimmed plates, locked plates, and uncontoured plates have not been shown to increase the risk of peroneal tendon pathology.

 

(OBQ15.128) A 41-year-old male underwent intramedullary nailing for a low-energy left femoral shaft fracture. At his follow-up appointment, he complains that his feet are pointing in opposite directions when walking. Using the imaging study shown in

Figure A, which of the following represents this patient's left femur malalignment? 

 

 

 

  1. Internal rotation malalignment of 44 degrees

  2. External rotation malalignment of 44 degrees

     

  3. Internal rotation malalignment of 21 degrees

     

  4. External rotation malalignment of 21 degrees

     

  5. Internal rotation malalignment of 63 degrees

 

PREFERRED RESPONSE 3

 

Figure A shows axial CT scan slices of the pelvis and knee. On the operative left side, there is an internal rotation malalignment of 21° compared to the contralateral side (44°-23°=21°).

 

Radiographic rotational malalignment after fixation of femoral shaft fractures may be measured by comparing the femoral anteversion of both femurs. This can be determined by measuring the angle between a line tangential to the dorsal bony contours of the femoral condyles and a line drawn through the axis of the femoral neck. Rotational differences of less than 10° are considered variations of normal.

 

Jaarsma et al. reviewed rotational malalignment after intramedullary nailing of femoral fractures. They report that rotational measurements by CT are superior to clinical assessment. They note a high incidence of malrotation after IM nailing of fractures. This has shown to be in the range 15% to 30%.

Figure A shows left femoral malrotation using CT-torsion measurements with axial cuts of the femoral neck and distal femoral condyles. Note the normal anteversion of the right femur (23 degrees; normal range 10-25).

 

Incorrect Answers:

Answer 1,2,4,5: To measure malrotation using axial CT cuts, the normal side is considered as neutral = 0 malrotation. For this patient, that correlates with 23 degrees of anteversion. If the femoral anteversion is increased to 44 degrees, this will mean a 21 degree increase in femoral anteversion from neutral (0 + [44-23] = +21). The opposite would occur if the femoral anteversion decreased = external rotation.

 

(SAE12FA.91) Figures 91a through 91c are the radiographs of a 10-year-old boy who has a 6-month history of progressive heel pain. The patient is a year-round soccer player and now experiences pain with most every step. What is the most appropriate management? 

 

 

 

  1. MRI

  2. Custom orthotics

     

  3. Activity modification

  4. Calcaneal epiphysiodesis

     

  5. Percutaneous Achilles tendon lengthening

 

PREFERRED RESPONSE 3

 

The patient has calcaneal apophysitis, an overuse syndrome common in children ages 9 to 12 years. Symptoms are usually the result of excess tension and a tight heel cord. Management includes activity modification, as well as heel cord stretching, nonsteroidal anti-inflammatory drugs, icing, and other modalities. Radiographs are typically negative; MRI is unnecessary. Custom orthotics are not indicated. The condition is self-limiting, in that the symptoms fully resolve once the apophyses fuses, such that surgery is rarely indicated.

 

(OBQ12.214) A 25 year-old-male presents with the injury seen in Figure A. Which of the following would be a contraindication to closed management with a functional brace? 

 

 

 

  1. Radial nerve injury

  2. 1 cm shortening

     

  3. 20 degree varus deformity

     

  4. Brachial plexus injury

  5. Comminuted fracture pattern

 

PREFERRED RESPONSE 4

 

Closed treatment of humeral shaft fractures with functional bracing is indicated in the vast majority of isolated injuries. An ipsilateral brachial plexus injury, however, is a contraindication to nonoperative management in a functional brace.

 

Indications for operative management of humeral shaft fractures are limited given the high rates of union and ability of adjacent joints to compensate for deformity. Intact muscular tone is necessary to effect bony apposition in closed treatment with a functional brace. The absence of neurologic and muscle function in patients with a flail extremity leads to increased rates of nonunion and malunion.

 

Rutgers and Ring conducted a retrospective review of patients managed with functional bracing of humeral shaft fractures at a single institution. The authors found a 90% overall union rate, with maintenance of shoulder and elbow motion. They caution though, that 29% of their proximal third fractures went on to nonunion.

 

Figure A demonstrates an AP radiograph of a comminuted humeral shaft fracture with varus alignment.

 

Incorrect Answers:

Answer 1: Radial nerve injury is not an indication for operative management as the vast majority of radial nerve injuries recover with conservative management Answer 2: 1cm of shortening is an acceptable deformity with closed management Answer 3: 20 degree varus deformity is not an indication for operative management Answer 5: Fracture comminution is not a contraindication to functional bracing

 

(SAE09FA.87) A 51-year-old man sustained an open fracture of his tibia in Korea 42 years ago. An infection developed and it was resolved with surgical treatment. For the past 6 months, an ulcer with mild drainage has developed over the medial tibia. The ulcer is small and there is minimal erythema at the ulcer site. A radiograph and MRI scan are shown in Figures 43a and Figure 43b. Initial cultures show Staphylococcus aureus susceptible to the most appropriate antibiotics. Laboratory studies show an erythrocyte sedimentation rate of 70 mm/h. What is the most appropriate surgical treatment at this time? Review Topic

 

 

  1. Irrigation and debridement of the cystic lesion and 6 weeks of IV antibiotics

  2. Curettage, debridement of nonviable bone, and placement of absorbable antibiotic beads, followed by a course of IV antibiotics from 1 to 4 weeks and a 6-week course of oral antibiotics

     

  3. Complete resection of the infected portion of bone, placement of an external fixator to stabilize the tibia, and 6 weeks of IV antibiotics

     

  4. Amputation

     

  5. Local debridement of bone and the overlying skin and soft tissues, 6 weeks of IV antibiotics, and free-flap wound coverage

 

PREFERRED RESPONSE 2

 

The patient has chronic tibial osteomyelitis that is due to low virulent bacteria. The history and studies do not suggest the need for an amputation or a free-flap procedure. This is a localized tibial infection that is in a healed bone; there is no need to resect the entire area of the tibia bone around the infection. The most appropriate treatment is curettage, debridement of nonviable bone, and placement of absorbable antibiotic beads, followed by a course of IV antibiotics from 1 to 4 weeks and a 6-week course of oral antibiotics. Studies have shown that in cases of localized osteomyelitis that are of low virulence, as little as 1 week of IV antibiotics followed by 6 weeks of oral antibiotics is successful.

(SAE08OS.141) The implant shown in Figures 47a and 47b is introduced submuscularly employing a minimally invasive technique. A percutaneous method of screw insertion is used distally. What nerve is most at risk? 

 

 

 

  1. Sural

  2. Saphenous

     

  3. Superficial peroneal

     

  4. Deep peroneal

     

  5. Posterior tibial

 

PREFERRED RESPONSE 3

 

Minimally invasive methods used for stabilizing complex periarticular fractures continue to evolve. Encouraging results suggest a diminished threat to the soft tissues and enhanced preservation of osseous blood supply. Contemporary locking implants combined with indirect reduction lead to desirable biomechanical and biologic environments for osseous and soft-tissue healing. Deangelis and associates, in a cadaveric tibial study, demonstrated the superficial peroneal nerve to be at significant risk during percutaneous screw placement in very distal targeted holes (within laterally applied tibial locking plates). Use of a larger incision and cautious dissection to the plate in this region were encouraged to minimize risk to this structure.

 

(SAE08AN.99) Following a fall from a height of 5 feet, a patient reports pain along the lateral border of the foot. The CT scan shown in Figure 54 indicates what pathology? 

 

 

 

  1. Impaction injury of the cuboid

  2. Retracted os peroneum

     

  3. Fifth metatarsal avulsion fracture

     

  4. Avulsion injury of the bifurcate (Y) ligament

     

  5. Lisfranc injury

 

PREFERRED RESPONSE 4

 

The CT scan reveals an avulsion of the dorsal beak of the anterior process of the calcaneus. This common fracture is an avulsion of the origin of the bifurcate ligament, which runs from the anterior calcaneal process to both the cuboid and the lateral aspect of the navicular. An inversion mechanism is common, and the fracture is often missed in evaluation for a suspected ankle sprain. MRI may be useful in the diagnosis of these occult injuries, and suspicion should be present when tenderness exists over the superior portion of the anterior process of the calcaneus.

(SAE12TR.80) A 38-year-old woman is polytraumatized in a motor vehicle crash. She has multiple injuries including a unilateral femur fracture. The patient is felt to be borderline and, although she is currently stable, she could potentially deteriorate quickly. Which of the following parameters has been suggested as an indicator of which patients would benefit from damage control? 

 

  1. Normothermia

     

  2. Hemoglobin of less than 9 g/dL

     

  3. Unilateral lung contusion evident on CT only

     

  4. Injury severity score of greater than 40 without thoracic injury

     

  5. Injury Severity Score of less than 18 with a pulmonary contusion

 

PREFERRED RESPONSE 4

 

Polytraumatized patients can be classified as stable, unstable, borderline, or in extremis. Management of the borderline patient is controversial because it is unclear which patients can safely undergo early definitive surgical stabilization of fractures, and which patients would benefit from temporizing "damage control" stabilization to allow adequate resuscitation and physiologic stabilization prior to definitive treatment. Although the question of damage control versus early total care is unresolved, there are several clinical parameters that have been suggested for use in deciding who should be treated with early damage control. These include Injury Severity Score of greater than 40, Injury Severity Score of greater than 20 with thoracic trauma, multiple injuries with severe pelvic/abdominal trauma and hemorrhagic shock, bilateral femoral fractures, pulmonary contusion noted on radiographs, hypothermia of less than 35 degrees C), and a head injury with an Abbreviated Injury Score of 3 or greater. A hemoglobin of 9 g/dL is not included in these suggested parameters.

 

(OBQ14.238) Total hip arthroplasty is most appropriate for the injury shown in Figure A for which of the following patients? 

 

 

  1. 66-year-old female golf instructor

  2. 66-year-old female with Parkinsons dementia

     

  3. 87-year-old male household ambulator

     

  4. 50-year-old male alcoholic with hepatic encephalopathy

     

  5. 24-year-old male laborer

 

PREFERRED RESPONSE 1

 

Figure A is an AP radiograph demonstrating a displaced femoral neck fracture. Active older patients who present with a displaced femoral neck fracture should be treated with total hip arthroplasty (THA).

 

Displaced femoral neck fractures can present a challenge to treat. In younger patients with good bone stock a closed vs. open reduction and internal fixation should be attempted. For active older patients a total hip arthroplasty is the best option, especially if there is pre-existing arthritis in the injured hip. THA provides the best function with the least pain and less need for repeat surgery (compared to hemiarthroplasty). For low-demand or debilitated patients, for patients older than age 80, or for those who can not reliably follow hip precautions a hemiarthroplasty provides the lowest risk of dislocation, and thus would be the treatment of choice.

 

Macaulay et al. present a prospective randomized trial of patients with femoral neck fractures treated with THA vs hemiarthroplasty. They found that functional outcomes and patient satisfaction were higher in the THA group without significant increased risk of complications. Inclusion criteria required patients to be over age 50, be a community ambulator, and were excluded for presence of dementia.

 

Abboud et al. retrospectively reviewed patients treated with THA for osteoarthritis and compared them to patients treated with THA for a femoral neck fracture. They found no significant difference between the two groups for outcomes or complications.

 

Figure A is an AP radiograph demonstrating a displaced femoral neck fracture.

Incorrect Answers:

Answer 2: Parkinsons dementia presents an increased risk for dislocation and would make a hemiarthroplasty a more suitable choice. Answer 3: According to AAOS guidelines, age greater than 80 and low demand status is an indication for hemiarthroplasty Answer 4: Hepatic encephalopathy would raise concern for the ability to follow hip precautions and presents an increased risk for dislocation. Thus a hemiarthroplasty would be a better choice. Answer 5: For a young male laborer a closed vs open reduction and internal fixation should be attempted rather than an arthroplasty.

 

(OBQ14.190) An 18-year-old patient sustains a comminuted left femoral fracture starting 6.5cm distal to the lesser trochanter. He undergoes antegrade femoral nailing in the supine position on a radiolucent table. Upon completion of proximal and distal interlocking, both patellae are positioned facing the ceiling and a lateral radiographs confirms that the posterior condyles of both limbs are aligned. On AP imaging of both femora, it is noted that the lesser trochanter of the left (injured) side is larger than the right (uninjured) side. Assuming symmetrical anteversion, the left femur has been nailed Review Topic

 

  1. with varus malalignment

     

  2. with valgus malalignment

     

  3. with external rotation malalignment

     

  4. with internal rotation malalignment

     

  5. with no malalignment

 

PREFERRED RESPONSE 4

 

When the lesser trochanter (LT) profile is larger than the uninjured side, the proximal fragment is externally rotated. This leads to an overall internal rotation (IR) malalignment of the distal fragment. Malalignment is described based on the distal fragment relative to the proximal fragment. For more proximal femoral fractures, the proximal fragment tends to be flexed and externally rotated due to the iliopsoas. Matching rotation requires external rotation of the distal fragment when the patient is supine on a fracture table.

Rotational malalignment is the most common complication of intramedullary nailing of a comminuted diaphyseal femoral fracture. The rotational profile of the lesser trochanter can be used to evaluate rotational alignment. The proximal femur is rotated until a neutral position is obtained as judged by the radiographic profile of the lesser trochanter. If the AP image shows a smaller lesser trochanter, there is IR of the LT. A larger LT indicates external rotation (ER) of the LT.

 

Jaarsma et al. describe CT imaging in determining rotational alignment. They note that the incidence of post-nailing malalignment > 10 ° is 40%, > 15 ° is 20-30%, and

> 20 ° is 16%. They also note that patients with ER deformities have more symptoms than those with IR deformities, and that small deformities <15 ° give rise to less complaints. This is because ER deformities lead to compensation with hip retroversion, which causes more symptoms than hip anteversion when compensating for IR deformities.

 

Incorrect Answers:

Answers 1 and 2: Difference in LT size between sides is not indicative of coronal plane malalignment.

Answer 3: ER malalignment of the distal fragment implies relative IR of the proximal fragment. IR of the LT results in a smaller-appearing LT compared with the other side.

 

(SBQ12TR.37) A 44-year-old male presents with the isolated injury seen in Figure A after a motor vehicle accident and underwent the operative treatment seen in Figure B within 8 hours from the time of incident. Which of the following complications is this patient at highest risk of developing?

 

 

 

 

 

  1. Pulmonary embolus

  2. Periprosthetic fracture

     

  3. Contralateral hip fracture

  4. Osteonecrosis

     

  5. Infection

 

PREFERRED RESPONSE 4

 

This young male patient has sustained a displaced femoral neck fracture and underwent open reduction internal fixation with 3 cannulated screws. Based on the available options, the patient is most at risk of developing osteonecrosis of the femoral head.

 

Femoral neck fractures in young patients typically are the result of a high-energy trauma. Fracture displacement has been shown to disrupt vascular supply to the femoral head by interrupting retinacular vessels and ligament teres vascularization, as well as increasing intracapsular pressure, producing a tamponade effect. The incidence of osteonecrosis in patients younger than 60 years with displaced femoral neck fractures has been shown to be between 15-30%. Quality of reduction is one key factor that has been shown to influence outcomes postoperatively.

 

Loizou et al. prospectively studied 1,023 patients who sustained an intracapsular hip fracture that was treated with internal fixation using standard fixation modalities. They showed that osteonecrosis was less common for undisplaced (4.0%) than for displaced fractures (9.5%). The population at greatest risk were women younger than the age of 60 with displaced fractures.

 

Barnes et al. review subcapital hip fractures. They found that late segmental collapse was more common in displaced fractures in women younger than age 75 years than in those older than age 75 years treated with internal fixation.

 

Figure A shows a displaced, Garden 3/Pauwels I hip fracture. Figure B shows anatomical fixation with 3 cannulated screws.

 

Incorrect Answers:

Answer 1: The incidence of pulmonary embolus after hip fracture is <1%. Answer 2: The incidence of periprosthetic fracture is <5%. Answer 3: The incidence of contralateral hip fracture is minimal in this patient. Answer 5: The rate of infection after hip fracture is <5%.

(SAE09FA.66) A 24-year-old woman was struck by a mini van in a parking lot and sustained a closed segmental tibia fracture that was treated with an intramedullary nail the following morning. Follow-up examinations reveal a slowly progressive clawing of all five toes, a progressive equinocavovarus contracture, and the patient is unable to perform a single heel rise on the affected limb. At 1 year after surgery, the patient now has a 10-degree equinus contracture that is not relieved with knee flexion. Treatment should now consist of 

 

  1. physical therapy and bracing.

     

  2. reassurance that these problems will resolve with time.

     

  3. posterior capsule release, Achilles tendon lengthening, and excision of the scarred muscle and tendon in the leg and foot.

     

  4. Achilles tendon lengthening, and flexor digitorum longus and flexor hallucis longus tenotomies at the individual digits with transfer of the posterior tibial tendon to the dorsum of the foot.

     

  5. flexor digitorum longus and flexor hallucis longus tenotomies at the individual digits with midfoot capsular release and hallux interphalangeal fusion.

 

PREFERRED RESPONSE 3

 

This is an example of a missed deep posterior compartment syndrome that typically presents 6 months after the injury with progressive clawing due to necrosis, scarring, and contracture of the posterior tibial tendon, flexor digitorum longus, and flexor hallucis longus. Treatment consists of debridement of necrotic muscle and scar tissue with corresponding tendon excision. After debridement and posterior capsule release, if the equinus is relieved with knee flexion, a gastrocnemius slide may be performed. Otherwise, the lengthening should be at the level of the Achilles tendon. Bracing will not address the claw toes.

 

(SAE08OS.189) A 73-year-old man sustains the fracture shown in Figure 62. Which of the following factors or combination of factors puts this patient at highest risk for nonunion if nonsurgical management is used? 

 

 

  1. Advanced age and shortening

  2. Fracture displacement and rotation

     

  3. Mid diaphyseal fracture

     

  4. Male gender

     

  5. Fracture comminution, fracture displacement, and advanced age

 

PREFERRED RESPONSE 5

 

Most textbooks and early publications list the incidence of complications of nonsurgical treatment of clavicle fractures as very low. However, recent studies on this topic have found an entirely different picture. The studies show that patients reported shoulder weakness and fatigability, upper extremity dysesthesia, and shoulder asymmetry with an incidence of 31%. Indications for surgery in the past have included open fractures, associated neurovascular injury, and widely displaced fractures tenting the skin. Fractures with more than 2 cm of shortening and comminuted fractures with significant displacement have been associated with poor outcomes. Nonunion after nonsurgical management was found to be more common in the study by Robinson and associates in displaced comminuted fractures, in patients with advanced age and female gender.

 

(SAE08OS.130) The essential lesion in recurrent or posterior instability following simple dislocation of the elbow typically involves which of the following structures? 

 

  1. Medial collateral ligament

  2. Lateral ulnar collateral ligament

     

  3. Coracohumeral ligament

     

  4. Anterior joint capsule

     

  5. Posterior joint capsule

 

PREFERRED RESPONSE 2

 

The lateral ulnar collateral ligament is the essential lesion in recurrent or persistent instability following simple dislocations of the elbow. Simple elbow dislocations are usually stable and may be managed by a short period of immobilization followed by early range of motion. Treatment of dislocations resulting in persistent instability frequently involves focusing on the lateral ulnar collateral ligament. The medial collateral ligament is repaired only if treatment of associated fractures and lateral collateral ligament injury does not restore stability.

 

(OBQ15.132) A fracture in the following location is most commonly associated with procurvatum and valgus malalignment? 

 

  1. Humeral shaft

     

  2. Distal femur

     

  3. Proximal tibia

     

  4. Distal third tibia shaft with intact fibula

     

  5. Subtrochanteric proximal femur

 

PREFERRED RESPONSE 3

 

Fractures of the proximal tibial shaft are associated with high rates of valgus and apex anterior (procurvatum) malalignment.

Proximal third tibial shaft fractures are often difficult to reduce anatomically due to the tendency for both valgus and flexion deformity at the fracture site. Many different techniques have been devised to overcome the deforming forces. These include (1) Poller blocking screws posterior and lateral to the intramedullary nail (IMN), (2) utilizing a semiextended knee position during IMN of proximal tibia fractures (3) use of a suprapatellar approach for IMN, (4) usage of a slightly more lateral starting point during conventional IMN, and (5) application of unicortical plate.

 

Ricci et al. describe the technique and results of using blocking screws and intramedullary nails to treat patients with fractures of the proximal third of the tibia. Post-operatively, all patients in their series had less than 5 degrees of angular deformity in the planes in which blocking screws were used to control alignment. At 6 months follow-up, 10/11 patients maintained this alignment.

 

Illustration A shows intra-operative sagittal radiographs of the proximal tibia. Note the use of Poller blocking screws in the posterior and lateral aspects of the proximal tibia.

 

Incorrect Answers:

Answer 1: The most common malalignment with a humeral shaft fractures is varus, shortening and internal rotation. Answer 2: The most common malalignment with a distal femur fracture is varus, shortening and recurvatum. Answer 4: The most common malalignment with a distal third tibia shaft + intact fibula is varus and internal/external rotation. Answer 5: The most common malalignment with subtrochanteric proximal femur fracture is flexion, abduction and external rotation.

 

(SAE08OS.112) An otherwise healthy 25-year-old man with an isolated closed mid-diaphyseal femoral fracture undergoes intramedullary nailing. Compared with nonreamed nailing, reamed femoral nailing is associated with a higher rate of Review Topic

 

  1. union.

     

  2. symptomatic pulmonary complications.

     

  3. infection.

     

  4. transfusion requirements.

     

  5. required secondary procedures.

PREFERRED RESPONSE 1

 

Bhandari and associates, in a meta-analysis, concluded that sufficient evidence exists to suggest that reamed intramedullary nailing of lower extremity long bone fractures significantly reduces rates of nonunion and implant failure in comparison with nonreamed nailing. Tornetta and Tiburzi, in a prospective randomized study, determined that reamed canal preparation led to faster healing of distal fractures treated with statically locked intramedullary nails. Blood loss was greater in the reamed group, but this did not translate into increased transfusion requirements. In this series, there was no advantage to nail insertion without reaming. In a prospective randomized multicenter study, the overall incidence of acute respiratory distress syndrome (ARDS) was found to be low with primary stabilization of femoral shaft fractures with intramedullary nailing. There was no difference in the incidence of ARDS between the reamed and unreamed groups. In a retrospective study performed by Handolin and associates, intramedullary nailing of long bone fractures in patients with multiple injuries and with a coexisting pulmonary contusion did not impair pulmonary function or outcome. No study has convincingly demonstrated an increased trend toward infection with reamed femoral intramedullary nailing.

 

(SAE12TR.25) Figures 25a through 25c show the radiographs, including a stress radiograph, of a 58-year-old woman who twisted her ankle on a step. She has no history of diabetes or vascular disease. Examination reveals a closed injury with moderate swelling about the ankle. Her neurologic examination is normal. She has a strong dorsalis pedis pulse and tenderness over the lateral malleolus and the medial side of her ankle. What is the most appropriate management? Review Topic

 

 

 

  1. MRI scan of the ankle

  2. Non-weight-bearing cast for 6 weeks

     

  3. Removable walking boot and progressive weight bearing

     

  4. Open reduction and internal fixation of the fibula

     

  5. Open reduction and internal fixation of the fibula with medial ligament repair

 

PREFERRED RESPONSE 3

 

The patient has a lateral malleolus fracture with an ankle mortise that is stable to a stress examination; therefore, surgical treatment is not indicated. In a stable lateral malleolus fracture, strict non-weight-bearing is not necessary, and a removable walking boot or walking cast can be used along with progressive weight bearing. The presence of tenderness or swelling medially at the ankle has been shown to be a poor indicator of medial-sided injury. The clinical utility of MRI scans in ankle fractures is controversial. Studies have used MRI scans to evaluate the competence of the deltoid ligament and have shown that the ligament may remain intact even with an increased medial clear space on a stress examination. In the patient, the stress examination does not show talar subluxation so the deltoid ligament is not incompetent.

 

(SAE09FA.3) A 19-year-old man was struck by a car and is seen in the emergency department with a grade IIIC open distal tibia and fibula fracture. Examination reveals that the toes are cool and dusky with a sluggish capillary refill. Angiography reveals a lesion in the posterior tibial artery amenable to repair. There is no sensation on the plantar aspect of the foot, and he is unable to flex his toes. A clinical photograph and radiograph are shown in Figures 2a and 2b. What is the next most appropriate step in management? 

 

 

  1. Irrigation and debridement with immediate intramedullary fixation, vascular repair, and primary closure

  2. Irrigation and debridement with external fixation, vascular repair, and delayed closure

     

  3. Irrigation and debridement with external fixation, vascular repair, exploration of the tibial nerve, and delayed closure

     

  4. Guillotine amputation at the fracture site with delayed closure

     

  5. Immediate below-knee amputation

 

PREFERRED RESPONSE 2

 

In the past, loss of plantar sensation in this grade IIIC tibial fracture would have been an indication for below-knee amputation regardless of the potential for vascular repair. However the 2002 LEAP study divided 55 patients with loss of plantar sensation into two groups, the insensate amputation group and the insensate limb salvage group, with 55% of patients in the insensate salvage group regaining normal sensation 2 years after injury. Furthermore, those in the salvage group who remained insensate after 2 years had equivalent outcomes to those in the amputation group. Because of these findings, limb salvage with vascular repair and external stabilization with delayed closure is deemed appropriate treatment. Immediate intramedullary fixation is not indicated. Because ischemia, contusion, and stretch can adversely affect the tibial nerve, the additional insult of exploration of the nerve is also not advisable given the soft-tissue compromise.

(SAE11OS.159) Figures 1 and 2 are the radiographs of a 40-year-old woman who sustained a twisting injury to her lower extremity. What additional information or studies are important in determining treatment options? 

 

 

 

  1. Full-length tibia-fibula radiographs

  2. Inability to bear weight

     

  3. History of recurrent ankle sprains

     

  4. Presence or absence of medial tenderness

     

  5. MRI scan

 

PREFERRED RESPONSE 1

 

The radiographs reveal a medial ankle injury with a widened medial clear space. No fibula fracture is visualized on this view; therefore, full-length radiographs looking for a proximal fibula fracture are required to determine treatment. The presence or absence of medial tenderness has been shown to not be a good predictor of unstable injuries. A history of previous injuries or ankle instability is typically lateral instability, which would not present with this radiographic appearance. An MRI scan can be used to evaluate subtle syndesmotic injuries, but there is a clear widening of the medial clear space in this case. The inability to bear weight is not helpful in determining the treatment options.

(SBQ12TR.24) In each of the following scenarios, atrophic fracture nonunion occurred after initial treatment with intramedullary nail fixation. Which scenario has shown to have the highest rate of osseous union if treated with exchange intramedullary nailing? 

 

  1. Oligotrophic nonunion of a comminuted humeral shaft fracture

     

  2. Oligotrophic nonunion of a transverse humeral shaft fracture

     

  3. Oligotrophic nonunion of an oblique distal femur fracture

     

  4. Oligotrophic nonunion of a comminuted tibial shaft fracture

     

  5. Oligotrophic nonunion of an oblique tibial shaft fracture

 

PREFERRED RESPONSE 5

 

Reamed exchange nailing is recommended for the management of aseptic nonunions of noncomminuted tibial shaft fractures. Union rates have been reported between 76-96% in large studies.

 

Tibial exchange nailing promotes osseous bone healing of non-unions by providing biological and mechanical support. The biological support is provided by reaming the medullary canal. This increases periosteal blood flow and stimulates periosteal new-bone formation. The mechanical support is provided by a larger-diameter intramedullary nail, which increases the rigidity and strength of the nail.

 

Brinker et al. reviewed the concept of exchange nailing of nonunited long bone fractures. They showed that exchange nailing is the most successful in the treatment of nonunions following closed or open fractures without substantial bone loss. Aseptic, noncomminuted diaphyseal femoral and tibial shaft fractures showed the highest rates of union with exchange nailing, which were found to be 76-100% and 72-96%, respectively.

 

Illustration A shows a heterotrophic non-union of the tibia after intramedullary nailing. The patient was treated with exchange nailing with a larger nail. On the right shows a 4 month post-op radiograph after exchange nailing showing osseous union at the fracture site.

 

Incorrect Answers:

Answer 1, 2: Exchange nailing has not produced good results following failures of intramedullary nail fixation of humeral shaft fractures. Union rates have been shown between 30-40%,

Answer 3: There is little evidence to support exchange nailing for distal femoral nonunions. Union rates have been shown at 50-60%. Answer 4: Nonunions following comminuted fractures do not appear to respond to exchange nailing. Union rates have been reported as 45-80%.

 

(SBQ12TR.79) A right-hand dominant female sustains a right proximal humerus fracture. The patient is provided a sling, and is recommended pendulum exercises with elbow range of motion to begin in 1 to 2 weeks. Which of the following would be an indication for surgical management? 

 

 

 

  1. Age greater than 70 years.

  2. Fracture pattern in Figure A

     

  3. Significant medical comorbidities.

     

  4. Fracture pattern in Figure B

     

  5. Fracture pattern in Figure C

 

PREFERRED RESPONSE 4

 

The patient has been treated with non-operative management for her proximal humerus fracture. Operative management should be considered in patients with head splitting proximal humerus fractures and in those with dislocations that cannot be reduced.

 

Head splitting proximal humerus fractures should be treated with operative management. Open reduction internal fixation versus hemiarthroplasty are used to treat this type of fracture. Surgical management is also considered in proximal humerus fractures in young patients, in fractures where the greater tuberosity is

displaced >5 mm, and in proximal humerus fractures associated with humeral shaft fractures.

 

Koval et al. studied 104 patients with one-part proximal humerus fractures treated non-operatively, and found 80% with good or excellent results. They also found that 90% of patients treated non-operatively had either no or mild pain about the shoulder at follow-up.

 

Lefevre-Colau et al. performed a randomized prospective study on 74 patients with an impacted proximal humerus fracture. One group was treated with early mobilization of the shoulder (within 3 days after the fracture) while the other group was immobilized for 3 weeks followed by physiotherapy. They concluded that early mobilization was safe and allowed for quicker return to functional use of the affected limb.

 

Figure A shows an AP radiograph of a right minimally displaced greater tuberosity proximal humerus fracture. Figure B shows AP and axillary radiographs of a right head split proximal humerus fracture that is posteriorly dislocated. Figure C shows an AP radiograph of a right minimally displaced Salter Harris II proximal humerus fracture. Illustration A shows an AP radiograph of a left valgus impacted proximal humerus fracture with a greater tuberosity fragment displaced >5mm treated with ORIF.

 

Incorrect Answers:

Answers 1, 2, 3, and 5 are all factors that would favor non-operative management for a proximal humerus fracture.

 

(SAE10PE.68) A 6-year-old girl sustains an ankle injury after falling on roller blades. An AP radiograph is shown in Figure 68. Treatment should consist of which of the following? 

 

 

  1. Closed manipulation and a long leg cast

  2. Closed manipulation and a short leg cast

     

  3. Long leg cast without manipulation

     

  4. Open reduction and internal fixation with a screw crossing the growth plate

     

  5. Open reduction and internal fixation with fixation parallel to the physis

 

PREFERRED RESPONSE 5

 

The child has a Salter-Harris type IV injury involving both the growth plate and the articular surface of the ankle. This injury pattern has a high risk of physeal arrest; open reduction and internal fixation is indicated to realign the physis and joint surface. The best method of fixation to avoid growth arrest is one that does not cross the physis. This is usually achieved by an epiphyseal screw or pins parallel to the physis. If the metaphyseal fragment were large enough, a transverse metaphyseal screw could be used. The incidence of growth arrest following physeal ankle injuries is high and long-term follow-up is indicated.

 

(SAE12FA.90) A football player who injured his right lower extremity during a game could not get up and reported extreme pain. The initial sideline evaluation showed a probable anterior cruciate, posterior cruciate, and lateral collateral ligament rupture with a very unstable knee. He also reports pain in his ankle and is unable to dorsiflex the ankle. He has limited sensation over the dorsum of his foot. Examination reveals no swelling of the ankle and no pain with passive range of motion of the ankle. What is the most likely diagnosis? Review Topic

  1. Tibial nerve injury

     

  2. Associated ankle fracture

     

  3. Acute compartment syndrome

     

  4. Injury to the common peroneal nerve

     

  5. Rupture of the tibialis anterior tendon

 

PREFERRED RESPONSE 4

 

It is not uncommon to sustain a peroneal nerve injury in association with a knee dislocation or multi-ligament injury. There should always be a high index of suspicion for this injury, and the vascular status to the leg should be carefully evaluated. From the history and examination, there is no indication that the ankle was fractured. A compartment syndrome will not develop within a few minutes of the injury. It takes several hours for a compartment syndrome to develop and become symptomatic. The tibial nerve supplies the plantar aspect of the foot. An acute rupture of the tibialis anterior tendon in a young person is very uncommon, and it is associated with pain and localized swelling about the ankle. It is also unlikely that it would lead to sensory loss.

 

(SAE11OS.113) Compared with myodesis, osteomyoplasty offers which of the following advantages in transtibial amputation? 

 

  1. Enhanced end-bearing

     

  2. Early prosthetic fitting

     

  3. Immediate weight bearing

     

  4. Fibular abduction

     

  5. Decreased surgical morbidity

 

PREFERRED RESPONSE 1

 

Osteomyoplasty in transtibial amputation, originally described by Ertl in 1949, features creation of a bone bridge between the distal tibia and fibula, which is

theorized to enhance bony stability and increase end-bearing of the residual limb, and may enhance patient-perceived functional outcomes relative to myodesis. Fibular abduction is a known complication of traditional myodesis techniques, and is believed to represent syndesmotic instability. Osteomyoplasty requires additional surgical time and increased surgical morbidity, and because the success of the procedure is dependent on achieving bony union, early prosthetic fitting and immediate weight bearing are typically contraindicated.

 

(SAE09FA.9) A 27-year-old man now reports dorsiflexion and inversion weakness after an automobile collision 6 months ago in which compartment syndrome developed isolated to the anterior and deep posterior compartments. Examination reveals the development of a progressive cavovarus deformity, but the ankle and hindfoot remain flexible. In addition to Achilles tendon lengthening, which of the following procedures is most likely to improve the motor balance of his foot and ankle? Review Topic

 

  1. Anterior tibialis tendon transfer to the dorsolateral midfoot

     

  2. Posterior tibialis tendon transfer to the dorsolateral midfoot

     

  3. Peroneus longus tendon transfer to the dorsolateral midfoot

     

  4. Peroneus brevis tendon transfer to the dorsolateral midfoot

     

  5. Flexor hallucis longus tendon transfer to the peroneus brevis

 

PREFERRED RESPONSE 3

 

Compartment syndrome of the anterior and deep posterior compartments results in anterior tibialis and posterior tibialis tendon weakness, respectively. Furthermore, the long flexors to the hallux and lesser toes will be weak as well. The intact peroneus longus overpowers the weak anterior tibialis tendon, resulting in plantar flexion of the first metatarsal, cavus, and hindfoot varus. Therefore, transferring the peroneus longus to the dorsolateral midfoot reduces the first metatarsal plantar flexion torque, and possibly augments ankle dorsiflexion torque.

 

(SBQ12FA.1) Figure A is a radiograph of a healthy, independent 51-year-old male. He is treated with immediate open reduction internal fixation to prevent which of the following complications? 

 

 

 

  1. Fracture non-union

  2. Avascular necrosis

     

  3. Skin necrosis

     

  4. Plantar flexion weakness

     

  5. Ankle stiffness

 

PREFERRED RESPONSE 3

 

Figure A shows an avulsion fracture of the calcaneal tuberosity. Immediate open reduction and internal fixation is required to prevent wound complications.

 

Displaced avulsion fractures of the calcaneal tuberosity should be managed urgently to prevent necrosis of the soft tissues overlying the heel. In these injuries, the Achilles tendon is securely attached to the fractured tuberosity. Urgent closed reduction and casting is usually not possible due to the power and proximal pull of the triceps surae. Surgical fixation is required. The best treatment modality is open reduction and bone-to-bone fixation with screws. Closed reduction and percutaneous pinning fixation is not strong enough to provide a stable fixation of the tuberosity.

 

Lui reported on avulsion fractures of the bony insertion of the Achilles tendon at the calcaneus. He stated that screw fixation alone is not sufficient for repair of these injuries. His technique involved two suture anchors used capture the small bone fragment to the calcaneus. This allowed for the pull of the triceps surae to be neutralized and early physical therapy.

 

Hess et al. looked at a case series of calcaneal tuberosity avulsion fractures that were

treated in a delayed fashion. All three patients with posterior tuberosity calcaneal avulsion fractures developed skin necrosis because of a delay in treatment.

 

Figure A shows a displaced posterior tuberosity calcaneal avulsion fracture. Illustration A shows skin breakdown overlying the posterior tuberosity calcaneal avulsion fracture.

 

Incorrect Answers:

Answer 1: The amount of displacement is an indication for fixation, however urgent treatment does not improve union rates with these fractures. Answer 2: Tuberosity calcaneal avulsion fractures rarely disrupt the blood supply to the avulsion fragment and are not associated with avascular necrosis. Answer 3: Plantar flexion weakness is a known complication of these injuries despite many treatment options. Answer 4: Ankle stiffness is most commonly related to surgical fixation methods and post-operative immobilization and delayed rehabilitation.

 

(SAE11UE.85) A 40-year-old man sustains a scapular body fracture after an all-terrain vehicle accident. Which of the following is the most commonly associated injury? 

 

  1. Chest injury

     

  2. Clavicle fracture

     

  3. Glenohumeral dislocation

     

  4. Humeral fracture

     

  5. Axillary nerve injury

 

PREFERRED RESPONSE 1

 

Chest injury (rib fracture, pneumothorax, hemothorax, contusion) is the most commonly associated injury in patients who have sustained a significant scapular injury. Chest injury becomes even more commonly found when the scapula has more than one zone of injury (ie, multiple fractures). Humeral fracture, clavicle fracture, and axillary nerve injury are not as common as chest injury.

 

(SAE08OS.72) Compared with surgically treated patients, patients with extra-articular distal third humeral shaft fractures that are treated nonsurgically with functional bracing can be expected to show which of the following findings? 

 

  1. Similar loss of elbow motion

     

  2. Greater loss of elbow extension

     

  3. Higher rate of varus malalignment

     

  4. Higher rate of functionally limiting malalignment

     

  5. Significantly better shoulder motion

 

PREFERRED RESPONSE 1

 

In a retrospective review of patients with extra-articular distal humeral shaft fractures treated surgically versus nonsurgically, the authors found that the amount of motion loss was not different between the treatment groups. Of 21 patients in the nonsurgical group, one lost 20 degrees of extension, one lost 30 degrees of extension, and one patient lost 15 degrees of flexion. Of the 19 patients in the surgical group, two patients lost 5 degrees of extension, and one each lost 10, 15, and 20 degrees of extension, respectively. One patient lost 5 degrees of flexion and one lost 15 degrees of flexion. The average loss of motion in the surgical group was 3 degrees, compared with 6 degrees in the nonsurgical group, but this difference was not significant. One hundred percent of the nonsurgically treated fractures healed. Both groups of patients regained shoulder motion within 10 degrees of normal. In the nonsurgically treated group, 10 healed with less than 10 degrees of malalignment, 6 healed with 11 to 20 degrees of malalignment, and three healed with greater than 30 degrees of malalignment, but the authors did not report any functional problems due to these deformities.

 

(OBQ15.65) A healthy 27 year-old-male is brought into the emergency department after a fall from height. He has a suspected left C8-T1 nerve injury. Which of the

following findings would most suggest a root avulsion injury rather than a brachial plexus injury at this level? 

 

  1. Reduced radial artery pulse

     

  2. Double break in the ipsilateral superior shoulder suspensory complex

     

  3. Elevated hemidiaphragm

     

  4. Musculocutaneous nerve deficit

     

  5. Drooping of the left eyelid

 

PREFERRED RESPONSE 5

 

Drooping of the left eyelid is a presenting feature of Horner's syndrome. Horner's syndrome represents a disruption of the sympathetic chain via C8 and/or T1 root avulsion after trauma.

 

Brachial plexus injuries are often classified as preganglionic vs. postganglionic injuries. Preganglionic injuries are typically avulsion injuries proximal to the dorsal root ganglion. Clinical features suggestive of lower root avulsion injury include a person falling from height clutching on object to save himself, Horner’s syndrome (drooping of the eyelid (ptosis), pupillary constriction (miosis) and anhidrosis), absence of a Tinel sign or tenderness to percussion in the neck, and a normal histamine test (C8-T1 sympathetic ganglion - intact triple response (redness, wheal, flare)).

 

Caporrino et al. reviewed 102 patients to assess the best modality (e.g. physical examination, MRI and nerve conduction studies [NCSs]) for diagnosing and localizing brachial plexus injuries. They found the best diagnostic performance with physical examination (sensitivity = 97.8%; specificity = 30.8%) and NCSs (sensitivity

= 98.9%; specificity = 23.1%). MRI had inferior performance for all measurements. They conclude that NCSs exhibited superior performance to MRI, and should be considered a more reliable supporting tool after detailed physical examination.

 

Incorrect Answers:

Answer 1, 2: The superior shoulder suspensory complex (SSSC) is located in close proximity to the brachial plexus. Disruption of the SSSC may affect the local anatomy and cause local brachial plexus or vascular injury. Answer 3: Elevated hemidiaphragm (Phrenic nerve C3,4,5) may be suggestive an upper brachial plexus root avulsion, e.g. C5, C6 injury. Answer 4: Musculocutaneous nerve deficiency (C5 deficiency) results in weakness to the biceps.

 

(SAE08UE.17) A patient reports hyperesthesia over the base of the thenar eminence following volar locked plating of a distal radius fracture. A standard volar approach of Henry was used. What is the most likely cause of the hyperesthesia? 

 

  1. Complex regional pain syndrome

     

  2. Wartenberg’s syndrome

     

  3. Carpal tunnel syndrome

     

  4. Palmar cutaneous nerve injury

     

  5. C7 radiculopathy

 

PREFERRED RESPONSE 4

 

The palmar cutaneous branch of the median nerve separates from the median nerve approximately 4 to 6 cm proximal to the wrist crease and travels between the median nerve and the flexor carpi radialis tendon. It supplies the skin of the thenar region. This nerve is at risk for injury with retraction of the digital flexor tendons in plating the distal radius. Wartenberg’s syndrome is compression of the superficial radial nerve which innervates the dorsum of the thumb and the first dorsal web space. Carpal tunnel syndrome causes dysesthesias of the thumb, index, and/or middle fingers. C7 radiculopathy affects the index and middle fingers.

 

(SAE11OS.84) What is the greatest benefit of external fixation for treatment of displaced and unstable pelvic ring injuries with hemodynamic instability? Review Topic

 

  1. It provides rigid fixation of the pelvis.

     

  2. It helps maintain a stable clot over injured vessels.

     

  3. It stabilizes the visceral injuries.

     

  4. It allows the patient to sit and eat without pain.

     

  5. It is more comfortable than skeletal traction.

PREFERRED RESPONSE 2

 

External fixation has been shown not to provide rigid fixation of the pelvis because a long moment arm from the fixator clamps to the posterior pelvis. Even with elaborate constructs, the fixator alone is inferior to internal fixation of the posterior ring. The main purpose of acute external fixation is to stabilize the initial clot forming about the injured pelvic plexus. This initial clot contains innate clotting factors, making it more stable, if not dislodged. If this clot is dislodged after hemorrhage and factor poor resuscitation, the ensuing hemorrhage will not have the same ability to form a stable clot around the injured vessels. The fixator does not stabilize any visceral structures. It interferes with the ability to sit depending on its application and is no more or less comfortable than skeletal traction.

 

(SAE11AN.8) A 35-year-old woman is involved in a head-on collision while driving. Initial radiographs are shown in Figures 8a and 8b. Injury to what vessel increases the risk for osteonecrosis of the injured bone? 

 

 

 

  1. Dorsalis pedis artery

  2. Perforating peroneal artery

     

  3. Lateral tarsal artery

     

  4. Artery of the tarsal canal

     

  5. Artery of the tarsal sinus

 

PREFERRED RESPONSE 4

The patient has a Hawkins type III talar neck fracture-dislocation with a risk of osteonecrosis ranging from 69% to 100%. Anatomic studies have shown that the artery of the tarsal canal supplies the lateral two thirds of the talar body. The other vessels listed provide no significant contribution to the talus.

 

(SAE08UE.80) Which of the following findings is a contraindication to isolated percutaneous pinning of a distal radius fracture? 

 

  1. Dorsal comminution

     

  2. Volar comminution

     

  3. Radial comminution

     

  4. Intra-articular fracture

     

  5. Physeal fracture

 

PREFERRED RESPONSE 2

 

Intrafocal pinning allows the Kirschner wires to be placed through a site of comminution and then drilled through intact cortex. Generally Kapandji intrafocal pinning is done for dorsal comminuted extra-articular dorsal bending fractures, but it also may be used to elevate and buttress radial comminution. Simple intra-articular fractures can also be treated with pinning alone. Intrafocal pinning works best as a dorsal or radial buttress to prevent shortening. When there is volar comminution, the fracture is prone to shortening and supplemental external fixation or plating is recommended.

 

(SAE09TR.80) Which of the following long bone fracture patterns occurs after a pure bending force is exerted to the bone? 

 

  1. Spiral

  2. Oblique

     

  3. Transverse

     

  4. Segmental

     

  5. Comminuted

 

PREFERRED RESPONSE 3

 

A pure bending force produces a transverse fracture pattern. Spiral fractures are mainly rotational, oblique are uneven bending, segmental are four-point bending, and comminuted are either a high-speed torsion or crush mechanism.

 

(OBQ05.272) A 65-year-old female presents with the injury seen in Figures A and B after a motor vehicle collision. She is hemodynamically unstable and undergoes emergent pelvic supra-acetabular external fixation followed by laparotomy. She is now hemodynamically stable and cleared for surgery. She has no evidence of neurologic deficit on examination. Which of the following factors is a relative contraindication to open reduction and plating of her posterior pelvic injury from an anterior approach? 

 

 

 

  1. Sacral fracture

  2. Prior laparotomy

     

  3. Supra-acetabular external fixtator

     

  4. Parasymphyseal fractures

     

  5. Ipsilateral proximal femur fracture

PREFERRED RESPONSE 1

 

An anterior approach to the sacroiliac (SI) joint is indicated with displaced SI joint dislocations that cannot be reduced with closed or percutaneous techniques. One contraindication to anterior exposure of the SI joint is comminuted sacral fracture patterns.

 

Posterior pelvic ring injuries that are unable to be reduced by closed techniques may require open reduction via anterior or posterior approaches. Relative contraindications to anterior approach include comminuted sacral fractures, morbid obesity, iliac wing external fixation, and ipsilateral diverting colostomy. In the presence of a comminuted sacral fracture, aggressive medial dissection would be required and would place the L5 nerve root at risk.

 

Simpson et al describe their initial results with open reduction and internal fixation of the SI joint via an anterior exposure in a series of 16 patients. They note that sacral alar comminution is a contraindication to the anterior approach

 

Jones provides an overview of the operative treatment of posterior pelvic ring injuries. He demonstrates reduction and fixation techniques via both anterior and posterior exposures.

 

Incorrect Answers:

Answer 2: Prior laparotomy is not a contraindication to open anterior approach if the bowel is in continuity and there is no evidence of wound infection Answer 3: Supracetabular external fixation does not interfere with anterior approach to the SI joint Answer 4: Anterior pelvic ring injuries such as parasymphyseal fractures do not affect the choice of approach Answer 5: An ipsilateral proximal femur fracture does not affect the choice of approach

 

(SAE11OS.58) The radiographic finding in Figure 58 is indicative of what type of acetabular fracture? 

 

 

  1. Anterior column

  2. Posterior column

     

  3. Associated both column

     

  4. Transverse

     

  5. Associated transverse plus posterior wall

 

PREFERRED RESPONSE 3

 

The radiographic image is an obturator oblique view of the left acetabulum and demonstrates a "spur" sign. It represents a spike of bone from the intact hemipelvis and no articular surface remains with the hemipelvis, which defines the associated both column fracture. The weight-bearing surface of the acetabulum is displaced with the femoral head. In all other patterns, at least part of the articular surface remains with the intact hemipelvis.

 

(SBQ12TR.58) A 35-year-old male was involved in a high speed motorcycle accident. He has a closed head injury, bilateral pulmonary contusions and splenic rupture. His orthopaedic injuries are shown in Figure A. He has a blood pressure of 90/50 mm Hg and a heart rate of 115, despite aggressive resuscitation. An arterial blood gas reveals that his blood lactate is 3.5 and base deficit is -6 mmol/L. Following successful closed reduction of the right hip in the operating room with a percutaneous inserted Schantz pin, what is the next most appropriate treatment for his orthopaedic injuries? Review Topic

 

 

  1. Bilateral open reduction and internal fixation

  2. Open reduction internal fixation on the right, reamed intramedullary nailing on the left

     

  3. Temporizing external fixation on the right, open reduction and internal fixation on the left

     

  4. Bilateral reamed intramedullary nailing

     

  5. Bilateral temporizing external fixation

 

PREFERRED RESPONSE 5

 

This patient presents with features of hemodynamic instability and a high injury severity score. The next most appropriate treatment would be temporizing external fixation bilaterally. This patient meets the criteria for damage control orthopaedics.

 

Damage control orthopaedics is an approach that contains and stabilizes orthopaedic injuries so that the patient's overall physiology does not undergo further inflammatory insult. As a result, external fixation of femoral shaft fracture and pelvic stabilization is an effective treatment under this strategy. Other indications include vascular injury and severe open fracture.

 

Pallister et al. reviewed the effects of surgical fracture fixation on the systemic inflammatory response to major trauma. They show that early stabilization of major long bone fractures is beneficial in reducing the incidence of acute respiratory distress syndrome and multiple organ failure. However, early fracture surgery increases the post-traumatic inflammatory response, which carries a higher complication rate compared to temporary fixation.

 

Tisherman et al. created clinical guidelines for the endpoints of resuscitation. Level I data found that standard hemodynamic parameters do not adequately quantify the degree of physiologic derangement in trauma patients. The initial base deficit, lactate level, or gastric pH should be used to stratify patients with regard to the need for ongoing fluid resuscitation.

Pape et al. retrospectively reviewed the impact of early total care vs. damage control orthopaedics in the treatment of femoral shaft fractures in polytrauma patients. They found a significantly higher incidence of acute respiratory distress syndrome (ARDS) with intramedullary nailing (15.1%) compared to external fixation (9.1%) when DCO subgroups were compared.

 

Figure A is a pelvic AP radiograph showing a right hip fracture-dislocation with an ipsilateral femoral shaft fracture. On the left side there is a displaced pertrochanteric hip fracture.

 

Incorrect Answers:

Answer 1-3: Open reduction internal fixation would not be indicated in damage control orthopaedics.

Answer 4: Bilateral reamed intramedullary nailing would be indicated in early total care of these injuries. As this patient is hemodynamically unstable, with a high injury severity score and bilateral lung contusions, damage control orthopaedics would be indicated.

 

(SAE11OS.60) An elderly woman with osteoporosis falls from a standing height, sustaining a low-energy fracture of the acetabulum. What structures are most likely fractured? 

 

  1. Posterior column and posterior wall

     

  2. Anterior column and medial wall

     

  3. Anterior column, posterior column, and ischium (T-type fracture)

     

  4. Anterior column and posterior column (transverse fracture)

     

  5. Anterior column, posterior column, and posterior wall (transverse/posterior wall fracture)

 

PREFERRED RESPONSE 2

 

Epidemiologic studies suggest that 4,000 acetabular fractures occur in elderly patients each year in the United States. This accordingly may become the most common age group to present with this fracture. In elderly patients with considerable osteoporosis, a typical fracture pattern may present with intrapelvic dislocation of the femoral head with compromise to the anterior column and "medial wall." The resulting fractures are often complex fracture patterns with extensive comminution and displacement. These

may present as atypical fracture patterns not always conforming to classic injury patterns described by Judet and associates. This fracture pattern seen commonly in geriatric patients results from low-energy falls with force directly applied to the greater trochanter. Fractures involving the posterior column and/or wall and transverse fracture patterns involving both the anterior and posterior columns occur infrequently in this age group. They are, however, more commonly encountered in younger age groups as a result of higher energy trauma.

 

(SAE07SM.29) An 18-year-old football player is injured after making a tackle with his left shoulder. He has decreased sensation over the lateral aspect of the left shoulder and radial aspect of the forearm. Motor examination reveals weakness to shoulder abduction and external rotation as well as elbow flexion. He has decreased reflexes of the biceps tendon on the left side but full, nontender range of motion of the cervical spine. What anatomic site has been injured? Review Topic

 

  1. Fourth cervical nerve root

     

  2. Upper trunk of the brachial plexus

     

  3. Middle trunk of the brachial plexus

     

  4. Lateral cord of the brachial plexus

     

  5. Axillary nerve

 

PREFERRED RESPONSE 2

 

The athlete has symptoms referable to the axillary, musculocutaneous, and suprascapular nerves resulting from an injury to the upper trunk of the brachial plexus. This portion of the plexus is formed by contributions of the fourth through sixth cervical nerve roots. This area is often contused or stretched following a tackling maneuver that results in either depression of the shoulder from contact at Erb’s point or traction of the upper plexus from forced stretching of the neck to the contralateral side.

(SAE11OS.28) Figure 28 is the lateral radiograph of a patient who sustained an intraarticular fracture of the calcaneus. The structure (*) depicted by the arrows most likely represents which osseous component of the calcaneus? 

 

 

 

  1. Middle facet

  2. Sustentaculum tali

     

  3. Extruded lateral wall

     

  4. Medial portion of the posterior facet

     

  5. Lateral portion of the posterior facet

 

PREFERRED RESPONSE 5

 

Fractures of the calcaneus occur as a result of shear and compression forces. Foot position at the time of impact, the force of the impact, and bone quality all dictate the degree of comminution and fracture line orientation. Two primary fracture lines are consistently observed, one of which divides the calcaneus into medial and lateral portions. An essential feature of this fracture line is that it creates a fragment (sustentaculum tali) that remains attached to the talus by the interosseous ligament. This medial portion (constant fragment) of the posterior facet retains its normal anatomic position beneath the posterior talus. Its corresponding lateral component (labeled with an * in the figure), however, can be found displaced inferiorly within the body of the calcaneus. It is often rotated 90 degrees (as depicted in Figure 28) in relation to the remainder of the subtalar joint. This gives the appearance of what has been described as the "double-density" sign. The middle facet is more anterior and less commonly displaced. The lateral wall is nonarticular.

(OBQ14.259) A 24-year-old woman is thrown from her motorcycle and sustains the closed injury shown in Figures A through C. Open reduction and internal fixation is planned. What surgical technique will best allow visualization of the joint surface and allow early range of motion? 

 

 

 

  1. Bryan-Morrey approach and parallel plating

  2. Triceps reflecting anconeus pedicle approach and parallel plating

     

  3. Triceps reflecting anconeus pedicle approach and orthogonal plating on the posteromedial and lateral surfaces

     

  4. Olecranon osteotomy and parallel plating

     

  5. Olecranon osteotomy and orthogonal plating on the posteromedial and lateral surfaces

 

PREFERRED RESPONSE 4

 

Access to complex intra-articular fractures is best achieved by an olecranon osteotomy (OO). Fixation can be with parallel plating or orthogonal plating.

 

Bicolumnar fixation of distal humerus fractures should follow the principles outlined by O'Driscoll: Distal fragments should be held by as many screws as possible; every screw in the distal fragments should pass through a plate; each screw should engage as many articular fragments as possible.

 

Galano et al. review treatment for bicolumnar distal humerus fractures. They note that the olecranon osteotomy, Alonso-Llames triceps sparing and Campbell triceps splitting approaches expose 57%, 46% and 35% of the articular surface, respectively. The OO and paratricipital (triceps sparing) approaches allow for early ROM. Protected motion is required for the O'Driscoll TRAP and Bryan-Morrey approaches for tendon-to-bone healing.

 

Coles et al. retrospectively reviewed the OO in fixation of 70 fractures. Osteotomy fixation was with an intramedullary screw and dorsal ulnar wiring, or with a plate.

The rate of OO increased with fracture difficulty (from AO type C1-C3). There was 1 delayed union but no nonunions.

 

Figures A and B show a AO/OTA type C2 intraarticular distal humerus fracture. Figure C is a coronal CT scan showing intraarticular comminution. Illustration A shows fixation of the fracture with bicolumnar plating through an olecranon osteotomy approach. Illustration B shows the various approaches to the distal humerus (left, Campbell triceps splitting; center left, O'Driscoll triceps reflecting anconeus pedicle; center right, Bryan-Morrey approach, leaving the triceps attached laterally to the fasciocutaneous flap, but elevating it off the ulna; right, olecranon osteotomy). Illustration C shows 3 methods of olecranon osteotomy (A and B, Intraarticular transverse; C-F, Extra-articular oblique; G, Intra-articular chevron).

 

Incorrect Answers:

Answer 1: The Bryan-Morrey approach provides excellent medial exposure, but inadequate lateral exposure. The triceps is lifted subperiosteally, and the anconeus is released subperiosteally. Triceps repair depends on tendon-bone healing. Answer 2: The O'Driscoll TRAP approach detaches triceps and anconeus from their distal insertions in a V-shaped flap that is reflected proximally. Repair to bone is via drill holes in the ulna. Early mobilization is not recommended. Answers 3 and 5: Orthogonal plating is performed on the POSTEROLATERAL and MEDIAL surfaces.

 

(SBQ12TR.84) Figure A shows a radiograph of a 30-year-old male who underwent fixation of a left leg injury just over two years ago. He presents with persistent pain and drainage from the distal wound despite 4 months of oral antibiotics. He has no systemic symptoms. He has a past medical history of Grave's disease and Irritable Bowel Syndrome. What would be the best management at this stage? Review Topic

 

 

  1. Chronic suppressive, culture-directed, antibiotic therapy

  2. Above knee amputation

     

  3. Endocrine consultation, irrigation and debridement, removal of hardware and negative-pressure wound therapy

     

  4. Irrigation and debridement, removal of hardware, over-reaming medullary canal, external fixation and culture-directed antibiotics

     

  5. Irrigation and debridement, retention of hardware, acute bone grafting and culture-directed antibiotics

 

PREFERRED RESPONSE 4

 

This is a case of fracture nonunion in the setting of chronic osteomyelitis and infected hardware. The best treatment option available would be irrigation and debridement, removal of hardware, ring external fixator and culture directed antibiotics.

 

The management of infected nonunion in the setting of chronic osteomyelitis is technically demanding. The aims of treatment are to eradicate the infection and obtain bone union. Non-surgical options are largely unsuccessful in patients with draining chronic osteomyeltis in the setting of infected hardware and nonunion. Surgical options involve incision and debridement of necrotic tissue followed by reconstruction of bone and possible soft tissue (to provide healthy viable coverage). The most common techniques are ringed fixator/circular frames, staged intramedullary device with or without external fixator, free tissue transfer, or radical

debridement, bone grafting, and fixation.

 

Motsitsi et al. reviewed the management of infected nonunion of long bones. They suggest that the Ilizarov technique is regarded as a standard treatment in infected nonunion of the tibia. When there is bone defect after debridement, the bone can be shortened or treated with bone transport.

 

Egol et al look at a series of patients with chronic osteomyelitis. Limb salvage should be attempted in all patients. The presence of a chronic draining sinus requires surgical debridement and culture directed antibiotics. Infected hardware should be removed. A two-stage strategy is the best and well-proven treatment option.

 

Figure A shows a intramedullary nail in the left tibia. There is a moderate amount of bone loss at the fracture site with mixed sclerotic bone suggestive of osteomyelitis.

 

Incorrect Answers:

Answer 1: Chronic suppressive therapy would be considered in a patient that is unable to undergo surgical intervention Answer 2: Amputation is a primary option for patients unable to undergo lengthy or multiple surgical interventions. Answer 3: After removing the hardware, the fracture non-union will need to be stabilized.

Answer 5: This patients has a chronic infection that requires radical bone debridement and local infection control. The use of acute bone grafting in a single stage procedure in the distal 1/3 tibia (poor blood supply) is less ideal, especially in the setting of infection.

 

(SAE12TR.23) Figure 23 is the radiograph of a 22-year-old woman who was involved in a motor vehicle collision. She reports isolated pain in her left shoulder. She is hemodynamically stable, respiring comfortably, and neurovascularly intact. Based on these findings, which of the following statements regarding treatment is most appropriate? 

 

 

  1. Union rates are in excess of 95% if treated nonsurgically.

  2. A figure-of-8 brace is superior to a sling for nonsurgical management.

     

  3. Open reduction and internal fixation increases the likelihood of a nonunion.

     

  4. Open reduction and internal fixation results in improved functional outcomes.

     

  5. Open reduction and internal fixation and nonsurgical management have equivalent outcomes at 1 year.

 

PREFERRED RESPONSE 4

 

The patient has sustained an isolated, closed, transverse fracture of the middle third of the clavicle with greater than 100% displacement and greater than 2 cm of shortening. Whereas the traditional treatment of clavicle fractures has been overwhelmingly conservative, recent reports suggest that surgical fixation should be considered for certain injury patterns. The union rates of displaced clavicle fractures are more recently noted to be approximately 85%, which is lower than the traditional literature. In a prospective randomized trial of clavicle fractures with greater than 100% displacement, union rates were higher and functional outcomes were better at all time points up to 1 year after injury in the surgical group when compared with nonsurgical management.

 

(SAE12TR.21) Figure 21 is the radiograph of a 45-year-old woman who was severely injured in a motorcycle crash. Her injuries include a traumatic subarachnoid hemorrhage, bilateral pneumothoraces with pulmonary contusions and flail chest, fracture-dislocation of the left hip, and open fractures of the right distal femur and proximal tibia. Antibiotics and tetanus are administered in the emergency department. The patient is intubated and bilateral chest tubes are placed. A closed reduction is

performed on the left hip. After appropriate resuscitation, what is the most appropriate initial management of the right knee injury? 

 

 

 

  1. Skeletal traction

  2. Irrigation and débridement of the open fractures

     

  3. Irrigation and débridement and spanning external fixation of the knee

     

  4. Open reduction and internal fixation of the proximal tibia and distal femur

     

  5. Percutaneous screw fixation of the articular fragments with retrograde femoral nailing and antegrade tibial nailing

 

PREFERRED RESPONSE 3

 

Although the radiographic evaluation is incomplete, the single lateral view shows a comminuted fracture of the distal femur with suspicion of intra-articular injury and an ipsilateral proximal tibia fracture. This is an open fracture that requires antibiotics, débridement , and skeletal stabilization. The fractures are complicated and the patient is polytraumatized; therefore, rapid but complete surgical débridement and simple stabilization of the knee with a spanning external fixator would be the most appropriate management. Definitive surgical stabilization will likely be complicated and is less desirable during the early post-injury period.

 

(OBQ14.212) A 72-year-old female presents to the office 5 weeks after distal radius fracture surgery with the findings seen in Figure A. She performed daily cleansing

with soap and water and dry dressings. Which of the following has been shown to decrease the risk of developing this complication? 

 

 

 

  1. A solution of 0.45% sodium chloride and hydrogen peroxide

  2. A solution of 0.9% sodium chloride and hydrogen peroxide

     

  3. A solution of 0.45% sodium chloride and chlorhexidine

     

  4. A solution of 0.9% sodium chloride and chlorhexidine

     

  5. None of the above, as specialized cleansing solutions do not decrease the risk this complication

 

PREFERRED RESPONSE 5

 

There is no difference between daily showers with soap, water and dry dressings, and solutions comprising (1) saline and chlorhexidine, or (2) saline and hydrogen peroxide.

 

The risks of external fixation include cellulitis, pin-track drainage, osteomyelitis and pin loosening. The rate of complications is about 20%. Chlorhexidine gluconate has broad spectrum activity against gram-positive and negative bacteria by disrupting cell membranes. It is not affected by blood and has low skin irritancy. Studies show that simple pin-site care (showers, dry dressings) maintains a low infection rate.

 

Egol et al. performed a randomized trial examining external fixation pin tracts about the wrist. They compared (1) weekly dry dressings, (2) daily pin care with half normal saline (NS) and hydrogen peroxide (H2O2) and (3) chlorhexidine discs with weekly changes. They found pin-site complications in 19%, with no difference between the 3 groups. They do not recommend additional wound care beyond sterile dressings.

 

Stinner et al. performed a survey on the use of half-pins by the limb lengthening and reconstruction society. They found that most respondents preferred hydroxyapatite coating (81%) because of improved fixation and decreased loosening (less radiographic pin-tract rarefaction and greater extraction torque, which may lead to

lower infection rates). Most encouraged shower (60%) and washing solution (67%)(soap and water, peroxide or saline) for pin site care.

 

Figure A shows an external fixator around the wrist with cellulitis around the proximal half-pins.

 

Incorrect Answers

Answers 1-4: Hydrogen peroxide and chlorhexidine based solutions have not been conclusively shown to decrease the rate of pin site complications.

 

(SAE12TR.18) Figures 18a through 18c show injuries sustained by a 22-year-old woman after falling 45 feet while mountain climbing. After being airlifted to the nearest trauma center, her arterial blood gas was 7.21, pO2 84, pCO2 48, and delta base -11 mmol/L. Her Hgb is 8.7 and her resuscitation is ongoing. Based on this data, what would be the best management of her orthopaedic injuries? 

 

 

  1. External fixation of the pelvis, external fixation of the distal femur, and splinting of the humerus

  2. External fixation of the pelvis, external fixation of the distal femur, and intramedullary nailing of the humerus

     

  3. External fixation of the pelvis, open reduction and internal fixation of the distal femur, and splinting of the humerus

     

  4. Open reduction and internal fixation of the pelvis, open reduction and internal fixation of the distal femur, and intramedullary nailing of the humerus

     

  5. Open reduction and internal fixation of the pelvis, open reduction and internal fixation of the distal femur, and open reduction and internal fixation of the humerus

 

PREFERRED RESPONSE 1

 

The patient is under-resuscitated and would benefit from minimally invasive stabilization of the pelvic ring and long bone fractures in a "damage-control" approach. External fixation of the pelvis and femur can be performed quickly and with minimal blood loss which should limit the "second hit" associated with more

prolonged, invasive surgery. Upper extremity fractures are best managed acutely with splints in this clinical setting. Definitive fracture fixation should be delayed until the patient is adequately resuscitated.

 

(OBQ15.111) A 34-year-old male arrives intubated with a closed head injury to the trauma bay after a motor vehicle accident. After initial hospital workup and resuscitation, he is transferred to the intensive care unit. In addition to multiple systemic injuries, he sustained the closed injury shown in Figure A. Intracompartmental pressure monitoring of the limb measure in a range from 28-30 mm Hg. Which of the following sustained blood pressure measurements would support the treatment of limb fasciotomy? Review Topic

 

 

 

1 110/60 mmHg

2 115/55 mmHg

 

3 92/64 mmHg

 

  1. both a and b

     

  2. a, b and c

 

PREFERRED RESPONSE 2

A reported indication to perform fasciotomy includes an ICP measurement that is elevated to 30 mm Hg below the diastolic blood pressure. This would be the case if this patient's blood pressure was consistently around 115/55 mmHg (dBp=55; ICP=30; delta p = dBp-ICP = 25 mmHg).

 

Given the poor outcomes associated with missed compartment syndromes, it is important to obtain both clinical and objective data when determining if a patient needs fasciotomies. A clinical assessment is the diagnostic cornerstone of acute compartment syndrome. However, the intracompartmental pressure measurement has been advocated to help confirm the diagnosis in patients where there remains uncertainty after clinical exam - especially with intubated patients. An absolute compartment pressure >30 mm Hg or a difference in diastolic pressure and compartments pressure (delta p) <30 mm Hg may help to confirm the necessity for fasciotomy.

 

McQueen et al. prospectively reviewed 116 patients with tibia fractures that had continuous monitoring of their anterior compartment for 24 hours. They found that using an absolute pressure of 30 mmHg would have resulted in 50 patients (43%) treated with unnecessary fasciotomies. They conclude using a differential pressure of

30 mmHg is a more reliable indicator of compartment syndrome.

 

Olson et al. provide a review of compartment syndrome for the lower extremity. They discuss a variety of injuries and medical conditions that may initiate acute compartment syndrome, including fractures, bleeding disorders, and other trauma. Although the diagnosis is primarily a clinical one, they also recommend supplementation with compartment pressure measurements in equivocal cases.

Figure A shows a closed comminuted tibial shaft fracture. Incorrect Answers:

Answer 1: dBp=60; ICP=30; dBp-ICP = 30 mmHg. Therefore delta is not <30, which would not definitely prove to be compartment syndrome. Both the absolute and delta p measurements are not greater, and less than, the published parameters. Answer 3: dBp=64; ICP=30; dBp-ICP = 34 mmHg. Therefore delta is not <30

 

(SAE08UE.108) A 35-year-old man sustained the closed injury shown in Figure 52 in his dominant extremity. Neurologic function is normal. Treatment should consist of 

 

 

  1. functional bracing.

  2. a sling and swathe.

     

  3. intramedullary nail fixation.

     

  4. open reduction and internal fixation.

     

  5. iliac crest bone graft.

 

PREFERRED RESPONSE 1

 

Functional bracing has been demonstrated to have a very high rate of healing without any functional limitations in a large series of patients. Surgery is reserved for “floating elbows,” open injuries, neurovascular injuries, and those fractures that go on to nonunion.

 

(SAE12TR.94) Figure 94 is the initial lateral radiograph of the foot of a 55-year-old woman who felt a pop in her left foot as she stepped off the curb. She subsequently had severe heel pain and could not bear weight. Examination in the emergency department revealed a bony prominence over the posterior aspect of the heel with blanching of the surrounding skin. What is the most appropriate orthopaedic management? 

 

 

  1. Immediate cast immobilization with maximum plantar flexion

  2. Immediate surgical treatment with percutaneous reduction and screw fixation

     

  3. Immediate open reduction and internal fixation via an extensile lateral approach

     

  4. Short leg splint, elevation, and delayed open reduction and internal fixation

     

  5. Short leg splint, elevation, and conversion to cast immobilization when soft-tissue swelling has resolved

 

PREFERRED RESPONSE 2

 

The patient has a calcaneal tuberosity fracture, similar to the tongue-type fracture except the fracture line exits posterior to the posterior facet. The Achilles tendon is attached to the displaced fragment and pulls the fragment proximally. These are relatively uncommon fractures, but have the same (or greater) potential as tongue-type fractures for soft-tissue compromise and necrosis. Immediate management with reduction and fixation is indicated to prevent heel ulceration and secondary complications such as deep infection. Fracture fixation generally does not require an extensile approach or plate fixation, and may benefit from decreasing the forces acting on the displaced fragment by supplemental gastrocnemius recession.

 

(SAE09FA.12) Figures 5a and 5b show the radiographs of a 56-year-old man who was seen in the emergency department following a twisting injury to his left ankle. Examination in your office 3 days later reveals marked swelling and diffuse

tenderness to palpation about the ankle and leg. What is the next most appropriate step in management? 

 

 

 

  1. MRI of the ankle

  2. CT of the ankle

     

  3. Technetium bone scan

     

  4. Radiographs of the tibia and fibula

     

  5. Repeat radiographs of the ankle in 5 to 7 days

 

PREFERRED RESPONSE 4

 

The radiographs show an isolated posterior malleolus fracture which, given the injury mechanism, is highly suspicious for a Maisonneuve injury. As with any suspected extremity injury, radiographs including the joints above and below the level of injury are acutely indicated. Although MRI may reveal a ligamentous injury to the ankle and CT may show asymmetry of the ankle mortise or syndesmosis, both studies are considerably more costly and are not indicated in the absence of a complete radiographic work-up. Technetium bone scan is nonspecific and would be of limited value in this instance, as would repeat radiographs of the ankle.

(SAE12TR.54) Figures 54a and 54b are the radiographs of a 23-year-old man who fell from a height and sustained an isolated injury to his right leg. Which of the following is a useful surgical technique to optimize alignment during intramedullary nailing? 

 

 

 

  1. Move the starting point slightly medial

  2. Move the starting point slightly lateral

     

  3. Hyperflexion of the knee

     

  4. Anterior blocking screw

     

  5. Medial blocking screw

 

PREFERRED RESPONSE 2

 

Fractures of the proximal metadiaphysis of the tibia can be treated successfully with intramedullary nails, but previous studies showed rates of malalignment of up to 84%. The typical deformity is valgus and procurvatum. An ideal starting point is mandatory and should be slightly lateral to the medial border of the lateral tibial eminence on a true AP view and very proximal and anterior on a true lateral view with appropriate coronal and sagittal trajectory of the entry reamer. A medial start point will exacerbate valgus deformity and should be avoided. A reduction should be obtained and maintained during reaming, implant insertion, and interlocking. This can be facilitated via a variety of techniques including intraoperative external fixation, percutaneous reduction clamps or joysticks, semi-extended positioning, blocking screws that are typically inserted posterior and lateral to the nail, and ancillary plate fixation. With careful attention to these techniques, more recent studies report low rates of malalignment.

 

(SBQ12TR.65) A patient falls and sustains the isolated injury seen in Figures A and

B. The surgical plan includes open reduction and internal fixation with a small mini-fragment plate using a direct lateral approach. During the approach, the forearm was placed in a fully pronated position. What would be the correct position of the forearm during plate application? 

 

 

 

  1. Full pronation

  2. 25 degrees pronation

     

  3. Neutral

     

  4. 25 degrees supination

     

  5. Full supination

 

PREFERRED RESPONSE 3

 

Using the lateral approach (Kocher or Kaplan), the correct placement of the arm should be in a neutral position so that the plate can be placed on the bare area of the proximal radius.

 

Displaced radial head fractures with less than 3 fragments can be amendable to open reduction internal fixation. The methods of fixation include buried or headless screws, if placed at the articular surface, or posterolateral plating, if placed in the bare area. The safe zone for plating is located at a 90-110 arc from the radial styloid to Lister's tubercle with the arm in neutral rotation. This position helps to avoid impingement of ulna against the plate with forearm rotation. It should be noted that during the approach, that the forearm should be fully pronated to avoid injury to the posterior interosseous nerve.

 

Mathew et al. reviewed the concepts of terrible triad injuries of the elbow. Radial

head fractures are treated conservatively when there is an isolated minimally displaced (less than 2mm) fracture with no mechanical block to motion. Open reduction internal fixation is used for Mason II or III fractures with < 3 fragments. Radial head replacement is considered for comminuted fractures (Mason Type III) with 3 or more fragments.

 

Cheung et al. reviewed the surgical approaches to the elbow. The lateral approach (Kocher or Kaplan) is most commonly used with these injuries. The Kocher approach utilizes the intramuscular plane between anconeus and extensor carpi ulnaris. Kaplan utilizes the plane between extensor digitorum commons and extensor carpi radialis brevis.

 

Figure A and B show AP and lateral radiographs of the left elbow. There is a displaced radial head fracture. Illustration A shows a schematic diagram of the radial head "safe zone" between the radial styloid to Lister's tubercle.

 

Incorrect Answers:

Answers 1,2,4,5: Plating the arm in these positions would not adequately expose the bare area of the radial head using the lateral approach to the elbow.

 

(OBQ15.8) Mirror therapy is indicated for which of the following clinical conditions? 

 

  1. Impaired proprioception after lower extremity injury

     

  2. Two-stage flexor tendon reconstruction

     

  3. Plumbism

     

  4. Anterior cruciate ligament reconstruction

     

  5. Phantom pain

 

PREFERRED RESPONSE 5

 

Mirror therapy is a useful technique to improve phantom pain in amputees. Phantom limb pain is a painful sensation that is perceived within a body part that no longer exists.

 

Under mirror therapy, a patient is allowed to feel the imaginary movement of the removed body part behaving as normal body movement through a mirror. The mirror

image of the normal body part helps reorganize and integrate the mismatch between proprioception and visual feedback of the amputated segment.

 

Tilak et al. performed a randomized controlled trial of 26 patients with phantom limb pain, and found that mirror therapy as well as transcutaneous electrical nerve stimulation both significantly improved pain scores, but no difference was seen between the two groups. They recommend use of both modalities in treatment of phantom limb pain, as they are noninvasive and inexpensive.

 

Chan et al. reviewed 22 patients utilizing mirror therapy for amputation related phantom limb pain, and found that patients reported a significant decrease in pain when utilizing this treatment modality as compared to covered mirror therapy or mental imagery therapy.

 

Illustration A shows a patient with an upper extremity amputation using a mirror to perform movements during a mirror therapy session.

 

Incorrect Answers:

Answer 1: Mirror therapy is not indicated for proprioception training. Answer 2: Mirror therapy is not indicated for flexor tendon injuries. Answer 3: Mirror therapy is not indicated for lead poisoning. Answer 4: Mirror therapy is not indicated for ACL reconstruction.

 

(SAE12TR.11) Figures 11a and 11b show the radiographs of the open fracture of a 46-year-old man who injured his elbow on his nondominant arm in a motorcycle crash. On the day of injury, he underwent irrigation and debridement of the fracture. He was also treated with antibiotics. Which of the following definitive treatment methods will most likely lead to the best functional outcome? 

 

 

  1. Cast immobilization

  2. Intramedullary screw fixation

     

  3. Open reduction and plate fixation

     

  4. Open reduction and internal fixation with tension band wiring

     

  5. Fragment excision and triceps advancement

 

PREFERRED RESPONSE 3

 

The patient has an open comminuted transolecranon fracture-dislocation. This occurs when the distal humerus is driven through the proximal ulna, and it is often associated with comminution of the olecranon and proximal ulna. The distal fragment translates anteriorly. Results of surgical treatment of transolecranon fracture-dislocations are best and most reliable when the fracture is reduced anatomically and plate fixation is used. Nonsurgical management is not indicated in this injury pattern. Excision of the comminuted fragments and advancement of the triceps will likely lead to persistent anterior instability of the elbow. Tension band wiring relies on cortical contact which will not be possible in this fracture. Intramedullary screw fixation is also not possible because of the significant comminution.

 

(SAE11OS.138) Figures A through C are the radiograph and CT scans of a 42-year-old man who sustained an injury to both of his ankles and underwent surgical repair 2 weeks prior to presentation to your office. One ankle is healing well. On the

contralateral side, he reports pain and restricted ankle range of motion. Management should consist of 

 

 

 

  1. the addition of more aggressive physiotherapy.

  2. observation and continued non-weight-bearing.

     

  3. addition of an anteriorly directed "syndesmosis screw."

     

  4. loosening the syndesmotic screws from an overtightened position.

     

  5. removal of screws, re-reduction of the syndesmosis, and revision fixation.

 

PREFERRED RESPONSE 5

 

This patient has a malreduced syndesmosis. The CT scans clearly show the fibula to be subluxated posteriorly relative to the incisura; therefore, surgical revision is warranted. Revision surgery should include either removal of the current screws with accurate reduction of the syndesmosis and new screw placement or repair of the posterior malleolar fragment, which will in turn reduce the syndesmosis. Addition of an anteriorly directed screw to the current construct will not change the malalignment. Loosening the syndesmotic screws or addition of aggressive physiotherapy will not correct the malrotation of the distal fibula within the incisura which is seen on the CT scan. Outcomes after these injuries are related to the reduction of the ankle mortise.

 

(OBQ14.235) A 28-year-old construction worker sustains the closed injury shown in Figures A and B after a fall from a height. He is taken to the operating room. What is the next best step? 

 

 

 

  1. Locked anterior tibial plating and fibular plating

  2. Locked medial tibial plating and fibular plating

     

  3. Reamed intramedullary nailing without fibular plating

     

  4. Unreamed intramedullary nailing and fibular plating

     

  5. Reamed intramedullary nailing and fibular plating

 

PREFERRED RESPONSE 5

 

This patient has an extraarticular distal tibia fracture and distal fibula fracture. Reamed intramedullary nailing and fibular plating is indicated in this case.

 

In the distal tibial metaphysis, there is no snug endosteal fit for an IM nail. Center-center nail placement in both proximal and distal fragments is necessary to maintain alignment. There is also increased stress on distal locking bolts to maintain fracture alignment. Assuming static medial-lateral distal locking screws, accurate coronal plane and rotational alignment is achieved by fibular plating as a first step. This also

prevents late loss of alignment because of distal locking screw toggle. Reamed nailing allows a stiffer, larger nail to be placed, and allows redistribution of endosteal osteogenic material to the fracture site. Although there is endosteal vascular compromise, this does not affect fracture healing because of intact periosteal supply.

 

Bhandari et al. conducted a prospective, randomized, blinded comparison of 622 patients who had reamed nailing, and 604 who had unreamed nailing. For closed fractures, a significantly greater number in the unreamed group required bone grafting, implant exchange and dynamization. There was no difference in groups for open fracture nailing.

 

Egol et al. retrospectively reviewed distal metaphyseal tibia-fibula fractures treated with IM nailing with (25 cases) and without (47 cases) adjunctive plating. They found that plating was associated with maintenance of reduction (significant) as was the use of 2 medial-lateral distal locking bolts (not significant). They recommend fibular plating when IM nailing for distal tibia fractures.

Figures A and B show an extraarticular distal tibia fracture with distal fibula fracture. Incorrect Answers

Answers 1 and 2: Locked tibia and fibular plating plating has more soft tissue dissection compared with fibular plating and tibia IM nailing. This increases the risk of wound complications. Answer 3: Fibular plating reduces postoperative valgus malalignment and late loss of alignment.

Answer 4: In closed tibia fractures, unreamed nailing has a significantly greater need for bone-grafting, secondary dynamization, and exchange nailing compared with reamed nailing. In open tibia fractures, this difference is not significant.

 

(SAE09FA.70) A 23-year-old man who was the restrained driver in a car involved in a high-speed motor vehicle accident sustained the closed injury shown in Figures 32a through 32c. Which of the following factors has the greatest impact on the risk of osteonecrosis? 

 

 

  1. Surgical stabilization within 6 to 8 hours of injury

  2. Extent of initial fracture displacement

     

  3. Nicotine use

     

  4. Posterior-to-anterior screw fixation

     

  5. Anatomic fracture reduction

 

PREFERRED RESPONSE 2

 

The incidence of osteonecrosis following displaced talar neck fractures is most related to the extent of initial fracture displacement. With increasing fracture displacement, the tenuous vascular supply to the talar body is more at risk for damage, thereby increasing the risk of osteonecrosis. Although displaced talar neck fractures have historically been considered a surgical emergency, recent studies have shown that the timing of surgical intervention bears no impact on the development of osteonecrosis. While nicotine use has an influence on fracture healing, it has never been shown to be a factor in osteonecrosis, nor has posterior-to-anterior screw fixation or the quality of fracture reduction.

 

(SBQ12TR.45) A 32-year-old female is involved in a motor vehicle collision and suffers a right hip dislocation. She is in the twelfth week of pregnancy. Evaluation in the emergency department reveals no other injuries and ultrasound reveals a strong fetal heart rate and no abnormalities. She undergoes emergent closed reduction but the hip remains unstable and a traction pin is placed. Post-reduction films are shown in Figure A. What is the most appropriate next step in management? Review Topic

 

 

 

  1. Acute open reduction internal fixation

  2. Exam under anesthesia

     

  3. Skeletal traction for 6-8 weeks

     

  4. Fetal monitoring until 15 weeks followed by open reduction internal fixation

     

  5. Percutaneous pinning

 

PREFERRED RESPONSE 1

 

This patient has a large posterior wall fracture of the right acetabulum with an unstable hip. The most appropriate next step in treatment is open reduction and internal fixation.

 

Fixation of acetabular fractures during pregnancy is not contraindicated in the setting

of stable fetal heart rate and no abnormalities on pelvic ultrasound. There is, however, an increased risk of complications for the mother and fetus. Injury severity and mechanism are most closely associated with increased rate of fetal complications. The trimester of pregnancy is not associated with increased risk of complications.

 

Leggon et al. reviewed 101 cases of pelvic and acetabular fractures in pregnant patients and found mechanism of injury and injury severity were associated with higher mortality for both mother and fetus. Trimester of pregnancy was not associated with increased mortality.

 

Flik et al. reviewed orthopaedic trauma in a pregnant patients and recommended fetal ultrasound for assessment of fetal well-being in all pregnant patients.

 

Desai et al. investigated orthopaedic trauma during pregnancy and reported minimal radiation risk to the fetus when obtaining x-rays. They also advocate for LMWH as one of the safest choices for anticoagulation.

 

Figure A is an x-ray showing a right posterior wall acetabular fracture. Figures B and C are Judet views of the pelvis focusing on the right hip. A large posterior wall fragment is visible in Figure B.

 

Incorrect Answers:

Answer 2,3,5: ORIF is the most appropriate treatment for this posterior wall fracture. There is no contraindication to ORIF. Answer 4: There is no indication to delay treatment as the fetal heart rate is normal and ultrasound shows no abnormalities.

 

(SAE09FA.11) A 29-year-old patient sustains a closed, displaced joint depression intra-articular calcaneus fracture. In discussing potential complications of surgical intervention through an extensile lateral approach, which of the following is considered the most common complication following surgery? 

 

  1. Nonunion

     

  2. Deep infection

     

  3. Delayed wound healing

     

  4. Peroneal tendinitis

     

  5. Posttraumatic arthritis

PREFERRED RESPONSE 3

 

Delayed wound healing and wound dehiscence is the most common complication of surgical management of calcaneal fractures through an extensile lateral approach, occurring in up to 25% of patients. Most wounds ultimately heal with local treatment; the deep infection rate is approximately 1% to 4% in closed fractures. Posttraumatic arthritis may develop despite open reduction and internal fixation, but the percentages remain low. Peroneal tendinitis may occur from adhesions within the tendon sheath or from prominent hardware but is relatively uncommon. Nonunion of a calcaneal fracture is rare.

 

(SAE12FA.77) Following surgery for an ankle fracture, which of the following is considered the most important factor in achieving a satisfactory outcome? Review Topic

 

  1. Physical therapy

     

  2. Early weight bearing

     

  3. Anatomic alignment

     

  4. Early range of motion of the ankle

     

  5. Calcium and vitamin D administration

 

PREFERRED RESPONSE 3

 

The only factor that is prognostic for outcomes is the quality of the reduction. None of the other factors has any effect on the outcome. Early range of motion or physical therapy may offer temporary effects, but these small advantages do not last beyond 3 months after surgery.

(SAE09TR.26) A 20-year-old man is brought to the emergency department after a high-speed motor vehicle accident. His initial blood pressure is 70/40 mm Hg. He is currently receiving intravenous fluids as well as blood. His Focused Assessment with Sonography for Trauma examination did not show any free fluid in his abdomen and his chest radiograph is unremarkable. An AP pelvis radiograph is shown in Figure 15. What is the next most appropriate step in the management of his pelvic injury? Review Topic

 

 

 

  1. Inlet and outlet views of the pelvis to better delineate the injury

  2. Angiography

     

  3. Laparotomy

     

  4. Open reduction and internal fixation of the pelvis

     

  5. Placement of a pelvic binder around the patient

 

PREFERRED RESPONSE 5

 

This hypotensive patient has an obvious open book injury of the pelvic ring on the AP pelvis radiograph and further radiographs are not needed prior to the initiation of treatment. Although angiography may be indicated if he does not respond to stabilization of his pelvis and fluid/blood administration, temporary stabilization of the pelvis with a sheet or binder should be performed first because it is simple, quick, and has been shown to be effective. This patient does not need a laparotomy at this point since the FAST examination did not show any free intra-abdominal fluid and his chest radiograph was unremarkable, leaving the most likely source of bleeding the pelvic fracture. Open reduction with internal fixation of a pelvic injury is not indicated in an acutely ill patient.

(OBQ15.34) A 25-year-old woman sustains a fall on an outstretched hand. She complains of elbow pain. Examination reveals tenderness over the lateral elbow and pain on elbow motion. Injury radiographs and CT scans are shown in Figures A and B, respectively. What is the next best step? 

 

 

 

  1. Splint until swelling subsides, then long-arm cast

  2. Excision of fracture fragments

     

  3. Closed reduction and percutaneous pinning

     

  4. Open reduction and internal fixation (ORIF)

     

  5. Radial head replacement

 

PREFERRED RESPONSE 4

 

This patient has Mason II radial head fracture. ORIF will give the best results.

 

Non-/minimally displaced radial head fractures without a block to rotation can be managed nonoperatively. Complete articular fractures with >=3 fragments do better with radial head replacement. Indications for ORIF include large articular surface fragments, > 2 mm of displacement, mechanical block to forearm rotation, or associated fractures or ligament injuries requiring surgery.

 

Pike et al. retrospectively compared patients undergoing ORIF for isolated radial head fractures with radial fractures associated with other fractures/dislocations. They found no differences in pain/disability and complications or secondary capsular release between groups.

 

Yoon et al. retrospectively compared isolated partial articular displaced (2-5mm) radial head fractures treated nonoperatively vs ORIF. They found no clinical benefit with ORIF compared to non-operative management. The ORIF group had more complications. Younger patient age and larger fracture displacement favored operative intervention. Younger patients fared worse.

 

Figures A and B are radiographs and 3D reformatted CT images showing a displaced partial articular radial head fracture.

Incorrect Answers:

Answer 1: Nonoperative management is indicated for non-/minimally displaced fractures.

Answer 2: Excision is an option for small partial articular fragments <25% of the radial head.

Answer 3: There is no role for closed reduction and pinning in an adult. Answer 5: Radial head replacement is indicated for complete articular fractures with

>= 3 fragments.

 

(SAE11OS.143) A 22-year-old woman underwent closed reduction and percutaneous pinning with casting of a displaced extra-articular distal radius fracture. The surgery was completed with a supraclavicular regional anesthesia. After the block wears off, she reports new onset dense numbness in the palmar aspect of the thumb, index, and middle fingers as well as severe pain in the hand. What is the next step in management? 

 

  1. Bivalve the cast and follow up in 1 week

     

  2. Return to the operating room for open carpal tunnel release

     

  3. Compartment pressure monitoring of the hand

     

  4. Emergent nerve conduction velocity studies

     

  5. Exploration of the supraclavicular brachial plexus

 

PREFERRED RESPONSE 2

 

The injury represents a somewhat uncommon problem after surgical treatment of distal radius fractures; however, vigilance is required to detect the acute presentation of a carpal tunnel syndrome. In this case, urgent release of the tunnel is recommended. Bivaling the cast alone is indicated when the pain is less severe, and only when the numbness is very minimal and more generalized. Compartment syndrome of the hand is almost unheard of in the setting of a distal radius fracture; rather it is more commonly associated with a crush injury to the hand. There is no role for emergent nerve conduction velocity studies or brachial plexus exploration.

(SAE12TR.89) Figure 89a is the radiograph of a 24-year-old man who was involved in a motor vehicle accident. A closed reduction is performed and a post-reduction CT scan is shown in Figure 89b. What is the next most appropriate step in management? 

 

 

 

  1. Total hip arthroplasty

  2. Removal of loose bodies

     

  3. Protected weight bearing

     

  4. Assessment of hip stability

     

  5. Open reduction and internal fixation

 

PREFERRED RESPONSE 4

 

The radiograph and CT scan show a posterior hip dislocation with an associated posterior wall acetabular fracture. The next step in management is assessment of hip instability. As suggested by Tornetta, assessment of hip instability with dynamic stress views is helpful to determine which posterior wall fractures are unstable and therefore require open reduction and internal fixation. Although protected weight bearing may be correct if the hip is stable, stability needs to be determined first. The CT scan reveals a small fragment in the cotyloid fossa. However, in this location, the presence of a loose body alone does not require surgical treatment. Hip instability needs to be assessed before determining if this fracture should be treated with open reduction and internal fixation. Total hip arthroplasty is not appropriate for a 24-year-old patient with a small posterior wall acetabular fracture.

(OBQ13.53) An 19-year-old male presents to the emergency room following an motor vehicle accident as an unrestrained driver. Examination reveals unilateral jugular vein engorgement. Chest and special view radiographs are seen in Figures A and B respectively. Following CT scan of the chest, the next step in management is Review Topic

 

 

 

  1. Nonsurgical management and follow-up CT scan in 6 weeks

  2. Closed reduction in the emergency room under sedation

     

  3. Closed reduction in the operating room under general anesthesia with thoracic surgery on standby, followed by figure-of-8 clavicle strap immobilization for 4 weeks

     

  4. Closed reduction in the operating room under general anesthesia with thoracic surgery on standby, followed by compression plating

     

  5. Open reduction in the operating room under general anesthesia, followed by transarticular pinning with K-wires

 

PREFERRED RESPONSE 3

 

This patient has a right posterior sternoclavicular (SC) dislocation. Management involves closed reduction and bracing. Closed reduction should be performed with a thoracic surgeon available in the event of mediastinal involvement.

 

The SC joint can dislocate anteriorly or posteriorly. Posterior dislocations are first treated with closed reduction. If closed reduction fails, open reduction is indicated. Early complications of posterior SC dislocation include pneumothorax, laceration/erosion/occlusion of great vessels, esophageal rupture and brachial plexus compression. Late complications include tracheoesophageal fistula, stridor and dysphagia.

 

Groh et al. reviewed traumatic SC injuries. Reduction maneuvers in posterior SC dislocation include: (1) traction on the arm and slowly bringing it into extension, (2) traction with the arm in adduction and posterior pressure applied to the shoulder, and

(3) pulling anteriorly on a towel clip encircling the medial clavicle. Chronic instability after posterior SC dislocations can be managed with figure-of-8 semitendinosus graft or medial clavicle resection and reattachment of the clavicle to the first rib with dacron tape.

Glass et al. performed a systematic review on SC dislocations. They found mediastinal compression occurred 30% of the time with posterior dislocations.

 

Figures A and B are radiographs demonstrating asymmetry of the SC joints, characteristic of a right posterior SC dislocation (Figure B is not a serendipity view). Illustration A demonstrates how in POSTERIOR dislocation, the clavicle appears INFERIOR, and in ANTERIOR dislocation, the clavicle appears SUPERIOR on a serendipity view radiograph respectively. Illustration B shows the imaging technique for a serendipity view radiograph. Illustration C is a reconstructed CT image of the patient showing left posterior SC dislocation.

 

Incorrect Answers:

Answer 1: Nonsurgical management is not advised because of potential complications of posterior dislocations. Unstable dislocations will remain dislocated at 6 weeks if untreated.

Answer 2: Closed reduction in the emergency room is ill-advised because of potential mediastinal involvement. An operating room and thoracic surgeon should be available during the reduction maneuver. Answer 4: Compression plating across the sternoclavicular joint is not recommended. Answer 5: Pinning with K-wires is contraindicated because of potential catastrophic consequences of wire migration.

 

(SAE11OS.38) Figures 38a and 38b are the MRI scans of a 28-year-old man who reports progressively worsening severe back pain for the past 3 months. He denies fevers, chills, weakness, or neurologic dysfunction. Examination reveals tenderness to palpation over the lumbar spine but normal neurologic findings. Laboratory studies reveal an elevated erythrocyte sedimentation rate and C-reactive protein; blood cultures are positive for methicillin-sensitive Staphylococcus aureus. In addition to intravenous antibiotics, what is the next step in management? Review Topic

 

 

  1. CT-guided biopsy

  2. Application of lumbar orthosis

     

  3. Repeat MRI within 48 hours

     

  4. Anterior lumbar debridement and fusion

     

  5. Posterior lumbar debridement and fusion

 

PREFERRED RESPONSE 2

 

The patient's symptoms and MRI findings are consistent with osteomyelitis and diskitis at L3-4 with a paraspinal fluid collection. Cultures confirm bacterial involvement. Given that finding, a biopsy of the level is unnecessary. Surgical treatment for infection is not indicated given the lack of neurologic deficit. Nonsurgical management is the best option, including both intravenous antibiotics and an external lumbar orthosis. A repeat MRI scan within a short duration would not impact clinical care. More important is close clinical follow-up to confirm response to treatment and identify any potential neurologic deficits that may develop.

 

(SAE09TR.54) A 57-year-old man involved in a motor vehicle accident sustains an injury to his right shoulder. A spot AP radiograph is shown in Figure 34. What is the next most appropriate step in the orthopaedic management of this patient? Review Topic

 

 

  1. Axillary view

  2. CT of the shoulder

     

  3. Closed reduction

     

  4. Sling and close follow-up

     

  5. Functional brace

 

PREFERRED RESPONSE 1

 

The next step in the management of this injury is completion of the shoulder trauma series. An axillary radiograph, which can be quickly performed in the emergency department, must be obtained to accurately assess the humeral head relationship to the glenoid. If difficulty is encountered, a “Velpeau” axillary may be substituted. If that fails to elucidate the status of the glenohumeral joint, a CT scan should be obtained.

 

(OBQ14.240) A 50-year-old woman loses her balance and falls off 3 stairs, landing awkwardly on her right knee. Examination reveals loss of dorsalis pedis and posterior tibialis pulses and the foot is cool to the touch. Her body mass index is 52. Reduction is performed successfully in the emergency room under sedation. Pulses are still not palpable and the foot remains cool to touch. Ankle brachial index is 0.72 (right) and

1.01 (left). What is the most appropriate next step? 

 

 

  1. Splint the knee and admit the patient for hourly ankle brachial index monitoring

  2. Emergent vascular surgical exploration

     

  3. Emergent external fixation and reassessment of perfusion

     

  4. CT angiography and admit the patient for hourly neurovascular checks

     

  5. Interventional radiology consult for formal angiography and stenting

 

PREFERRED RESPONSE 2

 

This patient has a posterior knee dislocation with an ischemic limb that does not reverse following reduction. Emergent vascular exploration and reconstruction is indicated.

 

Knee dislocations are associated with popliteal artery injury in 18-45% of cases and range from intimal tears to complete transection. Amputation rates of 85% have been reported if revascularization is delayed greater than 6 to 8 hours. Neurologic injury occurs in 15-40% of cases and is most common after posterolateral dislocation. The peroneal nerve is more commonly injured.

 

Rihn et al. reviewed the acutely dislocated knee. They recommend a vascular consult if pulses are weak, or ABI is compromised. They warn that in arterial injury, pulses, temperature and capillary refill can be normal. If the limb remains ischemic, surgical exploration and revascularization is indicated.

 

Medina et al. systematically reviewed neurovascular injury after knee dislocation in 862 patients. Vascular injury rate was 18%, and nerve injury rate was 25%. Repair was performed in 80% of vascular injuries, and amputation in 12%. The most vascular injury was seen in KDIIIL injuries (32%) and posterior dislocation (25%).

 

Figure A is an AP radiograph of a posterior knee dislocation. Figure B is a lateral showing the same injury.

 

Incorrect Answers:

Answer 1: The limb remains ischemic. Splinting and observation alone are inappropriate.

Answer 3: External fixation is appropriate for immobilization following reduction, especially for very obese patients (BMI>50) where casting/splinting is not possible, and especially to protect a vascular reconstruction. However, the MOST important next step is vascular exploration to attempt revascularization. Answers 4 and 5: CT angiography and standard angiography will only serve to delay vascular exploration of the ischemic limb. On-table angiograms will be performed during the surgery, obviating the need for preoperative angiography outside the operating room (OR).

 

(SAE12TR.32) A 26-year-old man is involved in a high-speed motorcycle accident. He sustains a grade IIIB open tibia fracture. Examination reveals a large soft-tissue defect and an insensate foot. What is the expected outcome in this scenario? Review Topic

 

  1. Equal functional outcome when limb salvage is compared with amputation

     

  2. Worse functional outcome with limb salvage than with primary amputation

     

  3. Better functional outcome when amputation is compared with limb salvage

     

  4. Amputation within 6 months of injury

     

  5. Permanent loss of plantar sensation

 

PREFERRED RESPONSE 1

 

The Lower Extremity Assessment Project data have shown that absent plantar sensation is not an indication for primary amputation. When looking at a comparison between an insensate salvage group and a sensate salvage group at 2 years follow-up, both groups had an equal proportion (55%) of normal plantar sensation and functionally both groups were equivalent. Absent plantar sensation at initial evaluation is not prognostic for long-term plantar sensory status or functional outcome.

(SAE11OS.24) A 20-year-old unrestrained driver sustained a midshaft femur fracture in a high-speed motor vehicle accident. The femoral neck was evaluated with a CT scan with 2-mm cuts through the hip; no fracture was identified. What additional studies (if any) should be performed to minimize the risk of having an undiagnosed femoral neck fracture? 

 

  1. Postoperative MRI scan

     

  2. Postoperative bone scan

     

  3. Preoperative AP pelvic radiograph

     

  4. No additional imaging studies are needed

     

  5. Intraoperative fluoroscopic images of the femoral neck

 

PREFERRED RESPONSE 5

 

Nondisplaced femoral neck fractures may occur concurrently with high-energy injuries of the femur. Preferably, these are identified prior to or during surgery so that the fracture can be stabilized to prevent displacement and minimize the risk of osteonecrosis. However, the diagnosis of these injuries can be difficult. Tornetta and associates reported on standardized protocol that involved preoperative radiographs and CT scans with fine cuts through the femoral head. This protocol improved the detection of femoral neck fractures compared with situations with no set protocol. Of the 16 fractures detected, 13 were identified preoperatively. Of the three fractures that were missed by the screening, one was iatrogenic, one of these was detected at the time of surgery with intraoperative internal/external views of the femoral neck, and one had a late displacement. The overall rate of nondisplaced femoral neck fractures in this study was 7.5%, of which 91% were treated at the time of initial surgery having been identified on preoperative and/or intraoperative radiographs. Care must be taken not to neglect careful scrutiny of the femoral neck at the time of surgery even if preoperative imaging studies do not detect a fracture. No one method has been shown to have a 100% success rate. Postoperative bone scans and MRI scans are not routinely used.

 

(SAE09TR.44) A 32-year-old man has an open comminuted humeral shaft fracture. Examination reveals absence of sensation in the first web space and he is unable to fully extend the thumb, fingers, and wrist. What is the recommended treatment following irrigation and debridement of the fracture? 

  1. Functional bracing

     

  2. Hanging long arm cast immobilization

     

  3. Intramedullary nailing

     

  4. Open reduction and internal fixation, radial nerve exploration

     

  5. External bone stimulator

 

PREFERRED RESPONSE 4

 

There is a high incidence of partial or complete laceration of the radial nerve with high-energy open fractures of the humeral shaft. The recommended treatment is irrigation and debridement of the fracture followed by open reduction and internal fixation and exploration of the radial nerve. If the nerve is completely lacerated, primary repair may be performed but poor outcomes have been reported. If a large zone of nerve injury is identified, delayed nerve grafting is advocated.

 

(SAE11OS.191) What complication is most likely to occur following proximal humeral fixation with a locked plate-and-screw construct? 

 

  1. Screw penetration

     

  2. Rotator cuff injury

     

  3. Axillary nerve damage

     

  4. Fracture of the humeral shaft

     

  5. Impingement

 

PREFERRED RESPONSE 1

 

Proximal humeral locking plates have been associated with screw penetration (incidence 23%). The rotator cuff injury is not due to the plate or its application and is associated with dislocations in the elderly. Axillary nerve damage, while possible, has a low reported incidence from open reduction and internal fixation of the proximal humerus with locking constructs. Impingement and fracture of the humeral shaft are also unlikely. More likely but not offered as a choice is the problem of varus

reduction which can result in failure. However, penetration of the screws remains the most commonly reported complication.

 

(SAE08OS.51) Patient outcome after open reduction and internal fixation of tibial plateau fractures shows that patients older than 50 years of age when compared to younger patients have 

 

  1. equal results for all fracture types.

     

  2. better outcomes in high-energy fractures.

     

  3. better outcomes in low-energy fractures.

     

  4. a higher wound complication rate.

     

  5. worse outcomes in low-energy fractures.

 

PREFERRED RESPONSE 5

 

Several studies have shown worse functional results in patients older than 40 or 50 years of age compared to younger patients after open reduction and internal fixation of tibial plateau fractures. Two studies showed that older patients with less severe fractures performed less favorably than younger patients with more severe injuries. Only 35% of patients older than 50 years were satisfied with their results independent of fracture type.

 

(SAE08OS.32) A patient is seen in the emergency department after a motor vehicle accident. He reports right hip pain and chest pain. Initial hypotension has responded to a fluid bolus. Radiographs reveal a posterior hip dislocation with a small posterior acetabular wall fracture. You are called at home and informed of the findings. What is the next most appropriate step in management? Review Topic

 

  1. Obtain a CT scan to assess the injury.

  2. Obtain an MRI scan to assess for osteonecrosis.

     

  3. Reduce the hip and evaluate hip stability.

     

  4. Perform open reduction of the hip in the operating room.

     

  5. Ask the emergency room physician to transfer the patient to a higher level trauma center.

 

PREFERRED RESPONSE 3

 

An immediate reduction of the hip is required. Transfer to a trauma center may be indicated to treat a possible chest injury and the acetabular fracture. Reduction of the hip dislocation should be considered emergent and should be performed prior to transfer. Additional diagnostic studies prior to hip reduction are not necessary. Most hip dislocations can be reduced closed and this is the preferred management.

 

(OBQ14.57) A 22-year-old female falls off the back of a motorcycle and sustains the injury in Figure A. She is hemodynamically unstable and massive transfusion protocol is activated. What is the correct ratio of transfusion of packed red blood cells, platelets and plasma? 

 

 

 

1 2:1:1

2 3:1:1

 

3 1:1:1

 

4 1:1:2

5 1:1:3

 

PREFERRED RESPONSE 3

 

When massive transfusion protocols are activated, the correct ratio of packed red blood cells, platelets, and plasma is 1:1:1. Implementation of these ratios allows for homeostasis of factors that avoid tipping the balance of the coagulation cascade.

 

Incorrect ratios of transfusion can lead to physiologically unfavorable states, including coagulopathy and life threatening metabolic imbalances. The lethal triad includes acidosis, hypothermia, and coagulopathy. Also possible but rare is associated citrate toxicity, and transfusion-associated lung injury.

 

Langford et al. reviewed the typically presentation, initial evaluation, management and treatment of pelvic fractures. APC-III fractures have the highest association with bleeding and the authors emphasize the importance of close hemodynamic status monitoring. Initial clot can be achieved with the placement of a pelvic binder around the greater trochanters, and the initiation of massive transfusion if necessary. Frequent, close monitoring of associated electrolytes are mandatory.

 

Elmer et al. explain the preferred ratio of 1:1:1 as a goal, not to achieve a physiologic hemodynamic state, but to offer a favorable environment to facilitate resuscitation while avoiding excessive fluctuations in the coagulation cascade and metabolic state. In this paper, they work through a case presentation of a patient requiring massive transfusion, in which electrolyte balance was maintained with supplemental calcium gluconate, potassium, and crystalloid fluids as needed with frequent, regular lab analysis.

Figure A exhibits a widely displaced APC pelvic injury. Incorrect answers:

Answers 1, 2, 4, and 5 are incorrect ratios of transfusion.

 

(SAE08UE.101) A 38-year-old woman with diabetes mellitus reports a 6-week history of fever and pain localized to the right sternoclavicular joint. Local signs on examination include swelling about the joint, erythema, and increased warmth. Initial aspiration of the joint reveals Staphylococcus aureus. Radiographs reveal medial clavicular osteolysis. What is the most effective treatment at this time? 

  1. Broad-spectrum parenteral antibiotics

     

  2. Repeat aspirations

     

  3. Irrigation and debridement

     

  4. Hyperbaric oxygen

     

  5. Resection of the sternoclavicular joint

 

PREFERRED RESPONSE 5

 

Based on the findings, the treatment of choice is resection of the sternoclavicular joint. Antibiotic therapy, repeat aspirations, hyperbaric oxygen, and simple irrigation and debridement are generally ineffective and associated with a high rate of recurrence.

 

(SBQ12TR.56) A 92-year-old female sustains the injury shown in Figure A to her nondominant extremity as the result of a non-syncopal ground-level fall. She denies any previous injury or pain of the elbow, and her medical history is significant only for osteoporosis and hypothyroidism. What is the most appropriate treatment for her injury? 

 

 

 

  1. Immediate range of motion as tolerated with a sling for comfort

  2. Long arm cast for 3 weeks, then physical therapy for motion

     

  3. Open reduction and internal fixation

     

  4. Radiocapitellar arthroplasty

     

  5. Total elbow arthroplasty

 

PREFERRED RESPONSE 5

 

Use of total elbow arthroplasty (TEA) in the elderly is a well-recognized method of treatment of complex distal humerus fractures. This procedure allows for improved ROM, improved patient-reported outcomes, and decreased revision rates as compared to fixation.

 

TEA is a preferred alternative for ORIF in elderly patients with complex distal humeral fractures that are not amenable to stable fixation. Elderly patients appear to accommodate to objective limitations in function with time, which is important, as most recommendations list restrictions of lifting no more than 5-10 pounds postoperatively.

 

McKee et al conducted a prospective, randomized, controlled trial to compare functional outcomes, complications, and reoperation rates in elderly patients with displaced intra-articular, distal humeral fractures treated with open reduction-internal fixation (ORIF) or primary semiconstrained total elbow arthroplasty (TEA). Patients who underwent TEA had a quicker procedure, improved DASH scores at 6 months, improved elbow ROM, and decreased revision rates.

 

Athwal et al review TEA and the options available at the time of publication. They also report on the techniques and purported advantages of arthroplasty as compared to fixation of complex distal humerus fractures.

 

Frankle et al reviewed patients >65 years old with distal humerus fractures at a minimum of 2 years follow-up. Outcomes were excellent in 33% of cases undergoing ORIF and 92% excellent with TEA. They recommend TEA in instances of arthritis, osteoporosis, or other diagnoses requiring steroids.

 

Figure A shows a significantly comminuted distal humerus fracture in an osteoporotic patient. Illustration A shows the same patient after undergoing total elbow arthroplasty.

 

Incorrect Answers:

1:Immediate range of motion is not recommended for this injury, even with the "bag of bones" treatment method. A brief period of immobilization is generally recommended for this technique. 2: Casting is not indicated for this injury. 3: ORIF of this injury will lead to worse outcomes as compared to arthroplasty. 4: Isolated radiocapitellar replacement is not indicated for this injury.

 

(SAE09TR.40) The injury shown in Figure 24 was most likely caused by what mechanism of injury? 

 

 

 

  1. Anterior posterior compression

  2. Lateral compression

     

  3. Vertical shear

     

  4. Combined mechanism

     

  5. Flexion-rotation

 

PREFERRED RESPONSE 2

 

The CT cut shows a fracture through the posterior portion of the iliac wing or a crescent fracture. This occurs after a laterally directed force is applied to the anterior part of the involved iliac wing.

(SAE11OS.63) A 10-year-old girl is seen in the emergency department after being involved in a motor vehicle accident. She has right hip pain and is unable to bear weight. She has no neurovascular deficits and no other injuries. Radiographs reveal a posterior dislocation of the right hip without apparent fracture. The acetabulum appears to be developing normally. What is the best course of treatment? Review Topic

 

  1. Open reduction under general anesthesia

     

  2. Closed reduction under general anesthesia with fluoroscopy

     

  3. Closed reduction under general anesthesia without fluoroscopy

     

  4. Conscious sedation in the emergency department and closed reduction with fluoroscopy

     

  5. Conscious sedation in the emergency department and closed reduction without fluoroscopy

 

PREFERRED RESPONSE 2

 

Hip dislocation in the pediatric population is a rare event. However, prompt recognition and rapid care for this injury is imperative to avoid future hip problems including osteonecrosis of the femoral head (a devastating problem for a pediatric patient). Reduction maneuvers can create violent impact between the posterior wall of the (intact) acetabulum and the femoral head, resulting in shearing of the proximal femoral physis and displacement of the epiphysis from the remainder of the femoral head in skeletally immature patients. Therefore, deep sedation with good muscle relaxation, such as that achieved with general anesthetic, is recommended. Reduction is best accomplished with fluoroscopy for a number of reasons, including assessment of concentricity of the hip joint after reduction, and to detect any catastrophic femoral head physeal separation that occurs during the reduction maneuver. Sedation in the emergency department is often insufficient to achieve acceptable muscle relaxation for the patient. Open reduction is only indicated if closed reduction fails completely or if the hip is not concentric after an apparently successful closed reduction

 

(SAE09FA.42) A 23-year-old woman has had a 14-month history of ankle pain after surgical treatment of multiple injuries resulting from a motor vehicle accident. Weight bearing began 4 months after surgery. The pain occurs with weight bearing and motion, but there is very little pain at rest. She has no pertinent medical history and

does not smoke. Figures 23a and 23b show current radiographs. What is the most appropriate surgical option? 

 

 

 

  1. Talectomy

  2. Revision open reduction and internal fixation (ORIF) with bone grafting

     

  3. Ankle arthrodesis

     

  4. Tibiotalocalcaneal arthrodesis

     

  5. Triple arthrodesis

 

PREFERRED RESPONSE 2

 

The radiographs reveal nonunion of a talar neck fracture. There is no radiographic evidence of osteonecrosis or significant degenerative arthritis. The results of talectomy are suboptimal. Arthrodesis would be indicated for degenerative arthritis. Revision ORIF is feasible and preserves motion. A vascularized graft should be considered whenever osteonecrosis is present, but the talar body appears viable in this case.

 

(SAE13BS.16) What role does quorum sensing play in the development of a bacterial biofilm? 

 

  1. Activates genes that produce virulence factors

     

  2. Creates planktonic bacteria

  3. Facilitates bacterial adhesion to a substrate

     

  4. Lowers antimicrobial resistance

 

PREFERRED RESPONSE 1

 

The development of a bacterial biofilm is a 2-stage process. The first step is the adhesion of individual bacteria to a substrate regulated by adhesions. After several bacteria have attached, quorum sensing (cell-to-cell communication) allows maturation of the biofilm and expression of genes that activate virulence factors. This can also increase the antibacterial resistance of the bacteria. Planktonic bacteria are individual free-moving bacteria.

 

(SAE10PE.18) A 14-year-old boy is involved in a motor vehicle accident and sustains the injury shown in Figures 18a and 18b. What is the most likely diagnosis? Review Topic

 

 

 

  1. Hawkins type I talar neck fracture

  2. Hawkins type II talar neck fracture

     

  3. Hawkins type III talar neck fracture

     

  4. Hawkins type IV talar neck fracture

     

  5. Talar body fracture

PREFERRED RESPONSE 2

 

Talar neck fractures are uncommon. In children younger than age 6 years, displacement is rare and closed treatment is usually successful in achieving union and avoiding osteonecrosis. In adolescence, however, talar neck fractures should be treated as they are in adults. This fracture is displaced, and there is dislocation of the subtalar joint. The tibiotalar and talonavicular joints remain reduced. In the classification originally created by Hawkins and modified by Canale and Kelly, this would be a Hawkins type II, carrying a 20% to 50% risk of osteonecrosis. The rate of osteonecrosis increases with the Hawkins grade. The presence of talar neck comminution and open talar neck fractures are also risk factors for osteonecrosis after talar neck fracture.

 

(SAE11UE.77) A 27-year-old man sustains an injury in a fall while downhill skiing. Two days after injury he is seen by an orthopaedic surgeon and is diagnosed with a clavicle fracture. Examination and radiographs reveal 3 cm of shortening between the fracture fragments of the midshaft clavicle fracture. The surgeon has a discussion with the patient concerning surgical versus nonsurgical treatment. With regards to results, the patient is informed that they are similar concerning which of the following? 

 

  1. Nonunion rates

     

  2. Infection

     

  3. Shoulder range of motion

     

  4. Shoulder strength

     

  5. Shoulder rotational endurance

 

PREFERRED RESPONSE 3

 

Shoulder range of motion is well maintained for both surgical and nonsurgical managment. Recent reports suggest that nonsurgical management of this fracture pattern may result in deficits of shoulder endurance and strength. Nonunion rates are significantly lower with surgical repair. Patient satisfaction, as determined by Constant scores, DASH, and patient-specific questionnaires, was higher with surgical intervention. Shoulder strength and rotational endurance are improved with surgical repair.

 

(OBQ13.96) A 25-year-old female presents to the emergency room for the fourth time in the last week. She has vague complaints of extremity pain. Physical examination by a male ER resident has been limited each visit because she is terrified of the pain that the clinician may cause. On physical examination, she is withdrawn and frightened. Regions of ecchymosis are noted throughout chest and abdomen. She has requested multiple radiographs, MRI and CT scans. Today's imaging (radiographs, MRI, CT scan) has been unrevealing. What is the most likely diagnosis? Review Topic

 

  1. Malingering

     

  2. Complex regional pain syndrome

     

  3. Anxiety disorder

     

  4. Intimate partner violence

     

  5. Fibromyalgia

 

PREFERRED RESPONSE 4

 

Based on the history and clinical presentation, the most likely diagnosis is intimate partner violence.

 

Domestic violence or intimate partner violence can be in the form of mental or physical abuse, neglect or abandonment. Close to 25% of women will experience domestic violence. Risk factors include young age (19-29 years of age), females, pregnancy and lower socioeconomic status. Affected patients will have repeated visits to the emergency room, find reasons to stay in a treatment facility for an extended period of time and constantly seek approval of their partner.

 

Shields et al. reviewed factors influence outcome in treatment of patients affected by domestic violence. They found that positive outcomes were associated with interdisciplinary approaches to management. This included better history assessment, providing written documentation regarding intervention and better access to information on community resources.

 

Illustration A is a chart documenting the frequency of female domestic violence throughout the world as of 2012.

 

Incorrect Answers

Answers 1, 2, 3, 5: These conditions are not consistent with this patient’s history.

 

(SAE11OS.199) Which of the following is the best method of initial pelvic stabilization for a patient with hemodynamic instability and the pelvic ring injury seen in Figure 199? 

 

 

 

  1. Symphyseal plating

  2. Iliosacral screw fixation

     

  3. Pelvic binder

     

  4. Pelvic C-clamp

     

  5. External fixation

 

PREFERRED RESPONSE 3

 

For a patient with an unstable pelvic ring injury and hemodyamic instability, the most appropriate initial treatment method is a pelvic sheet or binder. Symphyseal plating and iliosacral screw fixation require surgical intervention and may be appropriate following initial stabilization. External fixation and the pelvic C-clamp can be applied in the emergency setting, but usually are reserved for patients who do not respond to simpler less invasive methods initially.

(SAE11OS.124) Figures 124a and 124b are the radiographs of a 30-year-old man who sustained an ankle injury and has swelling with lateral tenderness. The patient denies any previous ankle injuries. After 6 weeks of rest and use of a removable ankle brace, he continues to have swelling, lateral pain, and popping. An anterior drawer test reveals a solid end point. Recommended treatment should include which of the following? 

 

 

 

  1. Ankle arthroscopy and debridement of an osteochondral lesion

  2. Peroneal retinacular reconstruction

     

  3. Brostrom-Gould lateral ligament reconstruction

     

  4. Immobilization in a walker boot in plantar flexion

     

  5. Ankle rehabilitation and physical therapy

 

PREFERRED RESPONSE 2

 

The radiographs and examination reveal peroneal tendon instability requiring surgical treatment for persistent symptoms and tendon instability. The radiographs demonstrate the "fleck sign," which is an avulsion of the posterior distal fibular ridge, and represents an injury to the superior peroneal retinaculum and probable peroneal dislocation. Peroneal tendon dislocations are typically present with vague lateral ankle findings associated with swelling and tenderness over the distal fibula. The tendons may be palpated as a ridge over the lateral fibula distally. Initial management of the acute injury with cast immobilization in plantar flexion/inversion with the use of a pad in the shape of a "U" or "J" is effective in 50% of patients; the rest will require surgical treatment. The indications for surgical treatment of peroneal dislocation/subluxation include continued pain and failure of nonsurgical management. Associated peroneal tendon tears can be found when performing retinacular reconstruction. Many techniques have been described including soft-tissue reconstructions, bone block procedures as well as fibular groove-deepening procedures. Radiographs do not reveal an osteochondral lesion. There is no evidence

of lateral ankle ligament instability. Ankle rehabilitation and physical therapy may further damage the unstable tendons.

 

(SAE08OS.154) Which of the following patients is more likely to have an overall poorer outcome following a lower extremity amputation? 

 

  1. A male patient with a below-the-knee amputation

     

  2. A male patient with a through-the-knee amputation performed in a zone of injury

     

  3. A male patient treated with a myoplasty at the time of wound closure

     

  4. A female patient with an above-the-knee amputation

     

  5. A female patient using a less technologically sophisticated prosthesis

 

PREFERRED RESPONSE 2

 

Many factors influence the outcome of lower extremity amputations with worse outcomes noted in patients with lower socioeconomic status, preexisting medical conditions, and low self-efficacy. Patients with amputations performed in a zone of injury, especially if this was a through-the-knee amputation, have a significantly poorer outcome than those with either above-the-knee or below-the-knee amputations. Patients with above-the-knee amputations have similar outcomes to those with below-the-knee amputations, although those with below-the-knee amputations have faster self-selected walking speeds. Gender and sophistication of the prosthesis have no bearing on outcome. The type of muscle anchoring technique used also plays no role in outcome.

 

(OBQ13.191) A 42-year-old woman complains of ankle pain with weightbearing for the last 2 years. She recalls spraining her ankle more than 10 years ago. She plays tennis and regularly walks 5 miles a day for exercise, but has had to give up these activities over the last few months because of pain. Examination reveals limitation in ankle dorsi- and plantar flexion. A course of non-operative management has been

unsuccessful. Which of the following options will most likely provide pain relief and allow her to return to her previous activity level? 

 

 

 

  1. Arthroscopic debridement

  2. Supramalleolar osteotomy

     

  3. Talar resurfacing

     

  4. Total ankle replacement

     

  5. Ankle arthrodesis

 

PREFERRED RESPONSE 2

 

This young, active patient has ankle valgus following previous trauma. A corrective supramalleolar osteotomy of the tibia will provide pain relief and improve range of motion, allowing return to sports.

 

Supramalleolar osteotomy may be performed for varus (medial opening wedge or lateral closing wedge) or valgus (lateral opening wedge or medial closing wedge) ankles. They are suited for near-normal ROM minimal talar-tilt or varus heel alignment, asymmetric ankle arthritis (confined to medial or lateral compartment; corresponding to Takakura Stage II or IIIA for medial ankle arthritis).

 

Pagenstert et al. looked at realignment surgery for posttraumatic arthritis in 35 patients. For valgus ankles, they performed 18 medial closing wedge and 1 lateral opening wedge osteotomies. For varus ankles, they performed 7 medial opening wedge and 4 lateral closing wedge osteotomies. There was improvement in pain (7/10 to 3/10), ROM (33° to 38°) and Takakura score (2.3 to 1.3).

 

Lee et al. described supramalleolar osteotomy for medial arthritis in 16 patients. There was improvement in AOFAS score (62 to 82), Takakura stage (2.9 to 2.3), tibial-anterior surface angle (85 to 100°). Patients with low postoperative talar tilt (TT) had better clinical and radiographic results than those with high TT. Greater postoperative heel valgus predicted for postoperative subfibular pain.

Figure A shows valgus alignment at the ankle. Illustration A shows the same ankle following medial closing wedge supramalleolar osteotomy. Illustration B is a table showing the Takakura classification. Illustration C demonstrates correction of the tibial-anterior surface angle (TAS) following supramalleolar osteotomy.

 

Incorrect Answers:

Answer 1: Arthroscopic debridement will not correct alignment and is not indicated. Answer 3: Talar resurfacing is a new technique without a proven track record and is not standard of care at this time. Answers 4 and 5: TAR and arthrodesis are indicated for arthritis involving the entire joint (Takakura IIIB or IV)

 

(OBQ14.196) A 21-year-old male is brought to the emergency department after being involved in gang-related violence. A radiograph of his pelvis is shown in Figure A. The patient is hemodynamically stable. Which of the following imaging modalities is the next best step in evaluating this patient for the most common associated injury? Review Topic

 

 

 

  1. Ultrasound bladder to exclude bladder perforation

  2. CT abdomen to exclude bowel perforation

     

  3. MRI pelvis to exclude genital injuries

     

  4. CT angiogram exclude laceration of major vessels

     

  5. CT acetabulum to exclude intra-articular foreign body

PREFERRED RESPONSE 2

 

Low velocity gunshot wounds (GSW) to the hip are most commonly associated with bowel perforation. Consultation with general surgery (or in some facilities, trauma surgery) is necessary to exclude this.

 

The incidence of GSW is increasing and it is the 2nd leading cause of death in young males in the US after motor vehicle accidents. The incidence of a GSW to the buttock is approximately 8% of all GSW to the extremities. Potential complications of pelvic and acetabular GSW include septic arthritis, enterocutaneous, enteroacetabular, and vesicoacetabular fistulas, infected nonunion, malunion, and injuries to the iliac vessels. The presence or absence of intra-abdominal injuries affects treatment and outcome.

 

Bartkiw et al. reviewed 2808 GSW and found 1235 associated fractures including 42 fractures of the hip and pelvis. Ten orthopaedic operative procedures were performed in 7 patients. Associated nonorthopaedic injuries included 15 small/large bowel perforations (36%), 7 vessel lacerations (17%), and 2 urogenital injuries (5%) that required surgery.

 

Najibi et al. reviewed 39 GSW to acetabulum. They found 32 simple and 7 associated fracture patterns. The most common simple and associated patterns were anterior column and both column, respectively. Bowel injuries were the most common associated injures (42%). Predictors of poor outcome include high-velocity missile, involvement of acetabular dome, abdominal injury, nerve injury, vascular injury, and male gender. Deep infection was associated with primary anastomosis of bowel injury and an associated fracture pattern.

 

Figure A shows a GSW to the right hip with acetabular fracture and visible bullet fragment.

 

Incorrect Answers:

Answers 1, 3, 4: Urogenital and vascular injuries are not as common as bowel injuries.

Answer 5: Intraarticular bullet fragments should be removed to prevent lead toxicity (plumbism), which arises from fragments dissolving in synovial fluid. Other reasons for removal include prevention of third body wear, joint sepsis and lead arthropathy. It is not the priority in this case and exclusion of bowel injury must take precedence.

(OBQ14.14) A 39-year-old female presents with the following motor vehicle crash with the injury seen in Figure A (immobilized in a pelvic binder). The iatrogenic neurologic injury most commonly caused by placement of the anterior construct for this injury, as shown in Figure B, would cause which of the following? 

 

 

 

  1. Weakness of hip flexion

  2. Weakness of ankle dorsiflexion

     

  3. Numbness of the medial thigh

     

  4. Numbness of the lateral thigh

     

  5. Numbness of the perineum

 

PREFERRED RESPONSE 4

 

This patient was treated with posterior stabilization, and an anterior subcutaneous internal fixator (ASIF). The most common neurologic injury seen following placement of the ASIF construct is irritation of the lateral femoral cutaneous nerve (LFCN), causing numbness and/or pain of the lateral thigh.

 

Unstable pelvic fractures can be treated in a multitude of ways. The ASIF construct is typically created by placing long pelvic screws or polyaxial pedicle screws in the supraacetabular region, similar to the supraacetabular pins for an anterior external fixator. Then a curved bar is placed subcutaneously and connected to the supraacetabular screws. They are typically removed after 3-4 months when fracture healing is complete.

 

Vaidya et al. present a retrospective review of the use of ASIF as definitive treatment of unstable pelvic fractures. All patients in the study tolerated the construct well. LFCN irritation was seen in 30% of patients, and resolved in all but one patient.

 

Müller et al. present a retrospective review of the use of posterior stabilization and ASIF. They report an acceptably low complication rate, and good to excellent outcomes in 64.5% of patients.

 

Figure A is a radiograph demonstrating a right APC3 and left APC2 pelvic injury, imaged in a pelvic binder. Figure B is a postoperative radiograph following posterior stabilization and ASIF.

Incorrect answers:

Answer 1: Weakness of hip flexion would be due to injury to the femoral nerve, which is not commonly associated with this construct. Answer 2: Weakness of ankle dorsiflexion would be due to injury to the L5 nerve root, which does not occur due to ASIF placement. Answer 3: Numbness of the medial thigh is due to injury to the obturator nerve, which is not commonly associated with this construct. Answer 5: Numbness of the perineum is due to injury to the sacral nerve roots, which is not commonly associated with this construct.

 

(OBQ14.32) A 72-year-old female sustains a displaced intracapsular femoral neck fracture. Which of the following is TRUE regarding the long term differences between possible treatment options for this injury? 

 

  1. Patients undergoing total hip arthroplasty are more likely to experience persistent pain than those undergoing internal fixation

     

  2. Patients undergoing total hip arthroplasty are less likely to require reoperation than those undergoing internal fixation

     

  3. There is no difference in functional outcome scores between internal fixation and total hip arthroplasty

     

  4. Patients undergoing internal fixation perform activities of daily living better than those undergoing total hip arthroplasty

     

  5. Mortality rates are higher following total hip arthroplasty than internal fixation

 

PREFERRED RESPONSE 2

 

Elderly patients with femoral neck fractures (FNF) undergoing total hip arthroplasty (THA) are less likely to require reoperation than those undergoing internal fixation.

 

Intracapsular FNF are common in elderly patients after a fall from standing height. Treatment depends on physiological age and displacement (Garden's classification). For displaced fractures, physiologically young patients are treated with internal fixation while physiologically old patients are treated with either hemiarthroplasty (debilitated, less active patients) or THA (more active patients, those with acetabular

disease or preexisting inflammatory arthritis).

 

Chammout et al. retrospectively compared the long term (17 years) results of THA (cemented both component) and ORIF (2 cannulated screws) in elderly patients (>65 years). They found no difference in mortality. But hip scores were higher and pain was better in the THA group, while reoperation rates were higher in the ORIF group. Walking speed was initially faster in the THA group, but later did not differ between groups. They recommend THA for elderly patients with displaced FNF.

 

Rogmark et al. prospectively compared closed reduction and internal fixation (CRIF) with arthroplasty (combining hemiarthroplasty and THA) at 2 years in elderly patients (>70 years). Failure rates were higher, pain was worse, and walking was more impaired after CRIF. They recommend arthroplasty for patients >70 with FNF.

 

(SAE12TR.104) A 22-year-old woman injured her ankle when she fell off a ladder. Radiographs reveal a displaced large posterior malleolus fracture of about 45% of the joint. What is the best definitive treatment? 

 

  1. Open reduction and internal fixation with absolute stability

     

  2. Open reduction and internal fixation with relative stability

     

  3. Closed reduction with casting

     

  4. Reamed locked intramedullary nailing

     

  5. External fixation

 

PREFERRED RESPONSE 1

 

Basic understanding of fracture care requires a fundamental knowledge of the principles regarding absolute and relative stability. Compression plating and anatomic reduction of articular fractures are examples of absolute stability. Bridge plating, external fixation, casting, and intramedullary nailing are all examples of relative stability. Both bone forearm fractures have long been treated with open reduction and internal fixation even in the light of open wounds. Results have been excellent with plate fixation. Recently, intramedullary nails that are contoured and locked have been used in the treatment of both bone forearm fractures, but they are not reamed. It is well established that with restoring the proper radial bow, length, and alignment,

optimal function can be achieved. Open reduction and internal fixation allows this achievement. In cases where comminution exists, absolute stability may have to be sacrificed so as to not strip small comminuted bone fragments. Therefore, a bridging technique is worthwhile. External fixation can be used as a temporary technique until the soft tissues are more amenable to definitive fixation. Cast treatment is not indicated in adult forearm fractures. Locking nails for forearm use are not reamed. With regards to articular fractures, anatomic reduction and rigid stabilization are required to achieve the best results and allow for fracture healing. This environment also allows for the best chance of the cartilage repair process to form "hyaline-like" cartilage. Open reduction and internal fixation with absolute stability is the mainstay of treatment for partial articular fractures such as split depression tibial plateau fractures and posterior malleolus fractures involving greater than about 25% to 30% of the joint. The gold standard for the treatment of a closed femur fracture is a reamed intramedullary locked nail. Results are uniformly excellent. This can be done without stripping of the soft tissues such as in open reduction and internal fixation. External fixation can be used as a temporary device in patients in extremis for damage control reasons.Moed BR, Kellam JF, Foster RJ, Tile M, Hansen ST Jr. Immediate internal fixation of open fractures of the diaphysis of the forearm. J Bone Joint Surg Am. 1986 Sep;68(7):1008-17. http://www.ncbi.nlm.nih.gov/pubmed/3745238')">View Abstract at PubMedJones DB Jr, Kakar S. Adult diaphyseal forearm fractures: intramedullary nail versus plate fixation. J Hand Surg Am. 2011 Jul;36(7):1216-9. Epub 2011 May

14. http://www.ncbi.nlm.nih.gov/pubmed/21571446')">View Abstract at PubMed

 

(SAE09TR.2) Which of the following choices best describes the fracture pattern shown in Figures 2a through 2c? 

 

 

 

  1. Anterior column

  2. Anterior wall

     

  3. Posterior column

     

  4. Both column

     

  5. Transverse

 

PREFERRED RESPONSE 3

 

The fracture pattern shown in the radiographs is a fracture of the posterior column. The only line interrupted on the AP pelvis is the ilioischial line. The obturator oblique view shows that the iliopectineal line is intact as is the outline of the posterior wall. The iliac oblique view shows an interruption of the ilioischial line and an intact anterior wall. Therefore, this fracture is a fracture of the posterior column.

 

(OBQ13.216) A 32-year-old professional skydiver lands awkwardly during a jump. He presents to the emergency room with bilateral knee injuries. Following successful closed reduction of both extremities, both feet are warm and pulses are present. Bedside doppler assessment is performed and the results are seen in Figure A. What would be the most appropriate next step in treatment? 

 

 

 

  1. Long-leg splinting of bilateral lower extremities, monitoring of bilateral pedal pulses for 48 hours

  2. Intravenous dextran administration, repeat doppler evaluation at 6 hourly intervals

     

  3. Perform CT angiography for bilateral lower extremities

     

  4. Perform CT angiography for the left lower extremity, monitor right pedal pulses for 48 hours

  5. Surgical exploration of bilateral lower extremities

 

PREFERRED RESPONSE 4

 

The ankle-brachial index (ABI) of left and right limbs are 0.78 and 0.96 respectively. CT angiography (or conventional angiography) of the left lower extremity is indicated because of abnormal ABI <0.9. It is sufficient to observe the right lower extremity (ABI >0.9).

 

Injury to the popliteal artery is present in 10-40% of knee dislocations (KD). ABI

<0.9 has sensitivity of 87% and specificity of 97% for the diagnosis of arterial disruption. Delayed recognition of an occlusive injury (>8 hours) is likely to result in above knee amputation.

 

Stannard et al. examined the role of selective arteriography based on serial physical examination. They found arterial injury in 7% (9 out of 134 knees), and abnormal physical findings in 10 patients, with only 1 false positive. They recommend arteriography for patients with decreased pulses, color or temperature, expanding knee hematoma, or an abnormal physical examination prior to presentation in the emergency department.

 

Mills et al. reviewed the value of ABI for diagnosing arterial injury after knee dislocation. They found that of the 29% with ABI <0.9, all required surgery for arterial injury. Of the remaining patients with ABI >0.9, none had vascular injury. They concluded ABI >0.9 has negative predictive value of 100%.

 

Nicandri et al. reviewed an algorithm for selective angiography. They recommend the following: (1) Intact pulses and ABI >0.9, observation for 24 hours. (2) Asymmetric pulses or ABI <0.9, arteriogram. (3) Hard signs of vascular injury (absent distal pulses, distal ischemia, active hemorrhage, expanding pulsatile hematoma), surgical exploration.

 

Figure A is a table showing systolic pressure readings at different sites. To calculate the ABI, the highest measured arterial pressure in the ankle or foot is divided by the higher brachial arterial pressure from both upper extremities.

 

Incorrect Answers:

Answer 1: If vascular exam is normal and ABI >0.9, neurovascular checks every 2-4 hours for 48 hours is appropriate. As ABI is abnormal on the left, CT angiogram is necessary.

Answer 2: Anticoagulation has not been shown to stop thrombus formation for non-flow limiting intimal flap tears. Answer 3: CT angiography is indicated if ABI is abnormal post reduction. CT angiography for the right lower extremity is unnecessary as ABI >0.9. Answer 5: Surgery is indicated if CT angiography (or conventional angiography) demonstrates intimal flap tears that are flow limiting (but not if the intimal flap is NOT flow limiting), if distal pulses are absent, or if distal ischemia, active hemorrhage, or expanding pulsatile hematoma are present.

 

(OBQ15.82) In a statement put forth by AAOS, the role of the orthopaedic surgeon in the face of domestic and family abuse includes all of the following EXCEPT: Review Topic

 

  1. Be aware that he or she may be the first physician to be caring for the victims

     

  2. Ensure that they maintain comprehensive and accurate medical records documenting the events and examinations

     

  3. Care should resume once the patient's social situation is evaluated and assessed

     

  4. Transfer an elderly victim who is in immediate danger to a hospital emergency department and notify the emergency department physician of the transfer and the reasons for your concern

     

  5. Advocate for appropriate legislation and public policy

 

PREFERRED RESPONSE 3

 

Regardless of the social situation, the orthopaedic surgeon's primary role is to proceed and offer expeditious and appropriate care for the patient's injuries.

 

The statement put forth by the AAOS implores the orthopaedic surgeon to be aware of the integral components to identify, document, and care for minors, elders, and/or partners who are victims of domestic abuse.

 

Zilmer et al. in a JAAOS review and the basis for the AAOS statement emphasizes the ability to identify abuse, which includes, but is not limited to frequent/multiple injuries, temporal abnormalities in multiple injuries, frequent visits/utilization of the emergency department, and/or unusual injuries/fracture patterns not consistent with the clinical picture. Meticulous documentation is of paramount importance, in addition to communicating your concerns to the appropriate emergency department personnel.

 

Incorrect answers:

Answers 1, 2, 4 and 5: All are expected roles of the orthopaedic surgeon in the face of domestic and/or family abuse.

 

(SAE08UE.74) An adult patient has a closed humeral fracture that was treated nonsurgically and a concomitant radial nerve injury. Six weeks after injury, electromyography shows no evidence of recovery. Management should now consist of 

 

  1. exploration and neurolysis/repair.

     

  2. MRI of the arm.

     

  3. functional electrical stimulation.

     

  4. radial nerve tendon transfers.

     

  5. observation.

 

PREFERRED RESPONSE 5

 

In patients with radial nerve injuries with closed humeral fractures, it has been reported that 85% to 95% spontaneously recover. Based on this premise, most surgeons favor expectant management of these injuries. Even if there is no evidence of recovery at 6 weeks, repeat electromyography at 12 weeks is advocated. If there is no clinical or electromyographic signs of recovery at 6 months, exploration is recommended. If the nerve is in continuity at the time of exploration, nerve action potentials are useful in helping determine the need for neurolysis, excision, and grafting, or if excision and repair is the best option.

 

(OBQ15.78) A 70-year-old male with longstanding diabetic neuropathy sustains a fall down a flight of stairs and sustains the injury shown in Figures A and B. In the operating room, direct reduction of the fracture is performed. The syndesmosis is assessed and found to be intact. The fibula is fixed with a small fragment locking plate and the medial malleolus is fixed with screws. What is the next best step? Review Topic

 

 

  1. No syndesmotic fixation. Immediate touch-down weightbearing in CAM walker boot.

  2. No syndesmotic fixation. Non-weightbearing for 4 to 6 weeks, followed by progressive weightbearing in CAM walker boot.

     

  3. No syndesmotic fixation. Non-weightbearing for 8 to 12 weeks.

     

  4. Syndesmotic screws. Non-weightbearing for 4 to 8 weeks.

     

  5. Syndesmotic screws. Non-weightbearing for 8 to 12 weeks.

 

PREFERRED RESPONSE 5

 

Ankle fractures in diabetics with neuropathy should be treated with enhanced fixation comprising stiff plates with syndesmotic screws, even in the absence of syndesmotic injury. Weightbearing should be delayed for 8 to 12 weeks after surgery, rather than 4-8 weeks (as for normal patients).

 

Diabetics with ankle fractures are prone to complications. Nonoperatively treated cases have up to a 50% incidence of skin breakdown in a cast. Surgically treated patients have up to 40% complication rate. Supplemental fixation can include include multiple syndesmotic screws, spanning external fixation, tibio-talar Steinmann pins, more rigid fibular plates, supplemental intramedulary fibular pinning. Weightbearing after surgery should be delayed (8-12weeks).

 

McCormack and Leith reviewed the complications in 26 diabetic ankle fractures. 19 patients had surgery. The incidence of complications was 42%. Two required amputation and died. None of the non-diabetic control group had complications. They concluded that in the older, diabetic patient with lower demands, especially if insulin-dependent, it may be preferable to accept a loss of reduction and malunion rather than risk the potentially devastating complications associated with operative intervention. If surgery is necessary, the results of this study provides a more accurate prognosis and allows for better discussion of surgical risks with the patient.

 

Roseunbaum et al polled AOFAS members on the treatment of diabetic ankle fractures. (1) For nondisplaced bimalleolar fracture with diabetic neuropathy, respondents preferred casting and nonweightbearing (NWB) for 8-12 weeks. (2) For Weber B fracture dislocation without neuropathy, without syndesmotic injury,

respondents preferred either 1/3 tubular or small fragment locking plates with syndesmotic screws and NWB for 8-12 weeks. (3) For bimalleolar fracture dislocation with neuropathy, without syndesmotic injury, respondents preferred small fragment locking plates with syndesmotic screws and NWB for 8-12 weeks.

 

Wukich and Kline reviewed the management of ankle fractures in diabetics. They found that neuropathy is more prevalent in patients with ankle fractures than without. They stress that 1% reduction in HbA1C results in 30% reduction in complication rate. Fracture healing is slower, callus is smaller, with less stiffness, tensile strength and collagen content. Amputation rate is up to 5% for patients treated operatively or nonoperatively. Complications are higher in patients with vasculopathy, neuropathy, or Charcot arthropathy.

 

Incorrect Answers:

Answers 1, 2, 3: Supplemental syndesmotic fixation will increase construct rigidity. Answer 4: Extended NWB should be enforced after ORIF of diabetic ankle fractures (8-12wk).

 

(OBQ13.232) A young gymnast fell awkwardly onto an outstretched hand during a competition. At the time of impact, his forearm was positioned in supination. Axial and posterolateral forces were loaded along the forearm into the elbow and the elbow underwent a significant valgus thrust. What injury pattern is most likely to result from the combination of these forces at the elbow? 

 

  1. Extension-type supracondylar fracture

     

  2. Flexion-type supracondylar fracture

     

  3. Anterior olecranon fracture dislocation

     

  4. Coronoid fracture, olecranon fracture and elbow dislocation

     

  5. Coronoid fracture, radial head fracture and elbow dislocation

 

PREFERRED RESPONSE 5

 

The combination of valgus, axial, and posterolateral rotatory forces (forearm supination) can result in a "terrible triad" injury of the elbow.

The key features of a terrible triad injury include a radial head fracture, coronoid fracture, and dislocation of the elbow. Disruption of the lateral collateral ligament complex often concomitantly occurs. While restoration of the bony anatomy is important for static stability, the key primary stabilizer that needs to be addressed is the lateral collateral ligament complex. In acute injuries LCL repair may be possible. In chronic injury, LCL reconstruction would need to be considered.

 

O'Driscoll et al. 1991, examined 5 patients with recurrent posterolateral rotatory instability of the elbow. They showed that by applying supination of the forearm with a valgus moment and an axial compression force to the elbow while it is flexed from full extension, this can demonstrate posterolateral rotatory instability of the elbow. The elbow is reduced in full extension and must be subluxated as it is flexed in order to obtain a positive test result (a sudden reduction of the subluxation).

 

O'Driscoll et al. 1992 looked at a cadaveric study of the elbow. They showed that external rotation and valgus moments with axial forces resulted in posterior dislocations in 12 of the 13 specimens when the anterior medial collateral ligament (AMCL) remained intact. Clinically, it valgus stability in pronation is demonstrated, the AMCL can be assumed to be intact.

 

Illustration A and B shows radiographs of a terrible triad injury. There is posterolateral dislocation of the elbow with associated radial head fracture, coronoid fracture.

 

Incorrect Answers:

Answer A: Extension-type supracondylar fractures typically occur with FOOSH injuries with posteriorly directed forces. Answer B: Flexion-type supracondylar fractures typically occur with the elbow in flexion with an anterior directed force. Answer C: Anterior olecranon fracture dislocation typically occurs with a direct high-energy blow that is applied to the posterior aspect of the forearm with the elbow in 90° flexion.

Answer D: Coronoid fracture, olecranon fracture and lateral collateral ligament injury typically occur with the arm positioned in supination, axial and VARUS moment forces.

 

(SAE12TR.45) Which of the following is associated with increased fetal morbidity and mortality in acetabular fractures during pregnancy? 

 

  1. Fetal position

     

  2. Surgical approach

  3. Mechanism of injury

     

  4. Fracture classification

     

  5. Trimester of pregnancy

 

PREFERRED RESPONSE 3

 

Fixation of pelvic and acetabular fractures in pregnancy is not contraindicated. However, both maternal and fetal morbidity and mortality is increased in this patient subset. Factors shown to be associated with increased fetal mortality include: injury severity, mechanism of injury, and maternal hemorrhage. Surgical approach, fracture classification, fetal position, and the trimester of pregnancy have not been shown to affect fetal morbidity or mortality.

 

(OBQ15.114) A 30 year-old male is involved in a motor vehicle collision and sustains a scapular fracture. In patients with scapular fractures, what other fracture is MOST commonly observed? 

 

  1. Spine fracture

     

  2. Rib fracture

     

  3. Clavicle fracture

     

  4. Humerus fracture

     

  5. Pelvic fracture

 

PREFERRED RESPONSE 2

 

Rib fractures are the most commonly observed fractures associated with scapular fractures.

 

Scapular fractures are associated with high-energy trauma, with motor vehicle collisions being a common mechanism of injury. These fractures are also associated with increased Injury Severity Scores and therefore clinicians need to be cognizant of concomitant injuries. In addition to associated orthopaedic injuries, pulmonary injuries (contusions, hemothorax, and pneumothorax) and head injuries are associated with scapular fractures. A CT chest should be considered in patients with scapula

fractures, to best visualize associated rib fractures and pulmonary injuries.

 

Baldwin et al. conducted a retrospective case control analysis using the US National Trauma Database that included 9,453 scapular fractures. In their study, the most commonly associated fractures were rib fractures (52.9%), followed by fractures of the spine (29.2%), clavicle (25.2%), and pelvis (15.3%). Lung and head injuries occurred in 47.1% and 39.1% of the cases, respectively.

 

Incorrect Answers:

Answers 1, 3, 4, 5: Although these fractures do occur in association with scapular fractures, Baldwin et al demonstrated that they do not occur as frequently as rib fractures.

 

(SBQ12TR.107) An 87-year-old female sustains the injury shown in Figure A after a fall from standing. At baseline, she ambulates with a walker in her home and lives with a 24-hour home health aide. She has a past medical history of stroke and mild dementia. Following medical optimization, what is the most appropriate treatment modality? 

 

 

 

  1. Intramedullary nail

  2. Hemiarthroplasty

     

  3. Closed reduction percutaneous pinning

  4. Total hip arthroplasty

     

  5. Sliding hip screw

 

PREFERRED RESPONSE 2

 

At baseline, with minimal ambulatory status and dependence on a 24-hour home health aide, the best treatment choice is hemiarthroplasty.

 

Arthroplasty is the gold standard for displaced femoral neck fractures in the elderly. In community ambulators with relative independent lives, total hip arthroplasty (THA) is recommended. For minimal ambulator, hemiarthroplasty is recommended.

 

van den Bekerom et al. randomized 252 patients over 70-years-old to either THA or hemiarthroplasty for displaced femoral neck fractures. With increased blood loss, operative time and dislocation risk, they did not recommend THA unless there was good preoperative ambulatory status, and/or pre-existing osteoarthritis or rheumatoid arthritis.

Figure A exhibits a displaced femoral neck fracture Incorrect answers:

Answers 1, 3, 5 are not the treatment standards for displaced femoral neck fractures in the elderly.

Answer 4 is incorrect because the patient is minimally ambulatory at baseline with little independence.

 

(SAE11OS.67) The radiograph seen in Figure 67 reveals an ankle fracture in a 65-year-old woman who slipped on the ice. She has a history of diabetes mellitus for the past 7 years and reports that she maintains fair control of her diabetes; her last HgbA1c was 8%. The patient is a community ambulatory who lives independently. Examination reveals she has absent sensation with the 5.07 monofilament. When determining management, the physician must consider which of the following? 

 

 

  1. Supplemental internal fixation

  2. Primary ankle arthrodesis

     

  3. Nonsurgical treatment to avoid infection

     

  4. Early bone grafting because of poor bone quality

     

  5. Early mobilization and weight bearing to minimize stiffness

 

PREFERRED RESPONSE 1

 

Increased immobilization and delayed weight bearing are indicated in the neuropathic population after treating an ankle fracture. Patients with diabetes mellitus and peripheral neuropathy have higher complication rates following ankle fractures treated surgically or nonsurgically. The elevated HgbA1c and neuropathy both predict a higher complication rate with this fracture. Outcomes after nonsurgical management of this fracture are poorer than after surgical treatment. Early bone grafting is not recommended in closed fractures, but the use of supplemental internal fixation is recommended because of the high risk of nonunion. More substantial constructs with supplemental fixation, locking fixation, fixation through the calcaneus and talus into the tibia, or external fixation are necessary. Primary arthrodesis is not recommended in this fracture pattern or in a relatively active patient.

(SAE09FA.95) A 59-year-old woman underwent open reduction and internal fixation (ORIF) of her ankle 6 months ago, with subsequent hardware removal 3 months later. She now reports persistent, diffuse ankle pain, swelling, and limited range of motion. Figure 48 shows an oblique radiograph of the ankle. What is the next most appropriate step in management? 

 

 

 

  1. Physical therapy

  2. Hardware removal

     

  3. Repeat placement of a syndesmotic screw

     

  4. Deltoid ligament reconstruction

     

  5. Revision ORIF with exploration of the syndesmosis and medial ankle

 

PREFERRED RESPONSE 5

 

The radiographs demonstrate persistent widening of the medial clear space with an ossicle. This represents soft-tissue interposition-scar tissue, the deltoid ligament, or the posterior tibialis tendon. Physical therapy will not improve the symptomatic malalignment. Hardware removal would be indicated for pain localized to the lateral fibula. Repeat syndesmotic screw fixation alone will not reduce the malalignment. Deltoid ligament repair may be necessary but will need to be combined with debridement of the medial ankle and syndesmosis, as well as repeat placement of one or more syndesmotic screws to maintain the reduction.

(SAE12TR.38) When compared with reamed intramedullary nailing for an unstable diaphyseal tibia fracture, unreamed nailing is associated with which of the following? 

 

  1. Longer surgical times

     

  2. Higher infection rates

     

  3. Lower functional outcome scores

     

  4. Similar union rates in open fractures

     

  5. Higher incidence of pulmonary complications

 

PREFERRED RESPONSE 4

 

The Investigators Randomized Trial of Reamed versus Non-Reamed Intramedullary Nailing of Tibial Shaft Fractures (SPRINT) study, a large, randomized, controlled trial, has shown a benefit of reamed intramedullary (IM) nailing versus unreamed IM nailing for closed tibial shaft fractures with regard to reoperation rates. No such association exists for open tibial fractures; ie, union rates are the same for open fractures. The infection rates are the same, as is functional outcome, and surgical time is potentially shorter for unreamed nails. The potential pulmonary benefits from unreamed nailing have never been clinically proven.

 

(SAE08OS.29) Locked lateral plate fixation of the proximal tibia is most appropriate for what Schatzker type fracture? 

 

  1. I - lateral plateau split

     

  2. II - lateral plateau split/depression

     

  3. III - lateral plateau depression

     

  4. IV - medial plateau fracture

     

  5. V - bicondylar tibial plateau fracture

PREFERRED RESPONSE 5

 

Locked lateral plate fixation of the proximal tibia has proven effective for bicondylar fractures (Schatzker types V and VI). The laterally based plate provides support for the lateral condyle fracture and locked proximal screws support the medial fragment from varus collapse. A medial antiglide plate can be used in conjunction with the locked plate. Lateral locked plate fixation has not been shown to have advantages for pure lateral fractures (Schatzker types I, II, and III) that require buttress plating or support of depressed articular components. Pure medial fractures (Schatzker type IV) are best treated with medial fixation.

 

(OBQ13.268) An 80-year-old patient presents 8 months postoperatively with right groin pain. Examination reveals a leg length discrepancy of 1.5cm. Recent radiographs are seen in Figures A and B. What is the most appropriate treatment plan? 

 

 

 

  1. Nail dynamization

  2. Hardware removal, correction of alignment with a Taylor spatial frame, insertion of bone autograft

     

  3. Exchange unreamed nailing with a longer, larger implant

     

  4. Augmentative plate fixation without nail removal, insertion of bone autograft

  5. Hardware removal, correction of alignment, plate fixation, insertion of bone autograft

 

PREFERRED RESPONSE 5

 

This patient has atrophic non-union (NU) and varus collapse following cephalomedullary nailing of a subtrochanteric fracture. The ideal treatment involves nail removal, correction of alignment, fracture fixation, and bone grafting. Fixation can be achieved with a nail or plate.

 

Subtrochanteric fractures can be treated with cephalomedullary nailing or fixed angle plates. Nailing of these fractures is technically challenging because the fracture must be reduced prior to nail passage. Failure to do so leads to varus and procurvatum malreduction.

 

Bellabarba et al. reviewed plating of femoral nonunions after intramedullary nailing. Of 23 nonunions, 21 healed at an average of 12 weeks. The remaining 2 cases required repeat plating (at 2 and 8 weeks) for hardware breakage because of noncompliance with weightbearing restrictions. They advocate plating because it allows for correction of malalignment and provides a biomechanically superior tension band construct.

 

Incorrect Answers:

Answer 1:Dynamization is less effective in femoral NU compared with tibial NU. It is suitable for NU of axially stable fractures without significant angular deformity, particularly if statically locked in distraction. It is not suitable in this case because of varus collapse and shortening. Answer 2: The use of tensioned wire fixators is poorly tolerated in the femur and has a high complication rate. Answer 3: Reamed exchange nailing is the procedure of choice after IM nailing in the ABSENCE of femoral deformity. It is less effective in this case of varus collapse and shortening. Exchange unreamed nailing is not useful because (1) the new nail follows the same path as the old, making deformity correction impossible, and (2) the new nail cannot be larger than the old nail. A larger implant is only possible following reaming, especially if the isthmus is to be crossed. Answer 4: Plating around a nail is inadvisable in this case because the existing hardware precludes deformity correction.

 

(SBQ12TR.48) A 28-year-old male college student sustains a severe foot injury from gunshot-related violence, and subsequently undergoes a lower-extremity amputation

as shown in Figure A. At long-term follow-up, which of the following is the strongest predictor of patient satisfaction as related to his injury? 

 

 

 

  1. Age less than 30

  2. Marijuana use

     

  3. Use of negative pressure wound therapy

     

  4. Male gender

     

  5. Ability to return to work

 

PREFERRED RESPONSE 5

 

The strongest factor to predict patient-reported outcomes after trauma-related lower extremity amputations is the patient's ability to return to work. This is likely due to the effect of the return to work on the physical, emotional, and financial aspects of the patient's life.

 

The LEAP study is a multicenter, prospective study evaluating multiple aspects of reconstruction versus amputation in the treatment of mangled extremity injuries. With regard to patient satisfaction, treatment variables such as decision for reconstruction versus amputation, or initial presence or absence of plantar sensation have little impact. In addition, demographic factors such as age, gender, socioeconomic status, and education level do not predict patient satisfaction. Instead, the most important predictors of patient satisfaction at 2 years after injury include the ability to return to work, absence of depression, faster walking speed, and decreased pain.

 

O'Toole et al reviewed 463 patients treated for limb-threatening lower-extremity injuries and identified factors associated with patient reported outcomes two years after surgery. They found that return to work was the most associated with outcomes, but that physical functioning, walking speed, pain levels, and presence of depression were also associated to a lesser extent with outcomes.

 

Bosse et al performed a multicenter, prospective study to assess outcomes of 569 patients with severe lower extremity limb injuries that resulted in either amputation or

limb salvage procedures. They found that at two years postoperatively, no significant differences were seen between groups in patient-reported outcome. Worse outcomes were associated with rehospitalization for a major complication, a low educational level, nonwhite race, poverty, lack of private health insurance, poor social-support network, low self-efficacy (the patient's confidence in being able to resume life activities), and involvement in disability-compensation litigation.

 

Figure A shows a clinical photograph of a Pirigoff amputation at early follow-up. This amputation is an end-bearing amputation that utilizes the plantar heel pad for weightbearing, and relies on a tibiocalcaneal arthrodesis.

 

Incorrect Answers:

1-4: These options are not as strong of a factor of patient satisfaction in long-term follow up after trauma-induced lower extremity amputation.

 

(SAE09TR.19) Following fixation of a displaced intra-articular fracture of the distal humerus through a posterior approach, what is the expected outcome? 

 

  1. Development of arthritic changes at 1 year

     

  2. Restoration of full elbow range of motion

     

  3. Loss of approximately 25% of elbow flexion strength

     

  4. Posterolateral rotatory instability

     

  5. Olecranon nonunion

 

PREFERRED RESPONSE 3

 

Following repair of a displaced intra-articular distal humerus fracture, the ability to regain full elbow range of motion is rare. Recent reports of olecranon osteotomy have yielded healing rates of between 95% to 100%. According to McKee and associates, patients can be expected to have residual loss of elbow flexion strength of 25%.

 

(SAE11UE.9) A 56-year-old man who tripped and fell out of his golf cart onto his right shoulder 4 days ago now reports mild pain while chipping. Examination reveals mild bruising over the lateral clavicle but good shoulder range of motion and strength. A radiograph is shown in Figure 9. Appropriate treatment at this time should include which of the following? 

 

 

 

  1. Intramedullary pinning

  2. Bone stimulator

     

  3. Sling for comfort, followed by gentle range-of-motion exercises

     

  4. Open reduction and internal fixation with a plate and screws

     

  5. Arthroscopic distal clavicle resection

 

PREFERRED RESPONSE 3

 

Treatment of this minimally displaced distal clavicle fracture should begin with nonsurgical management consisting of sling therapy followed by gentle motion therapy. Any form of surgical intervention at this time is unnecessary because this fracture pattern has a high incidence of union. A bone stimulator may be used if healing becomes delayed.

(SAE11OS.12) A 22-year-old woman sustains the injury seen in Figure 12 as a result of a motor vehicle crash. What factor is most closely associated with development of osteonecrosis? 

 

 

 

  1. Reduction quality

  2. Time from injury to surgery

     

  3. Presence or absence of a capsulotomy

     

  4. Type of implant used for internal fixation

     

  5. Location of the fracture within the femoral neck

 

PREFERRED RESPONSE 1

 

A displaced femoral neck fracture in a young patient is considered a surgical urgency and prompt anatomic reduction and internal fixation is recommended. There are a few studies that have specifically looked at the rate of osteonecrosis in this patient population. A review of femoral neck fractures in patients ages 15 to 50 years revealed that the incidence of osteonecrosis in displaced fractures was 27% compared with 14% in nondisplaced fractures. The quality of the reduction also influenced the rate of osteonecrosis. Time to reduction, type of implant, presence or absence of capsulotomy, and location of the fracture are not associated with osteonecrosis risk.

(SBQ12TR.90) A 40-year-old male sustained the injury seen in Figure A, and subsequently underwent the procedure shown in Figure B. One hour post-operatively he starts to complain of pain in the operative leg, and the pain is unchanged with active or passive stretch. The external dressing is released with little resolution of symptoms. His blood pressure is 115/78 mm Hg with compartment pressures in the leg measuring 31 to 35 mm Hg. His ABI index is 1.1 in the leg. What would be the next step in management? Review Topic

 

 

 

  1. MRI angiography of leg

  2. Four-compartment fasciotomy

     

  3. Follow-up examination the following day

     

  4. Continued monitoring and serial examinations

     

  5. EMG study

 

PREFERRED RESPONSE 4

 

The patient is at risk for developing compartment syndrome of the leg. The next most appropriate step would be to support his systemic blood pressure and monitor compartment pressures.

 

A clinical assessment is the diagnostic cornerstone of acute compartment syndrome. However, the intracompartmental pressure measurement has been advocated to help confirm the diagnosis in patients where there remains uncertainty after clinical exam.

An absolute compartment pressure >30 mm Hg or a difference in diastolic pressure and compartments pressure (delta p) <30 mm Hg may help to confirm the necessity for fasciotomy. However, the treatment of early compartment syndrome should be to initially improve the limbs perfusion pressure gradient. This can be done by treating underlying factors such as hypotension, coagulopathy, or vascular compromise due to either a true vascular injury or artificially by external compression. Frequent reassessment is then critical to effectively manage these patients. If clinical diagnosis persists despite these efforts, urgent fasciotomy would be considered.

 

McQueen looked at 116 patients with tibial diaphyseal fractures who had continuous monitoring of anterior compartment pressure for 24 hours. They found that using an absolute pressure of 30 mmHg would have resulted in 50 patients (43%) treated with unnecessary fasciotomies. They conclude using a differential pressure of 30 mmHg is a more reliable indicator of compartment syndrome.

 

White et al. looked at 101 patients with tibial fractures with satisfactory Delta P measurements. THey found that patients with elevated intramuscular pressures >30 mm Hg after tibial fracture do not have a greater incidence of complications than those with low pressures, so long that Delta P <30 mm Hg.

 

Figure A shows a Shatzker V tibial plateau fracture. Figure B shows fixation of fracture seen in Figure A.

 

Incorrect Answers:

Answer 1: MRI scan of the leg is not needed as the ABI=1.1 Answer 2: Fasciotomy would be considered after serial examinations with delta p <

30 mm Hg or persistently elevated absolute compartment pressures.. Answer 3: It would not be appropriate to leave this patient with impending compartment syndrome.

Answer 5: EMG studies help to assess neurological injury. This would not be necessary in the acute setting.

 

(SBQ12TR.57) A 56-year-old right hand dominant attorney falls from standing and sustains the closed injury shown in Figure A. The treating surgeon elects to fix her fracture using a plate and screw construct. Based on the available imaging, which of the following fracture characteristics best justifies this fixation choice? 

 

 

  1. Fracture displacement

  2. Intra-articular fracture extension

     

  3. The fracture extends distal to the coronoid

     

  4. Oblique fracture line

     

  5. Fracture comminution

 

PREFERRED RESPONSE 5

 

This patient has a displaced, intra-articular, comminuted olecranon fracture. Comminution is an indication for plate fixation.

 

Most displaced olecranon fractures are treated operatively. Options include tension band constructs, intramedullary screws, plate and screw fixation or fragment excision with triceps advancement. Any construct relying on inter-fragmentary compression (tension band, intramedullary screws) requires a non-comminuted fracture pattern. Plate fixation is indicated in the setting of comminution, extension past the coronoid, or in the setting of associated instability.

 

Bailey et al. retrospectively reviewed 25 patients who underwent plate fixation of displaced olecranon fractures. Twenty-two of 25 patients had good or excellent outcomes. Five of 25 patients (20%) of patients required plate removal for symptomatic hardware. The authors concluded that plate fixation was an effective treatment for displaced olecranon fractures, with good functional outcomes.

 

Figure A shows a displaced, comminuted olecranon fracture without evidence of propagation past the coronoid.

 

Incorrect answers:

Answer 1. All displaced olecranon fractures should be considered for internal fixation. Displacement alone does not direct choice of implant. Answer 2. Intra-articular extension is an indication for surgery to re-establish articular

congruity but does not dictate implant selection. Answer 3. Extension distal to the coronoid is an indication for plate fixation but there is no evidence of such extension on the radiograph shown Answer 4. This fracture is comminuted, without a distinct fracture line.

 

(SAE09FA.26) A 45-year-old man is seen in the emergency department after returning from a 2-hour airplane flight. He is reporting severe pain in his right leg but has no trouble moving his ankle, leg, or knee. Venous doppler testing reveals no evidence of deep venous thrombosis. He is placed on IV cephazolin but continues to worsen. On the third day in the hospital he has increased pain, some respiratory distress, and trouble maintaining his blood pressure. His leg takes on the appearance seen in Figure 15. An urgent MRI scan shows thickening of the subcutaneous tissues and superficial swelling in the leg but no evidence of an abscess. What is the next most appropriate step in management? Review Topic

 

 

 

  1. Triple antibiotic coverage

  2. Transfer to the ICU and a consult with infectious disease

     

  3. Urgent irrigation and debridement with gentle skin closure

     

  4. Urgent hyperbaric oxygen treatments and immunoglobulin

     

  5. Urgent aggressive debridement of skin, subcutaneous fat, and fascia

 

PREFERRED RESPONSE 5

The patient has necrotizing fasciitis, a rare and sometimes fatal disease that has many different etiologies. Signs that this is not a normal infection are the worsening clinical symptoms despite IV antibiotics and the systemic symptoms. He needs urgent surgical care before he becomes completely septic and unstable. He needs very aggressive debridement of his tissues. Hyperbaric oxygen and immunoglobulins are only anecdotally helpful, and would only be used after surgery.

 

(SAE12TR.29) Figure 29 is the radiograph of a 30-year-old man who sustained an isolated tibial shaft fracture. What is the most common deformity with nonsurgical management? 

 

 

 

  1. Varus

  2. Malrotation

     

  3. Valgus

     

  4. Valgus and procurvatum

     

  5. Valgus and recurvatum

 

PREFERRED RESPONSE 1

 

Studies have shown that approximately 25% of diaphyseal fractures of the tibia with intact fibulae will go onto varus malunion if treated nonsurgically. Limb-length

discrepancies are also common. Here the fibula acts as a strut, preventing valgus collapse but predisposing to varus collapse. Valgus and procurvatum is the typical deformity in proximal tibial fractures.

 

(OBQ14.135) A 25 year-old-male sustains a closed injury shown in Figure A. If a tibial intramedullary nail is placed with the starting points shown (arrows), what subsequent alignment will occur? 

 

 

 

  1. Neutral

  2. Varus, apex anterior

     

  3. Varus, apex posterior

     

  4. Valgus, apex anterior

     

  5. Valgus, apex posterior

 

PREFERRED RESPONSE 4

 

In proximal third tibial shaft fractures, due to the deforming forces of the pes anserine and the extensor mechanism, utilizing standard starting points during intramedullary nailing (IMN) will result in a valgus and apex anterior deformity.

 

There are several tips and tricks to avoid subsequent deformity following tibial IMN of a proximal third fracture. One way to avoid deformity is to use a more lateral starting point than normal to ensure nail placement in the true center of the canal, which is more lateral when compared to the tibial plateau.

Walker et al. studied 12 cadaveric tibias and inserted a Kirschner wire depending on rotated views of the knee. In order obtain a perfect starting point, a perfect anteroposterior as well as lateral of the knee must be obtained; otherwise, the authors noted that malrotation is bound to occur. With a perfect view, a more lateral starting point correlated with the center of the tibial canal.

 

McConnell et al. studied cadaveric and subsequent radiographic correlation on a lateral knee x-ray to determine the ideal 'safe zone' for the starting point of a tibial nail. This safe zone is more lateral and posterior, when looking at the axial cut of the plateau.

 

Figure A exhibits a proximal third tibia fracture with starting points that are not lateral enough, and too distal (on the lateral view), which will result in apex anterior and valgus deformity.

 

Incorrect answers:

Answer 1 (neutral alignment) will occur if a more lateral and more proximal start point is used. Answers 2,3, and 5 are deformities that will typically be seen following tibial IMN if desired start points are utilized.

 

(OBQ16.32) A surgeon contemplates performing a hemiarthroplasty (HA) or total hip arthroplasty (THA) for an active, community ambulating 70-year-old female with a displaced femoral neck fracture. Which of the following is true for these options? 

 

  1. There is no significant difference in operative time when using cemented stems compared with uncemented stems

     

  2. Comparing HA to THA, there is no difference in blood loss

     

  3. Longer term outcomes are better with HA

     

  4. Bipolar HA performed through a direct anterior or lateral approach leads to equivalent patient outcomes as THA

     

  5. Perioperative complication rate is greater after THA than bipolar HA

PREFERRED RESPONSE 5

 

THA has a higher perioperative complication rate than HA.

 

There are a few differences between THA and HA performed in the setting of displaced FNF. THA has a higher dislocation rate, greater blood loss, requires a larger exposure than HA, and the operation is longer. HA often requires reoperation because of progressive acetabular erosion. Patient outcomes and function are greater following THA than HA.

 

Florschultz et al. reviewed current management of FNF. They concluded that CRIF / ORIF is indicated for displaced femoral neck fractures in younger individuals, select active elderly, and medically unfit patients. HA is indicated for lower demand ambulatory older patients. THA is indicated for the active elderly and those with preexisting acetabular disease.

 

Avery et al. reviewed 7-10 year results of an RCT comparing THA with HA. More HA patients had died during follow-up. All surviving patients had polyethylene wear (THA) and acetabular erosion (HA). They concluded that there was lower mortality and a trend towards superior function in patients with a THA in the medium term.

 

Hedbeck et al. performed a RCT comparing HA and THA. At 4 years, there was improved function and quality of life with THA. They recommend THA in elderly, lucid patients with a displaced FNF.

 

Incorrect Answers:

Answer 1: Surgery using cemented stems has longer operative time compared with uncemented stems.

Answer 2: Blood loss is greater in THA compared with HA. Answer 3: Patient function and quality of life is greater after THA at all time points, and the difference is more significant at later time points. Answer 4: Patient outcomes are greater following THA (compared to HA) regardless of approach.

 

(SAE09TR.48) Which of the following is most predictive of a medial side ankle injury in the presence of a fibula fracture above the level of the joint? 

 

  1. Severe medial ankle tenderness

     

  2. Severe medial ankle ecchymosis

  3. Stress radiographs showing the medial clear space measuring 6 mm and the superior joint space measuring 3 mm

     

  4. Inability to ambulate

     

  5. Medial ankle swelling

 

PREFERRED RESPONSE 3

 

Isolated Lauge-Hansen supination-external rotation-type ankle fractures comprise 20% to 40% of ankle fractures and nonsurgical management is effective for managing SER-2 ankle fractures. Tornetta and associates recently showed that medial ankle tenderness, ecchymosis, and swelling are not reliable findings when trying to determine deltoid competence. Stress radiographs showing a medial clear space of greater than 4 mm or one that is also 1 mm greater than the superior joint space, or any lateral talar subluxation are indicative of deltoid incompetence and indicative of a SER-4 ankle fracture.

 

(SBQ12TR.43) A 31-year-old female smoker was involved in a skiing accident approximately 9 months ago and underwent open reduction internal fixation of the radius and ulna at the time of injury. She now returns to the clinic complaining of increasing pain with range of motion and activity. Radiographs from her most recent follow-up can be seen in Figure A. Laboratory tests show ESR, CRP and WBC count to be within normal limits. Which of the following options is the most appropriate next step in management? Review Topic

 

 

  1. Bone scan

  2. Above elbow cast

     

  3. Removable splint

     

  4. Reamed intramedullary nail

     

  5. Iliac crest bone grafting + compression plating

 

PREFERRED RESPONSE 5

 

This patient is presenting with an atrophic non-union of the ulna after open reduction internal fixation for a both bone forearm fracture 9 months ago. The most appropriate next step in management would be iliac crest bone grafting and compression plating of the ulna.

 

The primary issue with an atrophic nonunion is biological. The blood supply is poor and therefore incapable of purposeful fracture healing. Smokers, as in this vignette, are at high risk for nonunion. The treatment of an atrophic nonunion involves improving biology at the fracture site through use of autologous bone graft (e.g. iliac crest) and providing mechanical stability by means of compression plating (e.g. 3.5 mm LC-DCP).

 

dos Reis et al. reports excellent results of 31 cases of diaphyseal forearm fracture nonunions treated with autologous bone grafting and compression plating. Thirty of thirty-one patients went on to bony union within 3.5 months of revision surgery.

 

Nadkarni et al. presented a case series of 11 patients with non-unions of various long bones initially managed with intradmedullary (IM) nail fixation. The authors successfully used locking compression plates while retaining the IM nails in the treatment of the nonunion in all cases.

 

Figure A shows an AP radiograph of a both bone forearm fracture. Figure B shows an

AP and lateral radiograph of an atrophic non-union of the ulnar shaft. Illustration A shows a lateral x-ray of a fully healed radius and ulna after hardware removal 1 year after revision surgery.

 

Incorrect Answers:

Answer 1: With normal WBC, ESR and CRP levels, there is little reason to suspect an infectious non-union, thus, there is no indication for a bone scan at this time. Answer 2 & 3: Immobilization will not treat the underlying issue of biology. Autologous bone grafting is necessary in the treatment of atrophic non-unions. Answer 4: A reamed IM nail does not provide adequate mechanical stability and is not used in the treatment of radius or ulna non-unions.

 

(SAE08OS.156) A 4-year-old child fell from a trampoline and sustained the fracture shown in Figures 54a and 54b. What is the risk of osteonecrosis of the femoral head associated with this fracture? 

 

 

 

1 10%

2 25%

 

3 50%

 

4 75%

 

5 90%

 

PREFERRED RESPONSE 2

Fractures of the femoral neck in children carry a significant risk of osteonecrosis, particularly when significantly displaced or when near the epiphysis. Base of the neck fractures have a 20% to 30% risk of osteonecrosis. Malreduction or later growth disturbance can also occur, resulting in coxa vara in about 15% of patients. Treatment should consist of urgent reduction and stabilization.

 

(SAE09TR.88) Patients in compensated shock (normal vital signs) are thought to be at risk for which of the following? 

 

  1. A primed immune system with an increased risk of a systemic inflammatory response

     

  2. Nothing since they are no longer in uncompensated shock and their vital signs have normalized

     

  3. Higher nonunion rates after fracture fixation

     

  4. Higher infection rates after definitive fracture fixation

     

  5. Higher complication rates after temporizing external fixation of long bone fractures

 

PREFERRED RESPONSE 1

 

Patients who are in compensated shock have normal vital signs but still have hypoperfusion of organ beds such as the splanchnic circulation due to preferential perfusion of the heart and brain. The response to this continued hypoperfusion may be the development of a systemic inflammatory response that may lead to multiple organ failure. The patients are thought to be at risk for a “primed” immune system due to the ongoing stimulation of the immune system and may have an exaggerated response to a second stimulus such as surgery or infection. Other markers of resuscitation should be used besides vital signs to determine when resuscitation has been completed. The use of temporizing fixation has been shown to lower systemic complication rates, and the infection and union rate after staged fixation is not altered.

(SAE10PE.91) A 10-year-old boy is struck by a car and sustains open left tibia and fibula fractures with bone protruding through a 7-cm laceration, multiple deep and superficial abrasions over the anterior leg, and road gravel is present in the wounds. His foot is warm and well-perfused with normal sensation and he has no pain with passive range of motion of the toes. Optimal treatment should consist of Review Topic

 

  1. irrigation and debridement of the fractures and application of an external fixator.

     

  2. irrigation and debridement of the fractures and a reamed intramedullary nail.

     

  3. irrigation and debridement of the fracture and percutaneous Kirschner wire fixation.

     

  4. submuscular plating.

     

  5. reduction and a short leg cast.

 

PREFERRED RESPONSE 1

 

The patient has a grade 2 open fracture and therefore needs wound debridement as a first step, followed by fracture stabilization preferably with an external fixator. A reamed intramedullary nail is not indicated in a 10-year-old child with open growth plates. Submuscular plating is not needed in an open fracture and there is no mention of fracture debridement. Percutaneous Kirschner wires will not provide adequate fracture stabilization, nor will a short leg cast. Flexible nailing should be considered as another form of fixation.

 

(SAE11UE.61) Figure 61 shows the current radiograph of a 69-year-old woman who fell getting out of bed 10 months ago. At the time of injury she was diagnosed with a nondisplaced humeral surgical neck fracture. After 2 weeks of sling immobilization, physical therapy was started for range of motion. She continues to have pain and is unable to elevate her arm. What is the most likely diagnosis? 

 

 

  1. Rotator cuff tear

  2. Posttraumatic glenohumeral arthritis

     

  3. Proximal humerus fracture nonunion

     

  4. Shoulder infection

     

  5. Post fracture stiffness

 

PREFERRED RESPONSE 3

 

The patient has a nonunion at the fracture site. The humeral head fragment has the rotator cuff musculature attached but the head is not united to the humeral shaft. There is no evidence of glenohumeral arthritis or a rotator cuff tear. Given the history, she has no evidence of a shoulder infection at this time. In review of the radiographs, there is an established nonunion; therefore, examination for post fracture stiffness is not possible because the motion is occurring at the nonunion site.

 

(SBQ12TR.96) Interleukin-6 levels have been shown to be a reliable measure of which of the following? 

 

  1. Osteoporosis

     

  2. Severity of injury

  3. Sarcomatous tumor burden

     

  4. Bone turnover

     

  5. Hyperparathyroidism

 

PREFERRED RESPONSE 2

 

Interleukin-6 (IL-6) is a proinflammatory cytokine that is increased according to the level of injury sustained and acts to activate the host immune system. In addition, new literature exists to indicate its high sensitivity and specificity for detecting infection in total joint arthroplasty.

 

Overactivation of the immune system and cytokines can lead to systemic inflammatory response syndrome (SIRS), which results in end-organ damage, including small-vessel vascular damage; this would lead to parenchymal cell death from hypoxic insult.

 

Keel et al. report that immediate and early trauma deaths are determined by brain injuries or significant blood loss, while late mortality is caused by secondary brain injuries and host defense failure. The secondary effects are characterized by local and systemic release of pro-inflammatory cytokines, arachidonic acid metabolites, proteins of the contact phase and coagulation systems, complement factors and acute phase proteins, as well as hormonal mediators.

 

Pape et al. investigated the effect of surgeries as a "second-hit" phenomenon. They found that surgery on days 2-4 was associated with a greater amount of postoperative organ dysfunction than if the secondary surgery was done on days 5-8. They also found a significant association between IL-6 values above 500pg/dL at the time of surgery and development of multiple organ failure.

 

Sears et al. review the effect of the inflammatory response to trauma and the development of complications (death, multiple organ failure). They report that IL-6 and the HLA-DR2 molecules currently appear to have the most potential for use in predicting outcomes in trauma patients.

Illustration A is a diagram that shows some of the effects of IL-6 production. Incorrect Answers:

1,3-5: IL-6 levels are not known to be reliably altered by these factors.

(SAE11OS.129) Figures A and B show the six-month follow-up radiographs of a 62-year-old woman who sustained a hip fracture in a fall. Prior to the fall, the patient was active and had no difficulty with ambulation. The patient underwent open reduction and internal fixation with a sliding hip screw device. She has difficulty with ambulation, continues to walk with a walker, and reports startup pain. What is the most appropriate management at this time? Review Topic

 

 

 

  1. Valgus osteotomy

  2. Removal of the hardware

     

  3. Intramedullary fixation after removal of the hardware

     

  4. Conversion to total hip arthroplasty with a long cementless stem

     

  5. Conversion to total hip arthroplasty with a primary tapered stem

 

PREFERRED RESPONSE 4

 

The radiographs demonstrate a healed fracture with penetration of the screw through the femoral head into the acetabulum as well as osteonecrosis and collapse of the femoral head (Figure 129b). Conversion to total hip arthroplasty with a long stem is necessary to bypass the femoral cortical defects from the screw holes. A primary tapered stem is not appropriate because of the proximal femoral deformity and the stress risers associated with the screw holes. Removal of hardware, valgus osteotomy, and revision of the internal fixation are not appropriate in the presence of the femoral head collapse and acetabular penetration.

 

(OBQ15.182) An 84-year-old female community ambulator with a history of hypertension undergoes a right hip hemiarthroplasty for a femoral neck fracture. When performed in the post-operative period, the timed up and go (TUG) test may be used to predict which patient outcome? 

 

  1. Stair climbing ability

     

  2. Need for a walking aid

     

  3. Implant failure

     

  4. Balance impairment

     

  5. Independent performance of activities of daily living (ADL)

 

PREFERRED RESPONSE 2

 

The timed up and go (TUG) test may be used as a clinical indicator of function and the need for a walking aid in patients treated with hip hemiarthroplasty for femoral neck fracture at 2-year follow-up.

 

Hip fractures are a cause of significant functional decline for elderly patients. Many outcome tests have been developed to prediction function after hip fracture to manage patient expectations and to assist in rehabilitation planning. The TUG test objectively measures functional mobility and dynamic balance. The test is performed by timing the amount of seconds it requires for a patient to stand up from a chair, walk 10ft (3.05m), return to the chair, and sit.

 

Laflamme et al performed a prospective study evaluating the utility of the TUG test to predict functional outcomes in patients undergoing hip hemiarthroplasty for femoral neck fracture. The TUG scores were significantly higher at 4-days and 3-weeks postoperatively in patients requiring a walking aid compared with patients walking independently at two-years. Patients who performed the test in >58s at 4-days postoperatively had an eightfold greater risk of requiring an assistive device.

 

Springer et al prospectively analyzed the unipedal stance test (UPST) with eyes open and closed in healthy subjects to establish normative values for the test across age and gender groups. Performance on the test was found to be age-specific and not related to gender. The UPST is a method of quantifying static balance ability.

 

Kristensen et al studied the relative and absolute inter tester reliability of TUG in patients with hip fractures. The authors found that the TUG has a high interobserver reliability and an improvement by 6.2 seconds for a patient with a baseline of 20s indicates a change in functional mobility.

Video A shows the timed up and go test.

 

Incorrect Answers:

Answer 1. Stair climbing ability - The Lower Extremity Measure (LEM) includes stairs in the assessment. Answers 3, 5. Implant failure, Independent performance of activities of daily living (ADL) - Not predicted by the TUG test. Answer 4. Balance Impairment - Best assessed with the unipedal stance test (UPST).

 

(SAE10PE.79) A 12-year-old girl has the painful foot deformity seen in Figure 79. You advise her that she has juvenile bunions. How do they differ from adult bunions? 

 

 

 

  1. Metatarsus primus varus

  2. Large exostosis

     

  3. Rigidity of the metatarsal phalangeal joint

     

  4. Greater hallux valgus angle than in adult bunions

     

  5. Prominent bursal thickening over the medial eminence

 

PREFERRED RESPONSE 1

 

The hallmark of the juvenile bunion is metatarsus primus varus. Increased flexibility of the first metatarsal phalangeal joint leads to increased deformity. The hallux valgus

angle is less than the adult bunion. Bursal thickenings and prominence of the medial eminence are less in a juvenile bunion.

 

(OBQ14.234) A healthy 39-year-old male presents to clinic with posttraumatic elbow stiffness after a minimally displaced radial head fracture. His injury occurred 4 months ago with no improvement in range of motion despite 10 weeks of supervised physiotherapy. Follow-up radiographs reveal normal osseous anatomy. What is the next best step in treatment? 

 

  1. Intra-articular and extra-capsular cortisone injection

     

  2. Closed manipulation under anesthesia

     

  3. Aggressive home exercise program

     

  4. Continuous passive motion device

     

  5. Static or dynamic progressive elbow splinting

 

PREFERRED RESPONSE 5

 

Supervised exercise therapy with static or dynamic progressive elbow splinting over a 6 month period has shown to have the greatest improvement on DASH scores and functional range of motion (ROM) in patients with post-traumatic elbow stiffness.

 

The goal of treatment in post-traumatic stiffness is to restore a functional range of elbow motion (30° to 130°). Non-operative modalities are considered the first-line of treatment. Aggressive physical therapy has traditionally been advocated. However, the use of static or dynamic progressive elbow splinting with a turnbuckle has shown to provide better functional outcomes. Treatment is usually maintained over a period of 6-12 months. Surgery is considered when nonoperative therapy fails.

 

Gelinas et al. treated 22 patients with an elbow contracture using a static progressive turnbuckle splint for a mean of 4.5 +/- 1.8 months. The mean range of motion improved from 32 - 108, to 26 - 127 degrees (p = 0.0001). Their results suggest that static progressive splinting is an effective modality for postoperative elbow stiffness.

 

Lindenhovius et al. randomized sixty-six patients with post-traumatic elbow stiffness into static progressive elbow splint therapy or dynamic elbow splinting over a 12

month period. There was no significant difference in outcomes between treatment modalities. ROM increased by 40° vs. 39° at six months, respectively. DASH scores improved from 50 vs 45 at enrollment to 32 vs. 25 at six months, respectively.

Illustration A shows an image of a static progressive elbow splint. Incorrect Answers:

Answer 1: Intra-articular and extra-capsular cortisone injection have not shown to improve ROM in this scenario. Answer 2: Closed manipulation under anesthesia may worsen elbow stiffness and cause intra-articular damage. Manipulation causes significant swelling and inflammation with tearing of soft tissues, causing hemarthrosis and additional fibrosis in the joint. Answer 3: Aggressive home exercise program are not effective when formal physiotherapy has failed. Answer 4: Continuous passive motion machines have a limited role in treating established contractures. They do not seem to improve end-range mobility in these patients.

 

(SBQ12TR.94) A 60-year-old otherwise healthy female sustains the fracture in Figure

A. Immediate surgical treatment would most likely prevent which of the following complications? 

 

 

 

  1. Malunion.

  2. Regional osteopenia.

  3. Skin necrosis.

     

  4. Posttraumatic arthritis.

     

  5. Refracture.

 

PREFERRED RESPONSE 3

 

The patient has a calcaneal tuberosity fracture. The Achilles tendon is attached to the displaced fragment, which can cause soft tissue compromise and skin necrosis if not treated promptly.

 

Displaced calcaneal tuberosity and tongue-type calcaneus fractures have a high incidence of wound complications if not treated urgently. Recommended management includes surgical fixation often performed percutaneously. Lag screws are typically directed from the posterior superior tuberosity inferiorly and distal. In cases of skin sloughing and skin necrosis, flaps may be needed.

 

Gardner et al. reviewed 139 tongue-type calcaneus fractures, and found that 21% had some degree of posterior skin compromise. Additionally, there was a statistically significant increase in posterior skin compromise with delayed presentations.

 

Hess et al. reviewed 3 cases of calcaneal tuberosity avulsion fractures that let to skin necrosis because of a delay in treatment.

 

Tornetta reviewed the indications for percutenaous treatment of calcaneus fractures, and found it was successful and most useful in tongue-type calcaneus fractures.

 

Figure A shows a lateral radiograph of a R ankle showing a calcaneal tuberosity fracture. Illustration A shows an AP radiograph of a R ankle following percutaneous reduction and internal fixation of the calcaneus fracture with two cannulated screws. Illustration B shows a lateral radiograph of a R ankle showing interval reduction and fixation of the calcaneus fracture. Illustration C shows a clinical photograph of posterior skin compromise associated with a calcaneal tuberosity fracture.

 

Incorrect Answers:

Malunion, regional osteopenia, posttraumatic arthritis, and refracture risk would not be significantly altered with immediate surgical intervention as opposed to operative management within a reasonable time frame.

(OBQ14.97) A 21-year-old woman is struck by a car and sustains a Gustillo IIIB fracture of the tibia. The wound was debrided and immobilized with an external fixator. Radiographs are shown in Figure A. The soft tissue defect was covered with a free flap. Her recovery was complicated by wound infection with Klebsiella pneumoniae and Escherichia coli. One month after her injury, she underwent intramedullary nailing and placement of an antibiotic spacer measuring 15cm in length. Radiographs are shown in Figure B. At the next stage of surgery 6 weeks later, the surgeon should plan to do all of the following: 

 

 

 

  1. Excise the spacer

  2. Excise the spacer, debride all membranous tissue, perform exchange nailing

     

  3. Excise the spacer, debride all membranous tissue, bone graft the cavity

     

  4. Excise the spacer, preserve all membranous tissue, bone graft the cavity

     

  5. Excise the spacer, preserve all membranous tissue, bone graft the cavity, remove the nail

 

PREFERRED RESPONSE 4

 

The second stage of the Masquelet technique requires removal of the cement bolus, incision into the induced membranes and bone grafting. The existing hardware is preserved where possible as the fracture is still not stable. Bone graft is inserted INTO the membranous cavity, AROUND the nail.

 

The Masquelet staged technique of induced membranes is an option for filling large bone defects up to 25cm in length. This technique protects against autograft resorption, stimulates mesenchymal cell-to-osteoblast differentiation, maintains graft position, and prevents soft tissue interposition. Cement impregnation achieves high local antibiotic concentration without risk of systemic toxicity.

 

Ashman et al. discussed the techniques of addressing bone defects. Options include:

(1) acute limb shortening (up to 4cm in the tibia and humerus, and 7cm in the femur);

(2) distraction osteogenesis for defects up to 10cm long (at 1mm/day with consolidation period of 5days per mm, or total treatment time of up to 60days/cm), (3) autograft (up to 25cm of vascularized fibula, or 3cm of nonvascularized iliac crest),

and (4) Masquelet technique.

 

Taylor et al. reviewed the induced membranes technique. They found that the membrane is well vascularized and composed of type I collagen with fibroblasts with an inner epithelial cell layer. There is a high concentration of VEGF, RUNX2 (CBFA1), TGFß1, and BMP2. The membrane is sutured over bone graft to create a closed pouch. When a nail is present, they note a second internal membrane around the nail, potentially increasing local vascularity and osteoinductive factor concentration.

 

Figure A shows a Gustillo IIIB tibia fracture with a large bone defect held in a temporizing external fixator. Figure B shows the same defect following intramedullary nailing and with a cement spacer placed circumferentially around the nail in the defect.

 

Incorrect Answers

Answer 1: Excising the spacer alone will leave a cavity around the nail. Small amounts of bone form on the inside of the membrane, but without bone grafting, bone will not bridge the gap. Bone graft should be inserted into the cavity at the same sitting as cement removal. Answers 2 and 3: The membrane prevents bone graft resorption and contributes growth factors. Debriding the membrane will negate these effects. Exchange nailing will result in a period of instability and may compromise the membrane. Answer 5: The nail cannot be removed as yet because there is no fracture site stability.

 

(SAE09TR.15) Which of the following studies best increases the ability to diagnose femoral neck fractures in patients with femoral shaft fractures? 

 

  1. MRI

     

  2. Fine-cut CT scan

     

  3. Bone scan

     

  4. AP radiograph of the femur

     

  5. AP radiograph of the pelvis

PREFERRED RESPONSE 2

 

Tornetta and associates and Yang and associates found that nearly half of all femoral neck fractures associated with femoral shaft fractures were being missed at their institution. On the basis of the delayed diagnosis of these injuries, a best-practice protocol was developed by the attending trauma surgeons for the evaluation of the femoral neck in patients with a femoral shaft fracture. This protocol includes a preoperative AP internal rotation radiograph of the hip, a fine-cut (2-mm) CT scan through the femoral neck (as a part of the initial trauma scan), and an intraoperative fluoroscopic lateral evaluation of the hip just prior to fixation of the femoral shaft. In addition, postoperative AP and lateral radiographs of the hip are made in the operating room to specifically evaluate the femoral neck before the patient is awakened. They found that fine-cut CT (2 mm was the best screening tool in this group of patients) identified 12 of the 13 fractures, whereas 8 of the 13 fractures were visible on the dedicated preoperative AP internal rotation hip radiographs.

 

(SAE12TR.26) Figure 26 is the radiograph of a 33-year-old woman who was involved in a high-speed motor vehicle crash. Her initial blood pressure is 80/50 mm Hg and she has a pulse rate of 120 bpm. After hemodynamic stabilization and temporizing measures have been performed, the patient is cleared for surgery. What is the most appropriate method of definitive fixation? 

 

 

 

  1. External fixation

  2. Open reduction and internal fixation of the pubic symphysis with a two-hole plate

  3. Open reduction and internal fixation of the pubic symphysis with a two-hole plate and posterior triangular osteosynthesis

     

  4. Open reduction and internal fixation of the pubic symphysis with a multi-hole plate

     

  5. Open reduction and internal fixation of the pubic symphysis with a multi-hole plate and posterior plate osteosynthesis

 

PREFERRED RESPONSE 4

 

The patient has sustained an anterior posterior compression (APC) grade II pelvic ring injury. Initial management should consist of pelvic volume reduction with pelvic binding or sheeting. Once the patient is hemodynamically stable, the decision for definitive management should be made. In a retrospective review of more than 200 patients, Sagi and Papp investigated plate osteosynthesis of the pubic symphysis. They found significantly fewer malunions in the multi-hole plate group and a trend toward fewer surgeries in the same group. Typically external fixation should be reserved for temporary fixation and not a definitive management in stable patients. Posterior fixation is reserved for injuries with disruption of the posterior ligamentous constraints, typically APC grade III injuries. Triangular osteosynthesis is a strategy for fixation of unstable vertical shear fractures that require fixation of the pelvis to the lumbar spine.

 

(OBQ14.132) Which of the following activities produces greater hip joint contact pressures than full weight bearing during normal gait? 

 

  1. Performing isometric hip exercises

     

  2. Getting on a bed pan

     

  3. Ambulating with a cane

     

  4. Rising from a chair using the affected leg

     

  5. Toe-touch weight bearing with passive hip abduction

 

PREFERRED RESPONSE 4

Rising from a seated position on a chair on the affected leg has been shown to create the highest contact pressure within the hip, even higher than full weight-bearing during walking.

 

Limiting activities that create high contact pressures is important in situations such as after internal fixation of an acetabular fracture. Full weight bearing during a normal gait cycle is often considered too much contact pressure and considered a risk for early failure of fixation. During the postoperative period weight bearing and activities are limited to prevent this. It has been shown that the highest contact pressures, even higher than normal walking, are seen when rising from a chair on the affected leg.

 

Brand et al. analyzed joint reactive forces in patients walking with and without a cane. Compared to age matched controls they estimate that using a cane decreases the contact pressure in the hip to about 60% of normal.

 

Hodge et al. looked at data from an implanted hip prosthesis with pressure sensors. They found that some activities common to the early rehabilitative period, such as using a bed pan and performing isometric exercises about the hip, can create pressure approaching those of normal walking. The highest pressures recorded were when rising from a chair.

 

Incorrect answers:

Answers 1 and 2: Isometric hip exercises and using a bed pan cause contact pressure approaching that of normal walking, but not exceeding it. Answer 3: Ambulating with a cane reduces the contact force of the hip to about 60% of normal walking. Answer 5: Toe touch weight bearing with passive hip abduction significantly decreases the contact pressures in the hip. Toe-touch weight bearing with passive abduction creates even lower joint reactive force than complete non-weight bearing status.

 

(OBQ14.218) A 40-year-old man is thrown off his motorcycle and sustains an open Type IIIA fracture shown in Figure A. He is taken to the operating room for debridement and reamed intramedullary nailing with a 10mm diameter nail. He returns at 10 months with persistent pain at the fracture site with ambulation. Examination reveals healed wounds with no erythema, warmth or tenderness. Erythrocyte sedimentation rate and C-reactive protein levels are within normal limits. Radiographs taken at that time are shown in Figure B. What is the next best treatment step? 

 

 

  1. Adjunctive plate fixation without nail removal

  2. Nail removal and plating

     

  3. Partial fibulectomy at the same level as the tibia fracture and weightbearing cast application

     

  4. Exchange nailing

     

  5. Local administration of rhBMP-2

 

PREFERRED RESPONSE 4

 

This man had a mid-diaphyseal tibial fracture that has gone into nonunion. Reamed exchange nailing is indicated.

 

Tibial delayed union can be defined as lack of union from 20-26 weeks post-injury, while nonunion is defined as lack of healing at >9mths post-injury, or absence of progressive signs of healing on radiographs for 3 consecutive months. Persistent pain is a symptom of nonunion. ESR and CRP are performed to rule out infection.

 

Bhandari et al. performed a blinded, multicenter trial on 622 reamed tibial nails and 604 unreamed tibial nails. In closed fractures, patients in the unreamed nail group were at greater risk of primary events than the reamed nail group. There was no difference in groups for open fractures. Primary events were defined as bone-grafting, implant exchange/removal, dynamization, and debridement.

 

Hak reviewed aseptic tibial nonunion. They discuss exchanged reamed nailing for diaphyseal nonunion, adjunctive plate fixation for metaphyseal nonunion, and nail removal and plating for metadiaphyseal nonunion, external fixation for infected nonunion and distraction osteogenesis of defects.

 

Figure A shows a mid-diaphyseal tibial fracture Figure B shows nonunion following

IM nailing of the fracture. Illustration A shows union following exchange nailing with a larger 12mm diameter nail.

 

Incorrect Answers:

Answer 1: Adjunctive plate fixation improves mechanical stability and is indicated for hypertrophic nonunions. It functions best for metaphyseal nonunion. Answer 2: Nail removal and plating is an option for metadiaphyseal nonunion. In this location, exchange nailing is unlikely to improve mechanical stability. Further, the plate can be used to correct deformity, and other techniques can be used concurrently (interfragmentary lag screws, dynamic compression plates, push-pull compression devices, articulated tension device). Answer 3: Partial fibulectomy at the same level will destabilize the construct. This is compounded by removal of internal fixation and placement of an external cast. Answer 5: rhBMP-2 has been approved for open tibial fractures managed with IM nailing, but not for tibial nonunion. In contrast, rhBMP-7 is not FDA approved, but has been approved under Humanitarian Device Exemption for use for recalcitrant long bone nonunions where autograft is unfeasible and other treatments have failed.

 

(SAE08UE.113) Figure 55 shows the radiograph of a 30-year-old man who sustained a closed comminuted fracture of the right clavicle. Examination reveals decreased sensation in the radial nerve distribution. Weakness is noted with shoulder abduction, internal rotation, and wrist extension. A displaced bone fragment is most likely pressing on what portion of the brachial plexus? Review Topic

 

 

 

  1. C5 and C6 spinal roots

  2. Superior trunk

     

  3. Anterior division of the inferior trunk

     

  4. Posterior cord

     

  5. Lateral and posterior cords

 

PREFERRED RESPONSE 4

 

Clavicular fractures are occasionally complicated by injury to the brachial plexus. A displaced bone fragment pressing on the posterior cord proximal to the upper subscapularis nerve would account for these findings.

 

(SAE09TR.64) A 45-year-old male karate instructor sustained the injury shown in Figures 40a through 40c while practicing karate. The decision to proceed with surgery depends on which of the following factors? 

 

 

 

  1. MRI scan

  2. Physical examination

     

  3. Workers’ compensation status

     

  4. Surgeon availability

  5. Patient age

 

PREFERRED RESPONSE 2

 

The most important criteria in determining the need for surgery following a nondisplaced or minimally displaced tibial plateau fracture is knee stability to varus/valgus stress. Soft-tissue injury noted on MRI may be addressed at a later time following fracture healing. This fracture pattern is amenable to nonsurgical management. Decisions regarding surgical intervention may be made up to 2 weeks after injury.

 

(SAE09TR.61) A 30-year-old man falls off a 7-foot ladder and sustains the injury seen in the radiograph and the CT scan shown in Figures 39a and 39b. Medical history is negative. Management of this injury should include which of the following? 

 

 

 

  1. Closed treatment and casting

  2. Open reduction and internal fixation

     

  3. Primary subtalar arthrodesis

     

  4. Percutaneous fixation

     

  5. External fixation

 

PREFERRED RESPONSE 2

A Sanders type 2 intra-articular calcaneus fracture in a young healthy nonsmoker is best treated with open reduction and internal fixation. Whereas nonsurgical management is an option, Buckley and associates have shown that these fractures have a better outcome with surgical care. Percutaneous fixation is reserved for tongue-type fractures and subtalar arthrodesis is used in some type 4 fractures. External fixation has not been shown to be advantageous in closed fractures.

 

(SAE11UE.118) A 60-year-old woman with a history of osteoporosis fell from a standing height and sustained a supracondylar distal humerus fracture with an intercondylar extension. Which of the following plate constructs yields the highest stiffness for fixation of the fracture? 

 

  1. Single posterior Y plate

     

  2. Single medial plate with bicortical locking screws

     

  3. Dual plating with medial and posterolateral LC-DCP

     

  4. Dual plating with medial and posterolateral one third tubular plates

     

  5. Dual plating with medial and lateral LC-DCP

 

PREFERRED RESPONSE 5

 

Optimal treatment of distal humeral fractures relies on reestablishment of a congruent articular surface with a fixation construct that is stable enough to allow for early range of motion. Several biomechanical studies have been performed to evaluate the biomechanical strength of various plating configurations. These studies have shown that dual plate configurations are more stable than single plates, regardless of the type of plate used. One third tubular plates have been shown to be significantly weaker than LC-DCP or reconstruction plates, resulting in weaker constructs, and clinically higher rates of hardware failure and nonunion. Whereas traditional teaching has suggested plating in perpendicular planes, recent biomechanical studies have demonstrated that parallel medial and lateral plates confer a greater rigidity to the construct than perpendicular plating schemes.

 

(SAE11OS.89) What is the best indication for prosthetic radial head arthroplasty following fracture? 

 

  1. Mason type I fracture with full range of motion

     

  2. Mason type I fracture with decreased supination

     

  3. Mason type I fracture with decreased pronation

     

  4. Mason type III fracture with associated interosseous membrane injury

     

  5. Mason type III fracture without associated interosseous membrane disruption

 

PREFERRED RESPONSE 4

 

The Mason classification differentiates the degree of displacement, angulation, and mechanical block to motion. Most nondisplaced radial head fractures (Mason I) in which there is no block to motion can be treated nonsurgically. Mason type III injuries are severely comminuted radial head fractures. Fragment excision can be considered in unreconstructable fractures in which the interosseous membrane is intact. However, if the interosseous membrane has been disrupted, fragment excision can lead to proximal migration of the radius with associated wrist problems. In this case, radial head arthroplasty is indicated. Radial head arthroplasty may also be required when the radial head fracture is associated with other ligamentous injuries as seen following an elbow dislocation, or with an associated unstable coronoid fracture.

 

(SAE08OS.134) Disadvantages of anterior-inferior plate fixation for acute clavicular fractures relative to superior plating include 

 

  1. more prominent implants leading to higher rate of reoperation for implant removal.

     

  2. a higher nonunion rate.

     

  3. inferior fixation due to shorter screw lengths laterally.

     

  4. an increased need to detach the deltoid origin.

  5. an increased risk for injury to subclavian structures.

 

PREFERRED RESPONSE 4

 

Anterior-inferior plate fixation of midshaft clavicular fractures has evolved to be an alternative plate location compared to superior plating. The advantages of anterior-interior plating are reduced prominence of the hardware compared to the subcutaneous superior plates; the potential for placement of longer screws as the clavicle is wider front to back than top to bottom, especially laterally; and a potential for decreased risk to the subclavian structures. A relative disadvantage of anterior-inferior plating is a need to detach a small portion of the deltoid origin. Union rates for anterior-inferior plating are similar to those with superior plating.

 

(SAE10PE.33) An 18-month-old child was involved in a motor vehicle accident and sustained an isolated injury to the left upper extremity. A radiograph is shown in Figure 33. What is the most appropriate management for this injury? 

 

 

 

  1. Hanging arm cast

  2. Closed reduction with flexible intramedullary nail fixation

     

  3. Coaptation splinting and bandaging the arm to the thorax

     

  4. Closed reduction and external fixation

     

  5. Locking plate fixation

 

PREFERRED RESPONSE 3

Humeral shaft fractures in infants and young children heal rapidly and have excellent remodeling potential. Appropriate treatment in this age group is immobilization with a coaptation splint and bandaging the arm to the thorax for comfort. Internal fixation is appropriate in multiple trauma, and external fixation may be useful when soft-tissue injury is extensive.

 

(SAE09TR.4) Figures 4a and 4b show the radiographs of a 53-year-old woman who was injured in a fall. After initial closed reduction, what is the preferred treatment for this fracture? 

 

 

 

  1. Open reduction and internal fixation of the radial head and immobilization

  2. Medial collateral ligament repair

     

  3. Radial head replacement, ulnar nerve transposition, and external fixation

     

  4. Coronoid repair, radial head replacement, and lateral ligamentous repair

     

  5. Nonsurgical management in a hinged elbow brace

 

PREFERRED RESPONSE 4

 

This elbow fracture-dislocation involves a radial head fracture, coronoid fracture, and ulnohumeral dislocation (terrible triad). Several algorithms exist for treatment; surgical treatment is indicated. The treatment should address the radial head. Studies have shown replacement to be superior to repair in comminuted fractures. The coronoid may be addressed in unstable cases at the time of radial head excision and replacement. Lateral ligamentous repair is carried out during closure of the lateral elbow capsule. Medial ligamentous repair also may be undertaken but usually in concert with bony repair. Hinged external fixation remains an option when instability exists following bony and soft-tissue repair. Acute ulnar nerve transposition is rarely indicated.

 

(SAE08AN.7) During placement of an external fixator for a distal radius fracture, the most commonly injured nerve is a branch of which of the following nerves? Review Topic

 

  1. Ulnar

     

  2. Median

     

  3. Superficial radial

     

  4. Lateral antebrachial cutaneous

     

  5. Medial antebrachial cutaneous

 

PREFERRED RESPONSE 3

 

Pin track infections and sensory injuries are among the most common complications of external fixation for distal radius fractures. The proximal pins of most distal radius external fixators are placed in the “bare area” of the distal radius, about four finger-breadths above the radial styloid. This corresponds to the area where the dorsal sensory branch of the radial nerve penetrates the fascia dorsal to the brachioradialis tendon to become a subcutaneous structure. Injury to the superficial radial nerve may produce painful dysesthesias and neuromas.

 

(SBQ12TR.106) A 67-year-old female sustains the injury shown in Figure A after a trip and fall. When discussing the outcomes of surgery with the patient, which of the following statements is true? 

 

 

  1. Post-surgical mortality rates are significantly lower after total hip arthroplasty compared to hemiarthroplasty

  2. Internal fixation shows better outcomes (reoperation rate, functional status, and/or complication rates) compared to arthroplasty

     

  3. Bipolar hemiarthroplasty shows better outcomes (reoperation rate, functional status, and/or complication rates) compared to unipolar hemiarthroplasty

     

  4. A delay in surgery greater than 48 hours is recommended if the patient has multiple medical comorbiditiesm which are not fully optimized

     

  5. Dislocation rates are equivalent between total hip arthroplasty and hemiarthroplasty

 

PREFERRED RESPONSE 4

 

Moderate evidence supports that hip fracture surgery within 48 hours of admission is associated with better outcomes. However, patients with significant medical comorbidity should be fully optimized before surgery.

 

Although several studies have shown a benefit to surgery within 48 hours, no definitive time frame has been elucidated. The majority of literature has shown improved outcomes in regards to pain, complications, and length of stay with early surgery. Patients with significant medical comorbidities have been shown to have the highest mortality rates.

 

Moran et al. aimed to determine whether a delay in surgery for hip fractures had an affect on postoperative mortality among elderly patients. In an observational study of 2660 patients, they showed that mortality following hip fracture surgery was 9% at 30-days, 19% at 90-days, and 30% at 12-months. Patients with medical comorbidities had 2.5 times the risk of death within 30-days of surgery. In addition, individuals who had surgery delayed beyond 4 days had increased mortality at 90-days and 12-months.

 

Papakostidis et al. examined the timing of internal fixation of intracapsular fractures of the neck of femur on the development of late complications, particularly osteonecrosis of femoral head (ONFH) and non-union. They showed no benefit of

early surgery on incidence of AVN. However, delay of internal fixation of more than

24 hours showed increased rates of non-union.

Figure A shows a displaced right femoral neck fracture. Incorrect Answers:

Answer 1: Patients treated with hemiarthroplasty or total hip arthroplasty demonstrated no statistically significant difference in mortality. Answer 2: Arthroplasty consistently has better outcomes compared to internal fixation for the treatment of unstable/displaced (Garden III and IV) femoral neck fractures in elderly patients.

Answer 3: Outcomes of unipolar and bipolar hemiarthroplasty for unstable femoral neck fractures are similar. Answer 5: Dislocation rates are higher with total hip arthroplasty compared to hemiarthroplasty.

 

(OBQ13.85) A 34-year-old female is involved in a high-speed motor vehicle collision and sustains a traumatic proximal forearm amputation. She successfully undergoes debridement and closure, and six weeks later, is fitted with her temporary prosthesis. In order to optimize her outcomes upon returning to work as a secretary, which of the following is recommended? 

 

  1. Obtaining formal functional capacity testing

     

  2. Waiting for final prosthesis fitting prior to full release

     

  3. Minimize use of her prosthetic while at work

     

  4. Allowing return to work when full elbow range of motion is seen

     

  5. Offer outpatient psychological counseling

 

PREFERRED RESPONSE 5

 

Upper extremity trauma has serious, acute psychological effects that can linger long after the physical injury. These effects may negatively affect patient-reported outcomes, and may also be associated with worsening pain complaints. Coping and stress management techniques can be reviewed with formal psychological counseling, and should be offered to all patients who have underwent an amputation.

Richards et al surveyed 34 patients who had emergency upper extremity surgery and found high levels of psychological distress in patients, including 29% with high levels of both depression and post-traumatic stress disorder (PTSD). They also found that disability was strongly related to pain, depression, and PTSD symptoms.

 

Mallette et al assessed the attitudes of hand surgery patients and hand surgeons regarding psychologic influences on illness and compared their attitudes with those of the general population. They found that surgeons underestimated the openness of patients to discuss psychological issues and that patients believed in the strong effect of psychologic factors on healing and pain.

Illustration A shows a myoelectric prosthesis in a military veteran. Incorrect Answers:

Answer 1: Formal function capacity testing is not typically necessary unless Workers' Compensation is involved or formal disability proceedings occur. Answer 2: Final prosthetic fitting is not necessary for full release. Answer 3: Prosthetic use will vary according to needs and patient factors such as pain. Answer 4: Return to work does not have to wait for full return of elbow range of motion.

 

(SAE08OS.158) What is the best option for treatment of the fracture shown in Figure 55? 

 

 

 

  1. A 135-degree sliding hip screw device

  2. A 150-degree sliding hip screw device

     

  3. A cephalomedullary intramedullary nail

  4. External fixation using hydroxyapatite-coated pins

     

  5. Hip arthroplasty

 

PREFERRED RESPONSE 3

 

According to Haidukewych and associates, treatment of reverse obliquity trochanteric femoral fractures with 95-degree fixed angle plates or with cephalomedullary intramedullary nails provides results superior to sliding hip screw devices. Moroni and associates demonstrated that external fixation with hydroxyapatite-coated pins provides satisfactory results for complex trochanteric fractures. Hip arthroplasty is not indicated for reverse obliquity fractures because this procedure still requires healing of the greater trochanteric fracture fragment to the shaft.

 

(SAE11OS.54) A patient who underwent intramedullary nailing of a femoral shaft fracture 2 weeks ago now reports groin pain. What is the next most appropriate step in management? 

 

  1. Obtain a radiograph of the hip

     

  2. Obtain radiographs of the lumbar spine

     

  3. Obtain an MRI scan of the lumbar spine

     

  4. Review the radiographic report from the time of injury

     

  5. Reassure the patient that the pain will improve and order physical therapy

 

PREFERRED RESPONSE 1

 

Whereas ipsilateral fractures of the femoral neck and shaft are uncommon, it is critical to recognize a femoral neck fracture that may occur in conjunction with a femoral shaft fracture. The combined injury is seen in 2% to 9% of femoral shaft fractures and may initially be missed in as many as one third of the cases. Preoperative examination of a thin cut CT scan and dedicated AP internal rotation views of the femoral neck can help identify this injury. In addition, the intraoperative AP and lateral hip fluoroscopic view should be examined, and a dedicated radiograph of the hip obtained at the conclusion of the surgery. At follow-up, Tornetta and associates has recommended

obtaining a dedicated AP radiograph of the hip with the leg internally rotated 15 to 20 degrees. Because the femoral neck is anteverted, 15 to 20 degrees of internal rotation of the hip offers the best view of the femoral neck. Whereas associated lumbar spine pathology may cause groin pain, the presence of a missed femoral neck fracture must first be ruled out prior to investigating other sources of pain.

 

(OBQ14.120) Prescribing touch (10 to 15 kg) weight-bearing would be most appropriate in the following scenario? 

 

  1. Acute grade II anterior talofibular ligament ankle sprain

     

  2. Partial lateral menisectomy for incomplete radial tear

     

  3. Open reduction internal fixation for comminuted calcaneus fracture

     

  4. Cemented hemiarthroplasty for displaced femoral neck fracture

     

  5. Open reduction internal fixation for transtectal transverse posterior wall fracture

 

PREFERRED RESPONSE 5

 

Touch weight bearing (10 to 15 kg) regimens have shown to minimize joint reaction forces across the hip. This weight bearing restriction should be considered in patients who have undergone open reduction internal fixation of transtectal transverse posterior wall fractures.

 

The definition of touch weight bearing (also known as touch-down weight bearing) is ill-defined in the literature. Published data suggest touch weight bearing to be 10 to 15 kg of load applied to ground by the affected limb or less than 20% of body weight. In contrast, partial weight bearing is reported as 20 to 25 kg or 30% to 50% of body weight. Joint reaction forces across the hip have been shown to be lowest with touch weight-bearing. In this scenario, the foot should be flat against the ground so the flexor and extensor musculature that cross the hip are relaxed. With non-weight bearing restrictions, the musculature across the hip will be contracted, which increases contact pressures and joint reaction forces.

 

Rubin et al. looked at the validity of touch weight-bearing and partial weight bearing regimens. They found that most patients overload the limb up to 50% more than the target weight prescribed.

Lewis et al. showed that maintaining non-weight-bearing position of the involved leg produces increased compressive forces across the hip joint due to activation of the hip flexors compared to restricted weight-bearing.

 

Incorrect Answers:

Answer 1: Acute grade II lower ankle sprain can be managed with immediate full weight bearing as tolerated. Answer 2: Partial lateral meniscectomy for an incomplete radial tear can be managed with immediate full weight bearing as tolerated. Answer 3: Multifragmentary calcaneus fracture open reduction internal fixation should be managed initially with non-weight bearing. Answer 4: Uncomplicated cemented hemiarthroplasty for hip fracture can be managed with immediate full weight bearing as tolerated.

 

(SAE13PE.107) A 6-year-old girl is brought in for orthopaedic examination because she “walks funny” and occasionally trips. Her feet are normal in appearance. Internal rotation of her hips is 60 degrees and external rotation is 40 degrees bilaterally. The thigh-foot angle on the right is -20 degrees, and +10 degrees on the left. What is the source of her gait abnormality? 

 

  1. Increased femoral anteversion

     

  2. Increased femoral retroversion

     

  3. Internal tibial torsion, right

     

  4. External tibial torsion, left

 

PREFERRED RESPONSE 3

 

Normal rotation in the hips should total approximately 100 degrees. Hip internal rotation is usually greater than external rotation, especially in girls. The thigh-foot angle is measured in a prone position and measures the angle the midaxis of the foot subtends with the midline of the thigh. The thigh-foot angle quantifies tibial rotation. Normal adult thigh-foot angle is approximately 10 to 15 degrees. This patient has increased internal rotation of the right tibia compared with the left; this is the likely cause of her gait problem.

 

(SAE12TR.30) Figures 30a and 30b are the radiographs of a 61-year-old man with diabetes who fell from a ladder and sustained an isolated closed fracture. After realignment and splint application, what is the most appropriate next step in management? 

 

 

 

  1. CT scan

  2. Hybrid external fixation

     

  3. Ankle-spanning external fixation

     

  4. Open reduction and internal fixation within 6 to 8 hours

     

  5. Open reduction and internal fixation within 2 to 3 days

 

PREFERRED RESPONSE 3

 

The patient has sustained a high-energy severely comminuted AO/OTA C2 fracture of the distal tibia. This injury is notably fraught with soft-tissue complications that can lead to disastrous clinical results. In general, a staged protocol is now preferred in an effort to avoid these complications and has shown substantial decreases in infection rates and wound healing problems. A CT scan is certainly appropriate for preoperative planning but should be obtained after frame application because the indirect reduction that is achieved improves one's ability to understand the fracture characteristics and morphology. Hybrid external fixation has fallen out of favor because of its limited biomechanic rigidity and clinical results. Open reduction and

internal fixation in the acute phase (6 to 8 hours) or sub-acute phase (2 to 3 days) is difficult.

 

(SAE12FA.67) A 19-year-old woman sustained a displaced talar neck fracture while cliff jumping. The fracture is managed with open reduction and internal fixation. Which of the following best describes the findings in the 2-months postoperative radiographs shown in Figures 67a and 67b, and subsequent treatment plan? Review Topic

 

 

 

  1. There is a positive Hawkins sign, indicating the patient is unlikely to develop osteonecrosis.

  2. There is a positive Hawkins sign, indicating the patient has developed osteonecrosis.

     

  3. Hawkins sign cannot be determined on radiographs; therefore, MRI is required.

     

  4. No Hawkins sign is visible, and therefore the patient is not likely to develop osteonecrosis.

     

  5. No Hawkins sign is visible; therefore, the patient should be kept non-weight-bearing until a Hawkins sign appears

 

PREFERRED RESPONSE 1

 

The radiographs reveal a positive Hawkins sign, a subchondral lucency in the talar dome best seen on a mortise radiograph indicating viability of the talar body. Once a Hawkins sign appears, it is unlikely that that the patient will develop osteonecrosis.

Osteonecrosis is best diagnosed with radiographs. Although MRI can be helpful in assessing the extent of osteonecrosis, it is unnecessary for purely diagnostic purposes. A Hawkins sign typically will appear at 6 to 8 weeks after fracture; however, the absence of a Hawkins sign at that time does not necessarily indicate osteonecrosis. Most authors agree that even in the absence of a Hawkins sign, weight bearing can commence at 10 to 12 weeks after surgery.

 

(SAE08AN.12) A woman with a neck and chest tumor has weakness in the biceps and paresthesias in the thumb. Brachioradialis and infraspinatus function are normal. The lesion is affecting which of the following structures? 

 

  1. C6

     

  2. Upper trunk

     

  3. Middle trunk

     

  4. Posterior cord

     

  5. Lateral cord

 

PREFERRED RESPONSE 5

 

The lateral cord terminates as the musculocutaneous nerve and also contributes sensory fibers to the median nerve. Involvement of the C6 root or upper trunk could potentially cause weakness of the infraspinatus and the brachioradialis. The middle trunk and the posterior cord do not contribute motor fibers to the thumb or sensory fibers to the thumb.

 

SAE12TR.20) Figures 20a and 20b are the radiographs of a 19-year-old woman who was involved in a motor vehicle accident. What mechanism of injury is most consistent with the injury? 

 

 

  1. Vertical shear

  2. External rotation

     

  3. Sagittal translation

     

  4. Lateral compression

     

  5. Anterior posterior compression

 

PREFERRED RESPONSE 4

 

The radiographs show a lateral compression pelvic ring injury with a displaced superior ramus fracture, or tilt fracture. Tilt fractures are most commonly caused by a lateral compression mechanism. These injuries are often seen in female patients and careful examination, including vaginal examination, is required to rule out open fractures. Lateral compression results in internal rotation, not external rotation, of the pelvic ring. Tilt fractures are not commonly seen with anterior-posterior compression injuries or vertical shear injuries. Sagittal translation is not a term used to describe pelvic ring injuries.

 

(SAE12TR.40) A patient with an unstable pelvic ring injury has just undergone an emergent laparotomy and currently has a packed abdomen. Stabilization of the pelvic ring is performed with an anterior external fixator. What is an advantage of using an external fixator with pins in the iliac crest rather than pins in the anterior inferior iliac spine? 

 

  1. Greater pelvic ring stability

     

  2. Lower risk of pin tract infection

  3. Less reliance on fluoroscopy for pin placement

     

  4. Better ability to control a posterior pelvic injury

     

  5. Less likely to interfere with future incisions for definitive pelvic internal fixation

 

PREFERRED RESPONSE 3

 

There are relative advantages to both types of these external fixators. A frame based on the iliac crest is oftentimes easier to place rapidly because it is less dependent on fluoroscopy. This is also advantageous in this clinical scenario because the patient may not be on a radiolucent table. A frame with pins in the anterior inferior iliac spines may be advantageous in that the pin sites will be away from any future needed incisions if an ilioinguinal approach is needed. There is, however, a higher risk of lateral femoral cutaneous nerve injury or intra-articular pin placement at the hip joint with this frame configuration. This technique is generally more dependent on fluoroscopy for pin placement. Some biomechanic studies have shown advantages to AIIS-based frames but this does not give a definite clinical advantage because neither frame alone is adequate to definitively treat an unstable associated posterior pelvic ring injury. There is no known difference in pin site infection rates between these frame types.

 

(SAE11OS.131) During a percutaneous plating of a proximal tibia fracture requiring a 13-hole minimally invasive locking plate system, the placement of the distal most screws should be done through a small open incision to avoid injury to what structure? 

 

  1. Superficial peroneal nerve

     

  2. Saphenous nerve

     

  3. Posterior tibial artery

     

  4. Peroneal artery

     

  5. Peroneal tendons

 

PREFERRED RESPONSE 1

The superficial and deep peroneal nerves are consistently at risk near the distal holes of long locking proximal tibia plates but can be avoided with a small open incision for those screws. The peroneal tendons are more posterior at that level. The saphenous nerve is medial. The peroneal artery runs behind the fibula and is not at risk. The posterior tibial artery is posterior to the tibia.

 

(SAE12TR.46) Which of the following is the major blood supply to the heel pad? 

 

  1. Lateral calcaneal artery

     

  2. Lateral malleolar artery

     

  3. Artery of the sinus tarsi

     

  4. Artery of the tarsal canal

     

  5. Medial calcaneal branch of the posterior tibial artery

 

PREFERRED RESPONSE 5

 

The medial calcaneal branch of the posterior tibial artery is the major vascular supply to the heel pad. Heel pad avulsions are severe injuries associated with high-energy trauma and often carry a poor prognosis because of the potential for heel pad necrosis. The lateral calcaneal artery and the lateral malleolar artery, along with the lateral tarsal artery, provide perfusion to the lateral flap associated with a standard extensile approach to the calcaneus. The artery of the tarsal canal is a branch of the posterior tibial artery, and the artery of the sinus tarsi is a branch of the perforating peroneal artery. Both provide perfusion to the talus.

 

(OBQ13.82) A 68-year-old patient undergoes total knee arthroplasty for end-stage degenerative joint disease. Two years later, she trips and falls at home and sustains a

fracture seen in Figures A and B. Before her fall, she was a community ambulator and had no knee pain. The component is determined to be stable and the surgeon decides to treat this fracture with closed reduction and retrograde intramedullary fixation with a supracondylar nail. Which of the following statements is true? 

 

 

 

  1. The starting point tends to be more posterior than usual, resulting in hyperextension at the fracture site.

  2. An arthrotomy is not necessary

     

  3. A high-speed carbide burr is usually necessary to enlarge the box for nail entry.

     

  4. The backup plan should include devices that allow multiple points of fixation in the distal segment, such as dynamic condylar screw and fixed angle blade plate.

     

  5. The backup plan should include devices that resist varus collapse, such as condylar buttress plates.

 

PREFERRED RESPONSE 1

 

The patient has a cruciate-retaining (CR) prosthesis. The starting point for nail entry is more posterior than normal because of the femoral component. This leads to hyperextension at the fracture site.

 

Periprosthetic femur fractures above total knee implants occur in 2% of patients. It is important to note: (1) pre-injury function, to determine if the prosthesis was loose, (2) the type of implant (CR vs posterior stabilized, PS) as a PS implant with a closed box would make retrograde intramedullary nailing more difficult (the surgeon has to consider the size of the box vs size of the nail, and if the box is smaller than the nail,

must be prepared to enlarge the box with a metal-cutting burr, which has inherent problems of introducing wear debris into the joint), (3) pre-fracture radiographs help determine the position of the implants (flexion-extension, varus-valgus). These fractures can be treated with non-locking condylar buttress plates (not recommended today), fixed angle devices and intramedullary nailing.

 

McLaren et al. describe 7 osteopenic patients (mean age, 61yrs, range 47-84yrs) treated with retrograde supracondylar nailing. They suggest not reaming, and placing 2-3 screws in the distal fragment. This may require leaving the nail protruding by 1cm. They then suggest removing the protruding segment with a burr at the end of the procedure.

 

Haidukewych et al. debate plating vs nailing in a 80yr old osteopenic patient. It may be difficult to introduce retrograde intramedullary nails through the same incision if dense scar tissue is present. On the other hand, most plates require extensive dissection and do not respect the soft tissues and fracture biology, except for LISS plates and nails.

 

Figures A and B show a displaced Lewis and Rorabeck type II periprosthetic fracture. Illustration A shows the technique of retrograde supracondylar nailing. With the knee flexed, the fracture is reduced and the entry point is in the intercondylar notch. Illustration B shows a comparison between PS and CR implants. Note the "box" in the PS implant. This is absent in the CR implant. Illustration C shows the Lewis and Rorabeck classification.

 

Incorrect Answers:

Answer 2: A formal arthrotomy is necessary to protect the polyethylene liner Answer 3: A CR implant does not possess a box, unlike posterior-stabilized (PS) implants. PS femoral components may require box enlargement with a burr if retrograde nailing is planned. Answer 4: The backup plan should include plates which allow multiple points of fixation in the distal segment. These include locking periarticular plates and polyaxial locking plates, and non-locking condylar buttress plates. The DCS and ABP only have

1 (at most 2) point of fixation in the distal segment. Answer 5: The backup plan should include devices that resist varus collapse (especially in cases with medial comminution), such as angle-stable devices (ABP, DCS and locking plates). Non-locking condylar buttress plates will not resist varus collapse.

 

(SAE09TR.99) A 14-year-old boy sustained a 100% displaced distal radius Salter-Harris type II fracture. Neurologic examination demonstrates normal motor

examination and two-point discrimination. He undergoes fracture reduction to the anatomic position with the application of a long arm cast. Postreduction he reports increasing hand and wrist pain with diminution of two-point discrimination to 10 mm over the index and middle fingers over the next several hours after surgery. The cast is bivalved and the padding released relieving all external pressure over the arm. Reevaluation reveals increasing sensory deficit over the affected area. What is the next most appropriate management intervention? 

 

  1. Cast removal and measurement of carpal canal pressure

     

  2. Immediate carpal tunnel release and pinning of the fracture

     

  3. Continued observation

     

  4. Surgical reduction and pinning of the fracture

     

  5. Electromyography/nerve conduction velocity studies

 

PREFERRED RESPONSE 2

 

The patient has an evolving acute carpal tunnel syndrome. Initial management for this injury is to relieve all external pressure that may elevate the neural compression. Surgical decompression of the median nerve at the carpal tunnel is the optimal intervention. Further nonsurgical interventions (cast removal or further bivalving) are insufficient to alleviate the neural compression.

 

(SAE12FA.56) Figures 56a through 56c are the lateral radiograph and MRI scans of a 32-year-old woman who reports a 3-week history of heel pain, tenderness, swelling, and onset following an increase in running activity. What is the most likely diagnosis? 

 

 

  1. Plantar fasciitis

  2. Atrophic heel pad

     

  3. Achilles tendinitis

     

  4. Retrocalcaneal bursitis

     

  5. Stress fracture of the calcaneus

 

PREFERRED RESPONSE 5

 

The sagittal T1-weighted MRI scan reveals a linear streak of low signal intensity consistent with a stress fracture of the posterior calcaneal tuberosity. The surrounding area of hypointensity on the T1 MRI scan and the hyperintensity on the T2 MRI scan represent bone contusion, hemorrhage, and edema within the calcaneus, with an unremarkable radiograph. The plantar fascia, Achilles tendon, retrocalcaneal bursa, and heel pad all display normal signal in the accompanying MRI scans.

(SAE09TR.68) The major benefit of irrigation with a castile soap solution over irrigation with bacitracin solution for the treatment of the open fracture shown in Figure 42 can be seen in which of the following outcomes? 

 

 

 

  1. Decreased rate of postoperative infection

  2. Decreased rate of nonunion

     

  3. Decreased rate of primary wound healing problems

     

  4. Decreased rate of reoperation

     

  5. Increased rate of bone healing

 

PREFERRED RESPONSE 3

 

The mainstay of early treatment of open fractures includes irrigation and debridement. Prior to the development of antibiotics, this was traditionally accomplished with some form of detergent irrigation. Antibiotic irrigation has been in favor more recently but has mixed scientific results related to its use. Results of at least one major study show the use of a nonsterile liquid soap additive (castile soap) is at least as effective as the use of bacitracin with regards to the rate of postoperative infection and fracture healing, and shows a significant decrease in problems with soft-tissue healing.

 

(SBQ12TR.52) A 42-year-old male presents to your clinic for the first time with the radiographs seen in Figure A. He sustained the injury 4 weeks ago while skiing overseas and treatment was provided by the local orthopaedic surgeon. The operative note states that he sustained an Gustilo Type I open fracture. After surgical fixation of

this type of injury, what is the most common complication requiring reoperation? 

 

 

 

  1. Chronic elbow instability

  2. Post-traumatic arthritis

     

  3. Infection

     

  4. Heterotopic ossification

     

  5. Loss of elbow range of motion

 

PREFERRED RESPONSE 5

 

This patient sustained a terrible triad elbow fracture-dislocation. Reduced range of motion of the elbow joint is the most common complication REQUIRING reoperation with these injuries.

 

Terrible triad elbow fracture-dislocations are characterized by posterolateral dislocation/lateral collateral ligament (LCL) injury, radial head fracture and coronoid fracture. Displaced fractures result in elbow instability. Acute radial head stabilization, coronoid open reduction and internal fixation, and LCL +/- medial collateral ligament (MCL) repair/reconstruction is considered the most appropriate treatment for displaced fractures. Operative complications include elbow stiffness, recurrent instability, arthritis, failure of hardware, heterotopic ossification, posterior interosseous nerve palsy and infection.

 

Egol et al. looked at the functional outcomes of 27 patients that underwent fixation of terrible triad injuries. At one year follow-up, the average flexion-extension arc of elbow motion was 109 degrees +/- 27 degrees, and the average pronation-supination arc was 128 degrees +/- 44 degrees. Grip strength averaged 72% of the contralateral extremity. Although operative fixation led to functional elbow stability, results were poor.

 

They included a reference to McKee et al. to highlight that intra-articular fractures of

the elbow have high rates of stiffness. While not specific to terrible triads, they looked at the effectiveness of the posterior elbow approach in 25 patients that underwent internal fixation of intra-articular distal humerus fractures. They showed poor outcomes at a mean follow-up of 36 months with reduced range-of-motion, decreased strength and high re-operation rates.

 

Figure A shows AP fluoroscopic image of a terrible triad injury that has undergone operative fixation. The radial head and coronoid have undergone open reduction internal fixation, and the MCL bony avulsion has been repaired.

 

Incorrect Answers:

Answer 1: Chronic elbow instability is more common following type I or II coronoid fracture when not operatively managed. Answer 2: Post-traumatic arthritis results from chondral damage at time of injury and/or residual instability. Answer 3: Infection is more prevalent with open fractures, however Type I injuries are usually not associated with increase in infection rates. Answer 4: Heterotopic ossification is a common complication after fixation of these injuries. However, it does not always necessitate reoperation.

 

(SAE12TR.92) Figure 92 is the radiograph of a 45-year-old man who was thrown from his horse and now reports groin pain. Which of the following is the most common long-term sequelae of this injury? 

 

 

 

  1. Gait abnormality

  2. Sexual dysfunction

     

  3. Chronic low back pain

     

  4. Quadriceps weakness

  5. Posttraumatic osteoarthritis

 

PREFERRED RESPONSE 2

 

The radiograph reveals an anterior posterior compression injury to the pelvic ring which is commonly seen after horseback riding injuries. In a large series of patients with this type of injury, 18 of 20 patients had sexual dysfunction after sustaining this injury. Posttraumatic osteoarthritis of the sacroiliac joints may occur, but is less common in this type of injury. Chronic low back pain, gait abnormalities, and quadriceps weakness are not typically seen with this type of injury.

 

(SAE12TR.61) Figure 61 is the radiograph of a 34-year-old woman who was involved in a rollover motor vehicle accident. On arrival at the trauma center she is hypotensive and tachycardic. An abdominal CT scan reveals a spleen laceration. The patient remains hypotensive despite resuscitation and is taken to surgery for an emergent laparotomy and splenectomy. After surgery her delta base is -9 mmol/L. What is the most appropriate management of her pelvic ring injury? Review Topic

 

 

 

  1. Application of a pelvic binder

  2. Application of skeletal traction

     

  3. Open reduction and internal fixation

     

  4. Placement of percutaneous iliosacral screws

  5. Placement of an anterior pelvic external fixator

 

PREFERRED RESPONSE 2

 

The patient has a displaced iliac wing and bilateral rami fractures with superior migration of the right hip. To prevent further deformity, the patient's right hip should be placed into skeletal traction. A pelvic binder may worsen the deformity because of the iliac wing fracture. Anterior external fixation is problematic with an iliac wing fracture. Although the patient will eventually require open reduction and internal fixation and/or placement of iliosacral screws, she must be stabilized and resuscitated prior to this treatment.

 

(SBQ12TR.80) A 37-year-old male arrives to the trauma slot following a high-speed motorcycle crash. His Glasgow Coma score is 14 and his only orthopaedic injury is exhibited in Figure A. His current vital signs are a BP of 90/60, HR 120, and a lactate of 2.5 mMol/L. Other findings include a grade 1 splenic laceration and bilateral pulmonary contusions seen on chest radiograph. Which of the following has been suggested as an indication to perform damage control orthopedic care? 

 

 

 

1 HR >110

  1. Bilateral pulmonary contusions seen on chest radiograph

     

  2. SBP = 90mmHg

  3. Unilateral femur fracture

     

  4. Lactate = 2.5 mMol/L

 

PREFERRED RESPONSE 2

 

Pulmonary contusion severe enough to be diagnosed on chest radiograph alone is an indicator that the patient may benefit from damage control orthopaedics (DCO).

 

Despite the patient's overall stable nature, suffering pulmonary injury severe enough to be seen on x-ray alone suggests that temporary stabilization with staged definitive fixation may avoid potential morbidity.

 

Pape et al. review the evolution and balance of early total care (ETC) and DCO. Summarizing the literature, the authors report several standalone indicators that would justify DCO regardless of stable status. This includes: Injury Severity Score of greater than 40, Injury Severity Score of greater than 20 with chest trauma, multiple injuries with severe pelvic/abdominal trauma/ hemorrhagic shock, bilateral femoral fractures, pulmonary contusion noted on radiographs alone, hypothermia of less than 35 degrees C), and a head injury with an Abbreviated Injury Score of 3 or greater.

 

Figure A exhibits a right femoral shaft fracture. Illustration A exhibits a summarized table stating the criterion used to determine the condition of a polytrauma patient. (Table from Pape et al, PMID: 19726738)

 

Incorrect answers:

Answer 1: Heart rate alone (even elevated severely) is not an indicator to implement DCO.

Answer 3: A systolic BP of 90 mmHg is not an indicator to implement DCO. A SBP of 70mmHg or lower may indicate a patient in extremis (along with other parameters) indicating the benefits of DCO. Answer 4: A unilateral femur fracture alone in a stable patient is not an indicator of DCO; a patient with bilateral femur fractures, however, may benefit from DCO. Answer 5:A lactate of 2.5 mMol/L, while slightly elevated, does not warrant DCO. A lactate value of greater than 6 alone would indicate the need for DCO.

 

(SAE12TR.6) Figures 6a and 6b are the radiographs of a thin 23-year-old man who sustained a closed injury to his left arm in a fall. He has no other injuries and his

neurologic examination is normal. What is the most appropriate treatment? Review Topic

 

 

 

  1. Intramedullary nailing

  2. Hanging arm cast for 6 weeks

     

  3. Shoulder immobilizer for 4 to 6 weeks

     

  4. Open reduction and internal fixation

     

  5. Coaptation splinting with conversion to a fracture brace

 

PREFERRED RESPONSE 5

 

The patient is a thin man with an isolated left humerus fracture. The fracture has bony apposition and should be amenable to closed treatment; therefore the most appropriate treatment is coaptation splinting with conversion to a fracture brace. A hanging arm cast is not recommended for a transverse fracture because of the propensity to distract the fragments, especially if left in place for a long period of time. A shoulder immobilizer is not an appropriate treatment for a humeral shaft fracture. A transverse fracture line is sometimes considered a relative indication for surgical treatment if the fragments are distracted, but in this patient, immediate surgical fixation is not warranted in the absence of other indications for surgical treatment.

(SAE11UE.117) A 35-year-old construction worker sustained a midshaft clavicle fracture that developed a hypertrophic nonunion. One year after the injury, it was internally fixed without bone graft. Four months after the surgery he was asymptomatic and he was released to full activity. Five months following surgery, the patient was digging a ditch and he felt pain in the clavicle. The 4-month and 5-month postoperative radiographs are shown in Figures 117a and 117b. What is the most likely cause of this failure? Review Topic

 

 

 

  1. Iliac crest bone graft was not used to augment the fixation

  2. Infection

     

  3. Inadequate strength of the plate

     

  4. Use of superior plating rather than anterior plating

     

  5. Inadequate medial screw fixation

 

PREFERRED RESPONSE 2

 

In this patient, the hardware was intact for 5 months without any evidence of loosening prior to the catastrophic failure. This suggests that the primary cause of nonunion was poor biology rather than insufficient fixation. Biologic compromise can be caused by either infection, poor blood supply, or lack of osteogenic induction cells. Iliac crest bone graft has been used by some for any nonunion of the clavicle, but two studies have shown that bone graft is not necessary to achieve union. Rigid fixation is all that is required. Infection will still complicate any fixation technique. The radiographs show unicortical screw fixation medially, but the construct did not loosen; therefore, it is not the cause of failure.

 

(SAE12TR.93) A 23-year-old woman is involved in a motorcycle accident. She sustains bilateral femur fractures (Abbreviated Injury Score [AIS]=3), an intra-abdominal injury (AIS=3), facial fractures (AIS=2), and a pulmonary injury (AIS=2). What is her Injury Severity Score (ISS)? 

 

1 13

 

2 18

 

3 22

 

4 27

 

5 35

 

PREFERRED RESPONSE 3

 

The ISS is calculated as the sum of the squares of three highest AIS scores from the six body regions, thus this patient's ISS score is 22. The ISS does correlate with mortality, but the ISS does not score multiple injuries to the same body region, hence the bilateral femur fractures score the same as a unilateral fracture. The New Injury Severity Score (NISS) was developed because of this shortcoming of the ISS. The ISS is used in studies to characterize patient injury severity, with a value of 18 or above indicating polytrauma in many studies.

 

(SAE12TR.65) A patient undergoes open reduction and internal fixation of a displaced radial neck fracture. What position should the forearm be in during the approach and during fixation? 

 

  1. Supinated during the approach and neutral for plate application

     

  2. Neutral during the approach and pronated for plate application

  3. Pronated during the approach and neutral for plate application

     

  4. Pronated during the approach and pronated for plate application

     

  5. Pronated during the approach and supinated for plate application

 

PREFERRED RESPONSE 3

 

Pronating the forearm during the surgical approach decreases the danger to the posterior interosseous nerve by moving it away from the surgical field. Placing the plate straight lateral with the forearm in neutral rotation puts the plate in the safe zone that does not articulate with the proximal radioulnar joint to prevent impingement of the plate during forearm rotation.

 

(SAE13PE.109) Figure 109 is the radiograph of an 11-year-old boy who felt a snap in his right hip while jumping hurdles during track practice yesterday. He complains of pain to his right groin region and is walking with a limp. What physical examination test will cause the patient to experience the most discomfort? 

 

 

 

  1. Resisted hip adduction

  2. Resisted hip abduction

     

  3. Resisted hip extension

     

  4. Resisted knee extension

PREFERRED RESPONSE 4

 

The radiograph shows an avulsion fracture from the right anterior inferior iliac spine. This is the site of origin of the rectus femoris tendon. Contraction of the rectus femoris is most pronounced with extension of the knee. The adductor muscles, which would be tested with resisted hip adduction, originate predominantly on the symphysis pubis. The abductors, which would be tested with resisted hip abduction, originate on the outer iliac crest. The hip extensors, which would be tested with resisted hip extension, originate on the posterior iliac crest.

 

(OBQ14.2) A 33-year-old male patient presents with a comminuted open tibia fracture after involvement in a motor vehicle crash. He has a history of smoking but is otherwise healthy. He is given antibiotics, and taken immediately for irrigation and debridement, followed by an un-reamed stainless steel intramedullary nail. Due to bone loss there is a non-circumferential cortical defect measuring 12 mm at the fracture site.

 

All of the following factors in this patient's history and presentation increase his risk for adverse outcome EXCEPT: 

 

  1. High-energy mechanism of injury

     

  2. Use of un-reamed nail

     

  3. Implant material

     

  4. Fracture gap

     

  5. History of smoking

 

PREFERRED RESPONSE 2

 

Of the factors listed only the use of an un-reamed intramedullary nail for an open tibia fracture has not been shown to increase the risk of adverse outcome or need for reoperation.

 

The treatment of open tibia fractures with intramedullary nailing can be complicated by many factors. High energy mechanism of injury, use of a stainless steel nail,

residual fracture gap greater than 1 cm, and a history of smoking have all been shown to increase the risk of adverse outcome. The use of reamed and un-reamed nails for open tibia fractures have been studied, and no significant difference in outcome has been found.

 

Schemitsch et al. present data from a prospective randomized trial of tibia fractures treated with reamed or unreamed intrameduallry nails. They found no difference in risk of adverse outcome between reamed and un-reamed nails in open tibia fractures. They did, however, find an increased risk of adverse outcomes in high-energy mechanisms, use of stainless steel (versus titanium) rods, and a residual fracture gap of greater than 1 cm. They comment that their data did not show a significant increase in risk due to history of smoking, but cite other studies that have demonstrated such a relationship.

 

Bhandari et al. present data from a prospective randomized study of patients with tibia fractures randomized to reamed or un-reamed tibial nails. For closed fractures they found a lower rate of primary events (most commonly need for dynamization) in the reamed group. However, they found no difference in outcomes for either technique in open fractures.

 

Incorrect answers:

Answers 1, 3, 4, 5: Each of these factors have been shown to increase the risk of adverse outcome when treating an open tibia fracture with an intramedullary nail.

 

(SAE12TR.95) A 24-year-old man is involved in a motor vehicle accident at 60 mph. He sustains multiple injuries including an intra-abdominal injury requiring a splenectomy and a closed right femoral shaft fracture. Which variable will best indicate the patient's resuscitation status when deciding whether to proceed with definitive care of the fracture at the conclusion of the laparotomy? Review Topic

 

  1. Heart rate

     

  2. Hematocrit

     

  3. Base deficit

     

  4. Urine output

     

  5. Systolic blood pressure

PREFERRED RESPONSE 3

 

A metabolic parameter such as the base deficit or lactate level has been shown to better reflect the resuscitation status and survival after trauma. Normalization of hemodynamic parameters does not accurately reflect the resuscitation status and a patient can be in compensated shock (occult tissue hypoperfusion) despite normalization of the heart rate and blood pressure. The use of temporizing measures with delayed definitive fracture treatment has been shown to decrease systemic complications in these patients with occult hypoperfusion.

 

(SAE12FA.53) What is the most common complication following surgical treatment of a displaced talar neck fracture? 

 

  1. Osteonecrosis

     

  2. Varus malunion

     

  3. Posttraumatic arthritis

     

  4. Fracture delayed union/nonunion

     

  5. Wound dehiscence/delayed wound healing

 

PREFERRED RESPONSE 3

 

The most frequent complication is posttraumatic arthritis. With talar neck fractures, osteonecrosis is relatively common, occurring in up to 50% of patients. Fracture nonunion occurs in 10% to 12% of patients. Varus malunion can occur with medial comminution. Wound dehiscence and deep infection are much less frequently encountered.

(SBQ12TR.14) Elevated interleukin 6 (IL-6) is most closely associated to which of the following clinical outcomes in orthopedic trauma patients? 

 

  1. Decreased mortality rates

     

  2. Increased mortality rates

     

  3. Decreased osteomyelitis infection rates

     

  4. Increased rhabdomyolysis rates

     

  5. Increased compartmental syndrome rates

 

PREFERRED RESPONSE 2

 

Elevated levels of Interleukin 6 (IL-6) is most closely associated with higher injury severity scores and increased mortality rates in polytrauma orthopaedic patients.

 

Hyperstimulation of the inflammatory system by major trauma is considered to be the key element in the pathogenesis of severe inflammatory response syndrome and multi-organ dysfunction syndrome. IL-6 is a complex acute-reactant cytokinase that is expressed by cells in response to tissue injury. IL-6 levels are associated with injury severity, complications, and mortality. Patients with the most severe injuries have the highest IL-6 serum levels.

 

Sears et al. reviewed the markers of inflammation in major trauma. They suggest that interleukin-6 and human leukocyte antigen-DR class II molecules appear to have the greatest potential for use in predicting the clinical course and outcome in trauma patients. Early identification of traumatic patients, based on inflammatory markers and genomic predisposition, could help to guide intervention and treatment.

 

Pape et al measured the perioperative concentrations of interleukin-6 in sixty-eight blunt trauma patients with non-life threatening pelvic fractures. Release of proinflammatory cytokines were higher in patients undergoing surgical procedures that cause increased blood loss. The release of markers seems to be related to the type and magnitude of surgery, rather than to the duration of the procedure.

Illustration A shows a diagram of the acute inflammatory response after major trauma Incorrect Answers:

Answer 1: Mortality rates have shown to be increased with elevated levels of IL-6 in early trauma.

Answer 3: Osteomyelitis is a complication of open fracture trauma. Elevated levels of IL-6 and CRP are seen with this infection. Answer 4: IL-6 is not commonly measured in rhabdomyolysis. Creatine kinase is commonly elevated with musculoskeletal injury. Answer 5: Elevated IL-6 levels are not closely correlated with incidence incidence of compartmental syndrome.

 

(SAE12TR.100) A 63-year-old woman with osteopenia is struck by a motor vehicle and sustains a Schatzker 2 (AO/OTA Type B) fracture of the lateral tibial plateau. She has 1.5 cm of joint depression and 7 mm of condylar widening. What is the most appropriate surgical fixation for this injury? 

 

  1. Lateral non-locking construct

     

  2. Percutaneous screws

     

  3. External fixation

     

  4. Lateral locking construct

     

  5. Medial and lateral plating

 

PREFERRED RESPONSE 1

 

The patient has a significantly displaced partial articular fracture of the tibial plateau. Surgical treatment is preferred in an effort to restore the axis of the knee, achieve an articular reduction, and allow for repair of commonly associated soft-tissue injuries such as meniscal tears. This requires direct reduction, and fixation should provide subarticular support, interfragmentary compression, and buttress. This is best achieved with an undercontoured lateral nonlocking plate.

Illustration A shows a tibial plateau fracture. Incorrect Answers:

2: Screws alone are unlikely to be adequately stable in this patient.

3: External fixation is not enough to reduce and hold the joint reduced. 4: Locking plates do not provide buttress effect when used in pure locking mode. In addition, locking plates add significant incremental cost to the procedure. 5: A unicondylar injury does not require dual plating.

(SAE08OS.13) Which one of the following lower extremity amputations requires a soft-tissue balancing procedure to prevent deformity following amputation? Review Topic

 

  1. Gritti-Stokes

     

  2. Transtibial

     

  3. Transmetatarsal

     

  4. Knee disarticulation

     

  5. Lisfranc

 

PREFERRED RESPONSE 5

 

The Lisfranc level amputation removes the attachment of the peroneus brevis (base of the fifth metatarsal) and the peroneus longus (base of the first metatarsal), creating a varus deformity due to unopposed overpull by the tibialis anterior and posterior muscles. An anterior tibialis tendon transfer may be necessary. Other possible tissue balancing choices include posterior tibialis transfer and lengthening of the gastrocsoleus complex. Another option is to leave the base of the fifth metatarsal attached to preserve the eversion pull of the peroneus brevis. All the other amputations do not require soft-tissue balancing procedures to prevent deformities. However, adherence to the prescribed surgical techniques for reattachment of major muscle groups is important for optimizing limb strength and function.

 

(OBQ14.73) Figures A and B are post-operative radiographs of a 54-year-old female. In the first 6 months after this procedure, what is the most likely factor for functional impairment in this patient? 

 

  1. Osteonecrosis

  2. Anterior knee pain

     

  3. Re-fracture

     

  4. Hardware failure

     

  5. Non-union

PREFERRED RESPONSE 2

 

A residual deficit in muscle performance and anterior knee pain are expected in the majority of patients at 6 months after surgical fixation of their patella fractures.

 

Anterior knee pain is reported to be a common symptom following treatment of patellar fractures. A likely contributing factor to the anterior knee pain is scarring and tightness of the structures surrounding the knee, as well as patella maltracking due to quadricep/hamstring weakness and/or poor muscle synchrony. Other factors for anterior knee pain may include symptomatic hardware, which may be treated with removal of fixation after union has been achieved.

 

Lazaro et al. looked at the outcome data on thirty patients with isolated unilateral patellar fractures. Anterior knee pain during activities of daily living was experienced by twenty-four (80%) of the patients. The knee extensor mechanism on the injured side had deficits in strength (-41%), power (-47%), and endurance (-34%) as compared with the uninjured side.

 

Lebrun et al. reviewed a series of 40 operatively treated patella fractures and found that at over 6 years postoperatively, significant symptomatic complaints and functional deficits persisted based on validated outcome measures as well as objective physical evaluations. Removal of symptomatic fixation was required in 52% of the patients treated with osteosynthesis, whereas 38% of those with retained fixation self-reported implant-related pain at least some of the time.

 

Figure A and B show AP and lateral radiographs of a comminuted patella fracture treated with a tension band repair construct. The articular surface looks well reduced.

 

Incorrect Answers:

Answer 1: Osteonecrosis of the patella is rare, occurring in less than 5% of patella fractures. It most commonly occurs in the inferior pole of the patella. Answer 3: Re-fracture is rare after osteosynthesis with retained hardware. Answer 4: Hardware failure is more common with poor surgical technique. Short term reported data shows prominent and symptomatic implants as a result of breakage to occur in <50% of patients. Answer 5: Non-union is rare in minimally displaced patella fractures.

 

(SAE12TR.91) Figure 91 is the radiograph of a 20-year-old man who kicked a door while intoxicated. At the emergency department, his leg is placed into a long-leg cast.

After 2 hours, he reports increasing pain, numbness, and tingling in his toes. What is the most appropriate initial treatment? 

 

 

 

  1. Elevate leg on pillows

  2. Administer IV morphine

     

  3. Observation of the patient

     

  4. Bivalve and spread the cast

     

  5. Apply ice to the lower extremity

 

PREFERRED RESPONSE 4

 

The patient appears to have some indications of a compartment syndrome: increasing pain and signs of nerve compression. Tibia fractures also should heighten the suspicion for a compartment syndrome. Two basic mechanisms of compartment syndrome are that an increase in volume occurs in an enclosed space or there is a decrease in size of the space. In this situation, both are likely occurring; post-fracture swelling is occurring within a closed space and if a cast is in place that may constrict the space even more. One way to increase the available space for swelling would be to bivalve and spread the cast. If the extremity has been casted, then it is vitally important that the cast is bivalved and the surrounding soft dressings under the cast be removed so that all external compression of the compartment has been eliminated. In the face of compartment syndrome, elevation of the limb, masking the pain with morphine, application of ice, or observation alone are all inappropriate.

(SBQ12TR.88) When evaluating a fracture dislocation of the elbow, a varus and posteromedial rotation mechanism of injury typically results in what injury pattern? 

 

  1. A fracture of the radial head requiring ORIF

     

  2. A highly comminuted radial head fracture requiring radial head arthroplasty or resection

     

  3. An MCL injury requiring repair

     

  4. A type I avulsion fracture of the coronoid

     

  5. An anteromedial coronoid fracture

 

PREFERRED RESPONSE 5

 

A varus and posteromedial rotation mechanism of injury typically results in a fracture of the anteromedial facet of the coronoid which frequently requires reduction and fixation to restore stability.

 

A varus and posteromedial mechanism of injury about the elbow presents with an injury pattern distinctly different from other injury patterns. A key part of treating this injury pattern is recognizing a fracture of the anteromedial facet of the coronoid, which often requires reduction and fixation to restore stability about the elbow. It is important to recognize this during preoperative planning since this injury typically requires a medial approach.

 

Steinman presents a review article describing coronoid fracture patterns and their mechanisms of injury.

 

Doornberg and Ring present a level 4 review showing that coronoid fracture patterns and their required treatments are predictable based on mechanism of injury. Varus and posteromedial mechanisms were found to reliably create a fracture of the anteromedial facet of the coronoid, and were associated with sparing of the MCL and radial head.

 

Doornberg and Ring also presented a Level 3 review of anteromedial facet cornoid fractures. They found that they could not be adequately visualized and treated from a lateral approach, and that they typically required reduction and fixation to restore adequate stability to the elbow. This stresses the importance of recognizing this injury pattern during preoperative planning.

 

Illustrations A and B are AP and lateral radiographs of an elbow following a varus/posteromedial injury with an anteromedial coronoid facet fracture. Illustration C is a diagram demonstrating fracture lines that create an anteromedial facet fracture fragment. This fracture can be subclassified into three subtypes [anteromedial rim (a), rim plus tip (b), and rim and tip plus the sublime tubercle (c)]

Incorrect answers:

 

Answers 1, 2, 3: Radial head fractures, and MCL injuries are not seen in varus/posteromedial mechanisms. They are seen in terrible triad elbow injuries which are due to valgus and posterolateral rotatory forces. Answer 4: The coronoid fracture seen in these injuries is an anteromedial facet fracture, and not an avulsion fracture of the tip of the coronoid.

 

(SBQ12TR.78) A 67-year-old female patient presents with increasing right hip/thigh pain over the past three months, which is now recalcitrant to anti-inflammatories. There is no history of trauma or constitutional symptoms. Her past medical history consists of hypertension, coronary artery disease, osteoporosis and gastric reflux. Physical examination reveals mild pain at the extremes of range of motion of the hip and a painful right sided limp. A radiograph of the right hip is seen in Figure A. What would be the most appropriate treatment for this patient at this time? Review Topic

 

 

 

  1. Observation only

  2. Referral to physiotherapy

     

  3. MRI spine and hip

     

  4. Total hip arthroplasty

     

  5. Intramedullary femoral nail

 

PREFERRED RESPONSE 5

 

This osteoporotic female patient is presenting with subtrochanteric lateral cortical thickening and hip pain. This is consistent with an insufficiency fracture of the femur secondary to use of bisphosphonate medication for treatment of osteoporosis. The most appropriate treatment would be intramedullary femoral nail fixation.

 

Bisphosphonate medications have been shown to be associated with atypical (subtrochanteric) femur fractures. These patients often have prodromal hip pain and lateral cortical thickening on radiographs prior to fracture. In addition, there has shown to be a significantly increased risk of fracture in the presence of the “dreaded black line” that occurs at the site of thickening.

 

Lenart et al. examined a case series of patients using bisphosphonates for the treatment of osteoporosis. They identified 15 postmenopausal women who had been receiving alendronate for a mean (±SD) of 5.4±2.7 years and who presented with atypical low-energy fractures. Cortical thickening was present in the contralateral femur in all the patients with this pattern.

 

Goh et al. retrospectively reviewed patients who had presented with a low-energy subtrochanteric fractures. They identified 13 women of whom nine were on long-term alendronate therapy. Five of these nine patients had prodromal pain in the affected hip in the months preceding the fall, and three demonstrated a stress reaction in the cortex in the contralateral femur.

 

Figure A shows a right hip radiograph with subtrochanteric lateral cortical thickening. There is mild arthritic changes in the hip. Illustration A shows a bone scan and radiographs of subtrochanteric lateral cortical thickening that resulted in fracture.

 

Incorrect Answers

Answer 1: Symptomatic lateral femoral cortical stress fractures, known as "dreaded black lines", should be treated with early stabilization to prevent subtrochanteric fracture.

Answer 2: Referral to physiotherapy would not be indicated. Answer 3: Symptoms are related to a subtrochanteric stress fracture, which has been identified on radiograph. MRI would not be indicated. Answer 4: This patient has mild arthritis. Treatment with a long bridging femoral stem may stabilize the fracture, However, the gold standard treatment would be intramedullary nailing.

 

(SAE08AN.17) In patients with displaced radial neck fractures treated with open reduction and internal fixation with a plate and screws, the plate must be limited to what surface of the radius to avoid impingement on the proximal ulna? 

 

  1. 2 cm distal to the articular surface of the radial head

     

  2. 1 cm distal to the articular surface of the radial head

     

  3. Within a 90-degree arc or safe zone

     

  4. Within a 120-degree arc or safe zone

     

  5. Within a 180-degree arc or safe zone

 

PREFERRED RESPONSE 3

 

The radial head is covered by cartilage on 360 degrees of its circumference. However, with the normal range of forearm rotation of 160 to 180 degrees, there is a consistent area that is nonarticulating. This area is found by palpation of the radial styloid and Lister’s tubercle. The hardware should be kept within a 90-degree arc on the radial head subtended by these two structures.

 

(SAE08OS.67) A patient has a vertically and rotationally unstable hemipelvis following a motor vehicle accident. An indication for application of an anterior resuscitative pelvic external fixator is made. Two options with regard to pin insertion location are considered as seen in Figure 20. When compared to pins in position A, the pins in position B may be more advantageous because 

 

 

  1. there is less risk to regional neurologic structures.

  2. of superior bone purchase.

     

  3. they more definitively stabilize the posterior fracture.

     

  4. they may be more easily applied in the emergency department setting.

     

  5. there is less risk of intra-articular hip joint penetration.

 

PREFERRED RESPONSE 2

 

Pelvic external fixation can be used for the acute resuscitation of patients with pelvic fractures and for definitive treatment of certain injury patterns. Typically frames are constructed with anterosuperior half-pin placement within the iliac crest. Intracortical placement of these pins may be difficult and erroneous placement may render purchase inadequate. Recently, external fixation of the pelvic ring with half-pin placement into the dense supra-acetabular bone in the region of the anterior inferior iliac spine has gained popularity. Kim and associates, in a biomechanical model, demonstrated that anterior-inferior pin placement was biomechanically superior to conventional anterior-superior pin placement in rotationally and vertically unstable fracture patterns. Haidukewych and associates performed a cadaveric study that demonstrated the lateral femoral cutaneous nerve is at risk within a mean distance of 10 mm from the inferior half-pin site but the femoral nerve and femoral artery are not at risk. The average superior extent of the hip capsule was 16 mm above the joint. They suggested that these pins be inserted at least 2 cm above the hip to avoid potential hip capsule penetration. Poelstra and Kahler described a case during which the lower pins were inserted without the benefit of imaging using only palpable landmarks. However, this technique is better reserved for nonresusitative purposes permitting the use of multiplanar fluoroscopic imaging. Image guidance better ensures proper pin placement within the pelvic cortices, minimizing penetration of the hip joint and sciatic notch. No anterior external fixator, regardless of design or region of application, offers sufficient posterior stability to serve as definitive treatment for vertically unstable pelvic fracture variants.

 

(OBQ13.238) A 58-year-old African-American female who sustained an injury to her upper arm six months ago presents with persistent arm pain. She was initially treated with splinting, with conversion to fracture bracing. She is neurovascularly intact. An injury radiograph and a current radiograph are shown in Figures A and B respectively. What nutritional or metabolic disturbance is the most likely associated with this patient's diagnosis? Review Topic

 

 

 

  1. Vitamin A deficiency

  2. Low serum testosterone

     

  3. Low serum thyroxine

     

  4. Vitamin D deficiency

     

  5. Hypocalcemia

 

PREFERRED RESPONSE 4

 

This patient has sustained a humeral diaphyseal fracture that has gone on to an atrophic nonunion. Vitamin D deficiency is the most likely associated metabolic disturbance.

 

The incidence of nonunion with non-operative management of humeral shaft injuries ranges from 2-10%. Risk factors include vitamin D deficiency (most common), open fractures, segmental injuries, smoking and obesity. The optimal treatment is compression plating with bone grafting, which has been shown to be superior to intramedullary nailing with bone grafting or compression plating alone.

 

Ring et al. reviewed factors that contributed to humeral diaphyseal nonunion after fracture bracing. Fractures in the proximal to middle one-third of the shaft or fractures with a spiral/oblique pattern were more likely to go on to nonunion.

Brinker et al. reviewed 37 low-energy fractures that went onto nonunion. These patients were evaluated by clinical endocrinologists for evaluation of metabolic abnormalities. Thirty-one of the 37 patients (84%) had a metabolic issue, with 68% (25 of 37 patients) having Vitamin D deficiency.

 

Figure A demonstrates a humeral shaft fracture. Figure B demonstrates an atrophic nonunion of the humeral shaft fracture.

 

Incorrect Answers:

Answers 1, 2, 3,5: Deficiencies of these options have not been associated with nonunion

 

(SAE08OS.52) Which of the following is considered a disadvantage of knee disarticulation when compared with transfemoral amputation? 

 

  1. Increased energy expenditure

     

  2. Worse end-bearing surface

     

  3. Asymmetry of the prosthetic knee joint

     

  4. More pain with weight bearing

     

  5. More stump complications in pediatric patients

 

PREFERRED RESPONSE 3

 

The decision to perform a transfemoral amputation versus a knee disarticulation remains controversial because there are advantages and disadvantages to each procedure. One potential advantage of the knee disarticulation is that energy expenditure during walking may be less than that for a transfemoral amputee. Another advantage is that the knee disarticulation provides for an end-bearing residual limb. The knee disarticulation also includes complete preservation of the thigh muscles, leading to better muscular balance, and the bulbous shape of the residual limb leads to increased stability of the prosthesis. Disadvantages of knee disarticulation include limitations in knee-joint prosthetic components and concerns regarding the cosmesis of the prosthesis, as well as the asymmetry of the knee joint with respect to sitting and gait. Because knee disarticulations are relatively uncommon compared to transfemoral amputations, many prosthetists may have less familiarity with the fabrication and fit of this prosthesis. One of the more important issues with respect to knee

disarticulation versus transfemoral amputation is the degree of trauma involving the soft tissue around the knee, as increased tissue trauma may lead to painful residual limbs and eventually the need for revision amputation at a higher level. If the residual limb of the knee disarticulation is unable to accommodate the distal-end weight bearing of the socket, then patients with a knee disarticulation may experience decreased levels of function compared with patients undergoing transfemoral amputation.

 

(SAE08OS.169) A patient has a tibial shaft fracture and is suspected of having a compartment syndrome involving the deep posterior compartment. Associated signs and symptoms would include paresthesias over the 

 

  1. dorsum of the foot and pain with passive toe flexion.

     

  2. dorsum of the foot and pain with passive toe extension.

     

  3. plantar foot and pain with passive toe flexion.

     

  4. plantar foot and pain with passive toe extension.

     

  5. entire foot and pain with active toe extension.

 

PREFERRED RESPONSE 4

 

A compartment syndrome of the deep posterior compartment causes symptoms related to structures running through that compartment. The deep posterior compartment includes the tibialis posterior, flexor hallucis longus, flexor digitorum longus, popliteus muscle, as well as the posterior tibial artery and the tibial nerve. Elevated pressures in this compartment would cause paresthesias in the distribution of the tibial nerve (plantar aspect of the foot) and would cause associated pain with passive stretch of the muscles in the compartment (great toe extension).

(OBQ15.123) A 72-year-old woman falls onto her left hip after tripping over a curb during her daily 3-mile walk. An injury radiograph is shown in Figure A. What is the best long term solution? 

 

 

 

  1. Cannulated screws

  2. Valgus intertrochanteric osteotomy

     

  3. Unipolar hemiarthroplasty

     

  4. Bipolar hemiarthroplasty

     

  5. Total hip arthroplasty

 

PREFERRED RESPONSE 5

 

THA is the best long term solution for displaced femoral neck fractures (FNF) in active elderly patients.

 

The aims of surgery for FNF in elderly patients are immediate pain relief, rapid mobilization, and low complications and revision. THA has best pain relief, fewer reoperations, best survivorship and is most cost-effective but has longer operative/anesthetic time, blood loss, higher infection rate, and potential instability compared with HA.

 

Healy and Iorio examined the optimal treatment for elderly FNF. They compared internal fixation (120 patients) with arthroplasty (HA, 43 patients; THA, 23 patients). There was no different in reoperation or mortality rates between the 2 groups, but arthroplasty was more cost effective, had independent living, and longer interval to reoperation/death. THA had less pain, better function, and lower rates of reoperation than HA, and was most cost-effective. They concluded that THA was the best treatment.

 

Yu et al. performed a meta-analysis of randomized controlled trials to determine whether THA or hemiarthroplasty (HA) was superior. They found that THA had lower risk of reoperation (RR = 0.53), higher risk of dislocation (RR = 1.99), and

higher functional scores at 1 and 4 years. There was no difference in mortality, infection and complication rates.

Figure A shows a displaced left femoral neck fracture. Incorrect Answers:

Answer 1: Cannulated screws may be indicated for nondisplaced fractures (CRIF) or displaced fractures (ORIF) in young patients. But internal fixation is not predictable/ reproducible in all patients (especially elderly patients) because of variations in the condition of articular cartilage, bone stock, fracture angle, comminution, reduction, fixation, comorbidities, nonunion, osteonecrosis, reoperation, mortality, and postoperative rehabilitation.

Answer 2: Valgus intertrochanteric osteotomy (or sliding hip screw) is indicated for displaced fractures with high Pauwel's angle in young patients where it is desirable to avoid arthroplasty.

Answer 3: Unipolar HA is reserved for low demand elderly patients, and is associated with groin pain, hip stiffness, cartilage erosion from oversized / undersized femoral head and protrusio. Answer 4: Bipolar HA may be indicated for patients at risk for instability (neuromuscular/Parkinson’s disease, dementia). It is is associated with groin pain, acetabular cartilage deterioration, osteolysis from PE wear at the femoral head/mobile acetabular component, implant loosening, and early revision surgery.

 

(SAE09TR.92) A 12-year-old girl falls in gymnastics and sustains comminuted midshaft radius and ulna fractures. Closed reduction and cast immobilization are attempted but fracture redisplacement with 20 degrees of angulation occurs. Surgical treatment includes closed reduction and intramedullary fixation of both bones. What is the most common long-term complication for this fracture? 

 

  1. Infection

     

  2. Malunion

     

  3. Loss of forearm rotation

     

  4. Refracture

     

  5. Delayed union/nonunion

 

PREFERRED RESPONSE 3

Healing of forearm fractures in skeletally immature patients is the usual outcome. The use of intramedullary fixation has been reported to result in a lower frequency of refractures when compared to plate osteosynthesis due to the absence of diaphyseal holes after plate removal, which are considered stress risers. Regardless of implant technique, malunion and infection are infrequent. Loss of forearm pronation and supination is a common occurrence in surgically treated fractures due to the higher degree of soft-tissue injury, and periosteal stripping leads to fracture site instability and fracture comminution.

 

(OBQ15.73) A 46-year-old competitive cyclist falls while racing and suffers an isolated fracture as seen in Figure A. He is positioned on a fracture table and a closed reduction maneuver is attempted, unsuccessfully. Which of the following treatment plans is most appropriate? 

 

 

 

  1. Obtain a CT scan intraoperatively to reassess the quality of reduction

  2. Perform additional closed reduction maneuvers until the reduction is adequate for percutaneous fixation

     

  3. As long as the stepoff is less than 7mm in any plane of imaging, it is appropriate to proceed with percutaneous fixation

     

  4. Perform an open reduction and internal fixation using a sliding hip screw

     

  5. Perform an acute total hip arthroplasty through a direct anterior approach since the patient is on the fracture table

 

PREFERRED RESPONSE 4

When standard closed reduction maneuvers using a traction table are unsuccessful, displaced femoral neck fracture in young adults (< 50 years old) should be open reduced prior to fixation.

 

Displaced femoral neck fractures in young patients have created many treatment controversies that are ongoing in the literature due to devastating consequences of poor outcomes, including nonunion and osteonecrosis of the femoral head. Although poorly defined, the quality of reduction is associated with rates of both nonunion and osteonecrosis. A closed reduction that is malangulated (>10 degrees varus/valgus or anteversion/retroversion) or has significant displacement (5 mm or more in ANY view) is unacceptable, and an open reduction should be performed. Of course, this can be very difficult to assess in the operating room, where uncalibrated fluoroscopy with difficult to obtain tangential imaging is heavily relied upon to make this assessment. Accordingly, when the quality of closed reduction is questionable, the best treatment plan is to obtain a better reduction with direct visualization of the femoral neck prior to fixation.

 

A systematic review of the literature by Pauyo, et al. cites numerous studies showing a higher incidence of osteonecrosis of the femoral head in patients with displaced femoral neck fractures treated with unsatisfactory reductions. Furthermore, performing multiple closed reduction attempts is also associated with a higher risk of osteonecrosis.

 

Upadhyay et al. performed a randomized controlled trial of 102 patients with femoral neck fractures treated with closed or open reductions, which were randomized. The groups had similar rates of nonunions and osteonecrosis of the femoral head; however, subanalysis revealed a "poor" reduction was the highest predictor of poor outcome, whether the reduction was attempted open or closed. Interestingly, the quality of reduction was more important than the implant used or the timing of surgery (including surgeries performed > 48h after injury).

 

Figure A shows a pre-operative AP x-ray of the patient's high-energy femoral neck fracture. Illustrations A and B are intraoperative fluoroscopic and post-operative CT scans of this same patient, highlighting that fluoroscopy may "hide" the degree of residual displacement.

 

Incorrect Answers:

  1. While intriguing, this is not currently the standard of practice; additionally, the stem already states that the reduction is "unsuccessful"

  2. Repeated closed reduction maneuvers may further propagate comminution and damage the blood supply to the femoral head, in theory

  3. Any stepoff of 5 mm seen on x-ray is a marker of worse outcomes. Remember, tangential imaging of the femoral neck is difficult to obtain, and if 7 mm is seen, in actuality it may be a larger amount of displacement. Think of the femoral neck as a complex cylindrical tube of bone with asymmetric cortices (e.g. the calcar) - to obtain the perfect fluoroscopic image for measuring maximal displacement, a perfect perpendicular view to this displacement is required, which is very difficult to do before provisional fixation is placed.

5. Total hip arthroplasty is regarded as a salvage procedure for treating a femoral neck fracture in a young adult patient

 

(SAE13PE.93) Figure 93 is the radiograph of 3½-year-old girl who was evaluated for a progressive increase in tibia vara and complains that her feet are turning in. What is the most appropriate course of action? 

 

 

 

  1. Bilateral tibia and fibular osteotomies

  2. Valgus positioning knee-ankle-foot orthosis (KAFO)

     

  3. Use of a Denis-Browne bar

     

  4. Schedule a return visit in 4 months

 

PREFERRED RESPONSE 1

 

The radiograph shows advanced changes in the medial tibial growth plates. The metaphyseal-diaphyseal angle is more than 20 degrees. With these advanced changes, an osteotomy is indicated. Waiting will allow the problem to increase. Bracing (KAFO or night bar) has not been shown effective in advanced Blount disease.

(SAE12TR.37) Which set of patient characteristics has the highest risk of developing osteonecrosis after an intracapsular femoral neck fracture? 

 

  1. 45-year-old woman with a displaced fracture

     

  2. 55-year-old man with a nondisplaced fracture

     

  3. 70-year-old woman with a nondisplaced fracture

     

  4. 70-year-old man with a displaced fracture

     

  5. 85-year-old woman with a displaced fracture

 

PREFERRED RESPONSE 1

 

Loizou and associates prospectively studied 1,023 patients who sustained an intracapsular hip fracture that was treated with internal fixation using contemporary methods. The overall incidence of osteonecrosis was 6.6%. Osteonecrosis was less common for undisplaced (4.0%) than for displaced fractures (9.5%) and in men (4.9%) than women (11.4%) who had a displaced fracture. The incidence of osteonecrosis for those patients younger than 60 years and who sustained a displaced fracture was 20.6%, compared with 12.5% for those aged 60 to 80 years and 2.5% for those older than age 80 years. Barnes and associates reported that late segmental collapse was more common in displaced fractures in women younger than age 75 years than in those older than age 75 years.

 

(SAE08OS.183) A patient sustained a fracture of the left acetabulum. A single axial CT scan from a two-dimensional study is shown in Figure 61. This fracture pattern is best classified as 

 

 

  1. both column.

  2. anterior column.

     

  3. posterior column.

     

  4. transverse.

     

  5. anterior column-posterior hemitransverse.

 

PREFERRED RESPONSE 4

 

A transverse fracture divides the innominate bone into two portions. The fracture plane is horizontal (or semihorizontal) through the acetabulum at a variable proximal distal level. The superior segment retains a portion of the acetabular roof and the lower ischiopubic segment, the intact obturator foramen. A sagittally oriented fracture line is typically seen on axial view assessment.

 

(OBQ14.216) A 33-year-old female sustains the injury shown in Figure A as the result of a fall off a chair, and subsequently undergoes operative stabilization of her injury. Which of the following is most correlated with positive outcomes when treating this injury? 

 

 

  1. Immediate weightbearing

  2. Subchondral debridement of any osteochondral defect

     

  3. Repair of medial ligamentous structures

     

  4. Casting or splinting in a neutral position postoperatively

     

  5. Anatomic reduction of the syndesmosis

 

PREFERRED RESPONSE 5

 

Long-term outcomes after an ankle syndesmotic injury are most correlated with an anatomic reduction of the ankle syndesmosis. Formal open reduction of the syndesmosis has been shown to improve outcomes by improving the reduction quality.

 

While the importance of anatomic reduction of a syndesmotic injury is clear, controversy exists regarding the ideal method of fixation. No significant differences are reproducibly reported in regards to number of syndesmotic screws, size, or number of cortices. There is emerging data supporting the use of suture button fixation.

 

Wikeroy et al. reviewed 48 patients at a mean of 8.4 years, and they found that patients with a difference in the syndesmotic width between the operated and the nonoperated ankle of 1.5 mm or more showed inferior results. Posterior malleolar fragments and obese patients also had worse outcomes.

 

Schepers et al. published a review on the suture button device comparisons to traditional screw fixation, reviewing 6 biomechanical studies and 34 clinical studies. They found that the suture button systems have similar outcomes to screw fixation, but insufficient long-term and high-quality evidence prevented a strong conclusion. Implant removal in the suture button groups averaged 10%, while screw removal averaged 52%.

 

Sagi et al. reviewed 107 patients with ankle fractures and associated syndesmotic

injuries requiring surgery. They found that 39% were malreduced, but open reduction of the syndesmotic injury cut the malreduction rate by 2/3. They also reported that at a minimum of 2 years follow-up, patients with malreduced syndesmotic injuries demonstrated worse functional outcome scores.

Figure A shows an ankle fracture with obvious syndesmotic injury/widening. Incorrect answers:

1-4: These choices are not correlated with excellent outcomes with treatment of a

syndesmotic injury.

 

(SAE12FA.35) An 18-year-old man sustained a traumatic laceration of the common peroneal nerve when glass fell on the outer part of his leg 1 year ago. He has used a molded foot and ankle orthosis for the past 10 months, but would now like surgical intervention. Electromyography shows no function in the anterior or lateral compartments. He has 5/5 muscle strength of the superficial and deep posterior compartments. What is the most appropriate treatment? 

 

  1. Gastrocsoleus recession

     

  2. Subtalar fusion

     

  3. Split anterior tibial tendon transfer

     

  4. Split posterior tibial tendon transfer

     

  5. Flexor hallucis longus tendon transfer

 

PREFERRED RESPONSE 4

 

In a patient with a drop foot and with 5/5 muscle strength of the posterior tibial tendon, a split posterior tibial tendon transfer would be the most appropriate treatment option based on the options presented. The deep peroneal nerve innervates the anterior tibial tendon. This muscle has been affected by the injury; therefore, the anterior tibial tendon cannot be transferred. A subtalar fusion would help correct inversion and eversion deformities, but is not effective for plantar flexion deformities. The foot drop is caused by a neurologic condition in this patient, not a contracture of the gastrocsoleus complex. Therefore, a recession would not be beneficial. A flexor

hallucis longus tendon transfer would not take the deforming force and make it a corrective force.

 

(SAE12TR.74) Which of the following factors is associated with improved outcomes following surgery for hip fractures? 

 

  1. Immediate surgical intervention

     

  2. Early discharge to a skilled nursing facility

     

  3. Choosing spinal versus general anesthesia for surgery

     

  4. Choosing total hip arthroplasty instead of hemiarthroplasty for a displaced femoral neck fracture

     

  5. Correction of metabolic abnormalities prior to surgical intervention

 

PREFERRED RESPONSE 5

 

Many studies have looked at patient outcomes following hip fracture surgery. While early surgery in these patients is recommended, medical optimization prior to surgical intervention is warranted in all cases. Anesthetic type and discharge status have not been proven to alter patient outcomes. Total hip arthroplasty has improved function at 1 year compared with hemiarthroplasty; no changes in mortality have been reported.

 

(SAE12TR.3) Figure 3a is the initial radiograph of a 19-year-old man who sustained a closed clavicle fracture. Figures 3b and 3c show postoperative radiographs. If the patient had been treated nonsurgically, which of the following would most likely occur? 

 

 

  1. Normal shoulder strength and function

  2. Local sensory deficits

     

  3. Fracture union

     

  4. Infection

     

  5. Malunion

 

PREFERRED RESPONSE 5

 

Recent studies comparing surgical treatment with nonsurgical management in displaced clavicle fractures have revealed a decreased rate of malunion and nonunion with surgery. In addition, significant malunions can lead to functional deficits at the shoulder. Thus, with open reduction and internal fixation and anatomic or near-anatomic reduction, there should be a higher likelihood of normal shoulder strength and function. Infection and local sensory deficits would not be expected with nonsurgical management, whereas surgical treatment has a small risk of infection and a high likelihood of sensory deficits from iatrogenic damage to the supraclavicular nerves.

 

(SAE09TR.14) A 26-year-old man falls off a motorcycle and injures his left wrist. There are no open wounds and the neurovascular examination is normal. Radiographs

are shown in Figures 10a and 10b. Definitive management should consist of Review Topic

 

 

 

  1. closed reduction and casting.

  2. external fixation and percutaneous pinning of the distal radius.

     

  3. open reduction and internal fixation of the distal radius.

     

  4. open reduction and internal fixation of the distal radius and open repair of the ulnar styloid.

     

  5. nonbridging external fixation of the distal radius.

 

PREFERRED RESPONSE 3

 

The patient has a high-energy injury with resultant comminution of the distal radius metaphysis. Cast immobilization is likely to lead to radial shortening and angulation due to the comminution. Similarly, while external fixation and pinning has been successful in the past, some loss of radial length and volar angulation is typically noted. Present plate fixation devices for the distal radius employing locking screw technology have a superior ability to resist radial shortening and dorsal angulation. Fixation of the ulnar styloid is warranted when there is distal radioulnar joint instability or significant displacement of the styloid. This is more likely to occur with a fracture at the base of the styloid. In this instance, the distal radioulnar joint does not appear to be disrupted.

(SAE12TR.34) A 65-year-old woman with rheumatoid arthritis is involved in a motor vehicle accident. Her injuries include a right displaced femoral neck fracture, a left open tibial pilon fracture, a left open tibial plateau fracture, multiple rib fractures, and bilateral pulmonary contusions. Her vitals signs on admission are a heart rate of 115 bpm and a systolic blood pressure of 90 mm Hg. Laboratory studies show a hemoglobin of 10.0 g/dL and a delta base of -6.0 mmol/L. What finding in this patient is most significantly associated with increased mortality? Review Topic

 

  1. Heart rate

     

  2. Base deficit

     

  3. Hemoglobin

     

  4. Urine output

     

  5. Systolic blood pressure

 

PREFERRED RESPONSE 2

 

The severity of injuries and the lack of physiologic reserve in this and other elderly patients often result in mortality. Base deficit has shown to be a reliable predictor of mortality even in normotensive elderly blunt trauma patients. Although tachycardia, low systolic blood pressure, and low hemoglobin may all contribute to these patients' mortality, base deficit may be used as a predictor of mortality and a measure of resuscitation.

 

(SAE12TR.98) Clinical staging of osteomyelitis using the Cierney-Mader classification system takes into account which of the following factors? 

 

  1. Age and gender of patient

     

  2. Fracture type and type of bacteria

     

  3. Host status and extent of infected bone

     

  4. Immune status and chronicity of infection

     

  5. Bacterial resistance and source of infection

PREFERRED RESPONSE 3

 

The Cierney-Mader classification system takes into account three types of patients with osteomyelitis: (A) healthy, (B) those with comorbidities, and (C) a host in whom treatment will lead to greater morbidity than the infection. Furthermore, the disease is addressed based on its complexity: type I-medullary, type II-superficial, type III-localized, and type IV-diffuse.

 

(SAE09TR.62) A 24-year-old woman fell from a horse and landed on her outstretched right arm. Radiographs reveal an elbow dislocation with a type II coronoid fracture and a nonreconstructable comminuted radial head fracture. What is the most appropriate management? 

 

  1. Radial head resection, open reduction and internal fixation of the coronoid, and medial collateral ligament repair

     

  2. Radial head resection and lateral collateral ligament repair

     

  3. Radial head arthroplasty alone

     

  4. Radial head arthroplasty and lateral collateral ligament repair

     

  5. Radial head arthroplasty, open reduction and internal fixation of the coronoid, and lateral collateral ligament repair

 

PREFERRED RESPONSE 5

 

The combination of an elbow dislocation and a fracture of the radial head and coronoid is known as a terrible triad injury. To restore elbow stability, each injury must be addressed. The nonreconstructable radial head fracture requires implant arthroplasty. Open reduction and internal fixation of the coronoid is also necessary as is repair of the lateral collateral ligament complex which is usually avulsed from the lateral epicondyle region.